Book 1
Book 1
New York
Contents
Foreword v
Course Summary
1 37 45
1 Making
Economic Decisions
53
67 75
6 Equivalent Annual Worth 7 Internal Rate of Return 8 Benefit Cost Ratios and Other Measures
Benefit cost ratio Future worth Payback period Breakeven Present worth index
89 105 115
iii
13 Income Taxes
Corporate and personal taxes After tax analysis
189
14 Public-Section Engineering Economy 15 Inflation 16 Estimating Cash Flows 17 Sensitivity Analysis 18 Uncertainty and Probability 19 Multiple Objectives
Appendix A-Accounting Effects in Engineering Economy
Compound Interest Tables
iv
Foreword
Over the years I have determined that students often do not have study materials that are truly useful to them. They usually have the course textbook and the lecture notes that they have managed to take or someone has taken for them. Often these lecture notes leave much to be desired in terms of clear, organized, useful information. Occasionally a student will have copies of old exams given by their professor. These exams can be treasure troves of information for the student. For the past several years I have made my old quizzes and exams available to my students and have been told that they are the most useful of all the resources available for studying for an exam. This study guide is the product of the revision of a previous guide and the addition of new material. There are now problems covering every chapter in the companion text, Engineering Economy: Applying Theory to Practice. The majority of them are actual exam problems used in an introductory course in engineering economy. In general, the exam problems are just as they appeared on the actual course examination. Problems were carefully selected for publication so that the fundamentals of engineering economy found in your textbook are covered. This book begins with a summary of basic engineering economy principles. The main section of the book consists of over 375 problems with solutions. The problems offer a wide range of complexity and different solution formats. The last section contains a set of compound interest tables typically used in the solution of engineering economy problems. Thank you to the following engineering economy professors from across the country who have contributed problems to the earlier version of this guide: Hal Ball, B.G. Barr, Charles Buford (deceased), Gary Crossman, Anthony DeFruscio, R. James Diegel, John Eastman, Bryan Jenkins, Jack Lohmann, Hugo Patino, Joseph Pignatiello, Jr., Paul Schonfeld, Barry Shiller, Ralph Smith, and Thomas Ward. I would like to thank Dr. Ted G. Eschenbach for recommending me to the publisher of this guide. And I would like to thank Danielle Christensen, Jenny Boully, and Karen Shapiro all of Oxford University Press for their encouragement and help in preparing this book. A special thanks goes to Michelle Drumwright who re-typed the previous edition when electronic copies of it could not be found. And the biggest thanks of all go to my wife Ellen and daughter Abigail for their patience and understanding while I worked on this project. If you find any errors in the problems or solutions, please inform the editor in care of the Department of Engineering at the University of Tennessee at Martin or by e-mail at [email protected]. Also if you would like to submit a problem or problems for a future edition please use the same addresses. I hope you find this material facilitates a better understanding of engineering economy and that it helps you to succeed in your course! Ed Wheeler Co-editor
COURSE SUMMARY
This chapter is a brief review of engineering economic analysis/engineering economy. The goal is to give you a better grasp of the major topics in a typical first course. Hopefully this overview will help you put the course lectures and your reading of the textbook in better perspective. There are 26 example problems scattered throughout the engineering economics review. These examples are an integral part of the review and should be worked to completion as you come to them.
CASH FLOW
The field of engineering economics uses mathematical and economic techniques to systematically analyze situations that pose alternative courses of action. The initial step in engineering economics problems is to resolve a situation, or each alternative course in a given situation, into its favorable and unfavorable consequences or factors. These are then measured in some common unit -- usually money. Those factors that cannot readily be reduced to money are called intangible, or irreducible, factors. Intangible or irreducible factors are not included in any monetary analysis but are considered in conjunction with such an analysis when making the final decision on proposed courses of action. A cash flow table shows the money consequences of a situation and its timing. For example, a simple problem might be to list the year-by-year consequences of purchasing and owning a used car:
Year Beginning of first Year 0 End of Year 1 End of Year 2 End of Year 3 End of Year 4
Car purchased now for $4,500 cash. The minus sign indicates a disbursement. Maintenance costs are $350 per year
The car is sold at the end of the 4th year for $2,000. The plus sign represents a receipt of money.
Course Summary
This same cash flow may be represented graphically: Receipt of money Disbursement of money $2,000
$4,500
The upward arrow represents a receipt of money, and the downward arrows represent disbursements. The x-axis represents the passage of time. EXAMPLE 1 In January 1999, a firm purchases a used typewriter for $500. Repairs cost nothing in 1999 or 2000. Repairs are $85 in 2001, $130 in 2002, and $140 in 2003. The machine is sold in 2003 for $300. Compute the cash flow table. Solution Unless otherwise stated in problems, the customary assumption is a beginning-of-year purchase, followed by end-of-year receipts or disbursements, and an end-of-year resale or salvage value. Thus the typewriter repairs and the typewriter sale are assumed to occur at the end of the year. Letting a minus sign represent a disbursement of money, and a plus sign a receipt of money, we are able to set up this cash flow table: Year Beginning of 1999 End of 1999 End of 2000 End of 2001 End of 2002 End of 2003 Cash Flow -$500 0 0 -85 -130 +160
Notice that at the end of 1994, the cash flow table shows +160. This is the net of -140 and +300.
Course Summary
If we define Year 0 as the beginning of 1990, the cash flow table becomes: Year 0 1 2 3 4 5 Cash Flow -$500 0 0 -85 -130 +160
From this cash flow table, the definitions of Year 0 and Year 1 become clear. Year 0 is defined as the beginning of Year 1. Year 1 is the end of Year 1. Year 2 is the end of Year 2, and so forth.
EQUIVALENCE
In the preceding section we saw that money at different points in time(for example, $100 today or $100 one year hence) may be equal in the sense that they both are $100, but $100 a year hence is not an acceptable substitute for $100 today. When we have acceptable substitutes, we say they are equivalent to each other. Thus at 8% interest, $108 a year hence is equivalent to $100 today. EXAMPLE 2 At a 10% per year interest rate, $500 now is equivalent to how much three years hence? Solution $500 now will increase by 10% in each of the three years. Now = $500.00 End of 1st year = 500 + 10%(500) = 550.00 End of 2nd year = 550 + 10%(550) = 605.00 End of 3rd year = 605 + 10%(605) = 665.50
Course Summary
Thus $500 now is equivalent to $665.50 at the end of three years. Equivalence is an essential factor in engineering economic analysis. Suppose we wish to select the better of two alternatives. First, we must compute their cash flows. An example would be: Alternative A B -$2,000 -$2,800 +800 +1,100 +800 +1,100 +800 +1,100
Year 0 1 2 3
The larger investment in Alternative B results in larger subsequent benefits, but we have no direct way of knowing if Alternative B is better than Alternative A. Therefore we do not know which alternative should be selected. To make a decision we must resolve the alternatives into equivalent sums so they may be compared accurately and a decision made.
Symbols
i = = = = Interest rate per interest period. In equations the interest rate is stated as a decimal(that is, 8% interest is 0.08). Number of interest periods. A present sum of money. A future sum of money. The future sum F is an amount, n interest periods from the present, that is equivalent to P with interest rate i. An end-of-period cash receipt or disbursement in a uniform series continuing for n periods, the entire series equivalent to P or F at interest rate i. Uniform period-by-period increase in cash flows; the arithmetic gradient. Uniform rate of period-by-period increase in cash flows; the geometric gradient.
n P F
G g
= =
Course Summary
Functional Notation
To Find Single Payment
Compound Amount Factor Present Worth Factor Uniform Payment Series Sinking Fund Factor Capital Recovery Factor Compound Amount Factor Present Worth Factor Arithmetic Gradient Gradient Uniform Series Gradient Present Worth A P G G (A/G, i, n) (P/G, i, n) A A F P F P A A (A/F, i, n) (A/P, i, n) (F/A ,i ,n) (P/A , i, n) F P P F (F/P, i, n) (P/F, i, n)
Given
Functional Notation
From the table above we can see that the functional notation scheme is based on writing (To Find / Given, i, n). Thus, if we wished to find the future sum F, given a uniform series of receipts A, the proper compound interest factor to use would be (F/A, i, n).
Course Summary
The present sum P increases in n periods to P(1 + i) n . This gives us a relationship between a present sum P and its equivalent future sum F: Future Sum =(Present Sum)(1 + i) n F = P(1 + i)n This is the Single Payment Compound Amount formula. In functional notation it is written: F = P(F/P, i, n) The relationship may be rewritten as: Present Sum =(Future Sum)(l + i)-n P = F(1 + i)-n This is the Single Payment Present Worth formula. It is written: P = F(P/F, i, n) EXAMPLE 3 At a 10% per year interest rate, $500 now is equivalent to how much three years hence?
Course Summary
Solution This problem was solved in Example 2. Now it can be solved using a single payment formula. P = $500 n = 3 years F = unknown i = 10%
F = P(1 + i)n = 500(1 + 0.10)3 = $665.50 This problem may also be solved using the Compound Interest Tables. F = P(F/P, i, n) = 500(F/P, 10%, 3) From the 10% Compound Interest Table, read(F/P, 10%, 3) = 1.331. F = 500(F/P, 10%, 3) = 500(1.331) = $665.50 EXAMPLE 4 To raise money for a new business, a man asks you to loan him some money. He offers to pay you $3,000 at the end of four years. How much should you give him now if you want 12% interest per year on your money? Solution F = $3,000 n = 4 years P = unknown i = 12 %
P = F(l + i)-n = 3,000(1 + 0.12)-4 = $1,906.55 Alternate computation using Compound Interest Tables: P = F(P/F, i, n) = 3,000(P/F, 12%, 4) = 3,000(0.6355) = $1,906.50 Note that the solution based on the Compound Interest Table is slightly different from the exact solution using a hand calculator. In economic analysis, the Compound Interest Tables are always considered to be sufficiently accurate.
Course Summary
Solving this equation for A: A = P [i(1 + i)n] / [(1 + i)n - 1] Uniform Series Capital Recovery formula Since F = P(1 + i)n, we can multiply both sides of the P/A equation by(1 + i)n to obtain: (1 + i)nP = A [(1 + i)n - 1] / i which yields F = A [(1 + i)n - 1] / i Uniform Series Compound Amount formula Solving this equation for A: A = F [i /(1 + i)n - 1] Uniform Series Sinking Fund formula In functional notation, the uniform series factors are: Compound Amount(F/A, i, n) Sinking Fund(A/F, i, n) Capital Recovery(A/P, i, n) Present Worth(P/A, i, n)
EXAMPLE 5 If $100 is deposited at the end of each year in a savings account that pays 6% interest per year, how much will be in the account at the end of five years?
Course Summary
Solution A = $100 n = 5 years F = unknown i = 6%
F = A(F/A, i, n) = 100(F/A, 6%, 5) = 100(5.637) = $563.70 EXAMPLE 6 A woman wishes to make a uniform deposit every three months to her savings account so that at the end of 10 years she will have $10,000 in the account. If the account earns 6% annual interest, compounded quarterly, how much should she deposit each three months? Solution F = $10,000 n = 40 quarterly deposits A = unknown i = 1 % per quarter year
Note that i, the interest rate per interest period, is 1 %, and there are 40 deposits. A = F(A/F, i, n) = 10,000(A/F, 1 %, 40) = 10,000(0.0184) = $184 EXAMPLE 7 An individual is considering the purchase of a used automobile. The total price is $6200 with $1240 as a down payment and the balance paid in 48 equal monthly payments with interest at 1% per month. The payments are due at the end of each month. Compute the monthly payment. Solution The amount to be repaid by the 48 monthly payments is the cost of the automobile minus the $1,240 down payment. P = $4960 n = 48 monthly payments A = unknown i = 1% per month
A = P(A/P, i, n) = 4,960(A /P, 1 %, 48) = 4,960(0.0263) = $130.45 EXAMPLE 8 A couple sold their home. In addition to cash, they took a mortgage on the house. The mortgage will be paid off by monthly payments of $232.50 for 10 years. The couple decides to sell the mortgage to a local bank. The bank will buy the mortgage, but requires a 1% per month interest rate on their investment. How much will the bank pay for the mortgage? Solution A = $232.50 n = 120 months P = unknown i = 1% per month
10
Course Summary
Arithmetic Gradient
At times one will encounter a situation where the cash flow series is not a constant amount A. Instead it is an increasing series like: 125 150 175 200
100
n=5 This cash flow may be resolved into two components: 150 175 200 G n=5 p* p' p 2G 3G (n 1)G
100
125
=
G = 25 n=5
We can compute the value of P* as equal to P' plus P. We already have an equation for P' = P' = A(P/A, i, n) The value for P in the right-hand diagram is:
& !1 $ i " n % in % 1) + P # G( ( i 2 1$ i n + ! " * '
This is the Arithmetic Gradient Present Worth formula. In functional notation, the relationship is: P = G(P/G, i, n) EXAMPLE 9 The maintenance on a machine is expected to be $155 at the end of the first year, and increasing $35 each year for the following seven years. What present sum of money would need to be set aside now to pay the maintenance for the eight-year period? Assume 6 % interest.
Course Summary
Solution G = 35 155
11
P = unknown
n = 8 years i = 6%
In the gradient series, if instead of the present sum P, an equivalent uniform series A is desired, the problem becomes:
175 200 G = A* A = 100 + 0 2G 3G (n-1)G
100
125
150
A'
In functional notation, the Arithmetic Gradient (to) Uniform Series factor is: A =G(A/G, i, n) It is important to note carefully the diagrams for the two arithmetic gradient series factors. In both cases the first term in the arithmetic gradient series is zero and the last term is(n - 1)G. But we use n in the equations and functional notation. The derivations(not shown here) were done on this basis and the arithmetic gradient series Compound Interest Tables are computed this way. EXAMPLE 10 For the situation in Example 9, we wish now to know the uniform annual maintenance cost. Compute an equivalent A for the maintenance costs.
12
Solution
Course Summary
G = 35 155
n = 8 years i = 6%
Geometric Gradient
The arithmetic gradient is applicable where the period-by-period change in the cash flow is a uniform amount. There are other situations where the period-by-period change is a uniform rate, g. A diagram of this situation is: An A4 A3 A2 A1
P where An = A1(1 +g)n-1 g = Uniform rate of period-by-period change; the geometric gradient stated as a decimal(8% = 0.08). A1 = Value of A at Year 1. An = Value of A at any Year n.
Course Summary
13
EXAMPLE 11 It is likely that airplane tickets will increase 8% in each of the next four years. The cost of a plane ticket at the end of the first year will be $180. How much money would need to be placed in a savings account now to have money to pay a students travel home at the end of each year for the next four years? Assume the savings account pays 5% annual interest. Solution The problem describes a geometric gradient where g = 8% and i = 5 %.
& 1 % (1 $ g ) n (1 $ i ) % n ) + P # A1 ( ( + i%g ' * & 1 % (108 . ) 4 (105 . ) %4 ) + P # 180.00( ( + 0.05 % 0.08 ' * . & % 0119278 ) # 180.00( + ' % 0.03 * # $715.67
Thus, $715.67 would need to be deposited now. As a check, the problem can be solved without using the geometric gradient: Year 1 2 3 4 A1 = A2 = 180.00 + 8%(180.00) = A3 = 194.40 + 8%(194.40) = A4 = 209.95 + 8%(209.95) = Ticket $180.00 194.40 209.95 226.75
P = 180.00(P/F, 5%, 1) + 194.40(P/F, 5%, 2) + 209.95(P/F, 5%, 3)+ 226.75(P/F, 5%,4) = 180.00(0.9524) + 194.40(0.9070) + 209.95(0.8638) + 226.75(0.8227) = $715.66
14
Course Summary
At beginning of year 1000.00 + 1%(1000.00) 1015.00 + 1%(1015.00) 1030.23 + 1%(1030.23) 1045.68 + 1%(1045.68) = = = = =
Amount in Account $1,000.00 1,015.00 1,030.23 1,045.68 1,061.37
The actual interest rate on the $1,000 would be the interest, $61.37, divided by the original $1,000, or 6.137%. We call this the effective interest rate. Effective interest rate =(1 + i )m - 1 , where i = Interest rate per interest period; m = Number of compoundings per year. EXAMPLE 12 A bank charges 1% per month on the unpaid balance for purchases made on its credit card. What nominal interest rate is it charging? What effective interest rate? Solution The nominal interest rate is simply the annual interest ignoring compounding, or 12(1%) = 18%. Effective interest rate =(1+0.015)12 -1 = 0.1956 = 19.56%
Criteria
Economic analysis problems inevitably fall into one of these categories: 1. Fixed Input The amount of money or other input resources is fixed.
Course Summary
2. Fixed Output
15
Example: A mechanical contractor has been awarded a fixed price contract to air-condition a building. 3. Neither Input nor Output Fixed This is the general situation where neither the amount of money or other inputs, nor the amount of benefits or other outputs are fixed.
Example: A consulting engineering firm has more work available than it can handle. It is considering paying the staff for working evenings to increase the amount of design work it can perform. There are five major methods of comparing alternatives: present worth; future worth; annual cost; rate of return; and benefit-cost ratio. These are presented in the following sections.
PRESENT WORTH
In present worth analysis, the approach is to resolve all the money consequences of an alternative into an equivalent present sum. For the three categories given above, the criteria are: Category Fixed Input Fixed Output Neither Input nor Output Fixed Present Worth Criterion Maximize the Present Worth of benefits or other outputs. Minimize the Present Worth of costs or other inputs. Maximize [Present Worth of benefits minus Present Worth of costs ] or, stated another way: Maximize Net Present Worth.
16
Course Summary
In situations like this, the solution is either to use some other analysis technique(generally the annual cost method) or to restructure the problem so there is a common analysis period. In the example above, a customary assumption would be that a pump is needed for 12 years and that Pump A will be replaced by an identical Pump A at the end of 6 years. This gives a 12-year common analysis period. Pump A Pump B Correct Present Worth Comparison This approach is easy to use when the different lives of the alternatives have a practical least common multiple life. When this is not true(for example, life of J equals 7 years and the life of K equals 11 years), some assumptions must be made to select a suitable common analysis period, or the present worth method should not be used. EXAMPLE 13 Machine X has an initial cost of $10,000, annual maintenance of $500 per year, and no salvage value at the end of its four-year useful life. Machine Y costs $20,000. The fast year there is no maintenance cost. The second year, maintenance is $100, and increases $100 per year in subsequent years. The machine has an anticipated $5000 salvage value at the end of its 12-year useful life. If interest is 8%, which machine should be selected? Solution The analysis period is not stated in the, problem. Therefore we select the least common multiple of the lives, or 12 years, as the analysis period. Present Worth of Cost of 12 years of Machine X = 10,000 + 10,000(P/F, 8%, 4) + 10,000(P/F, 8%, 8) + 500(P/A, 8%, 12) = 10,000 + 10,000(0.7350) + 10,000(0.5403) + 500(7.536) = $26,521 Present Worth of Cost of 12 years of Machine Y = 20,000 + 100(P/G, 8%, 12) - 5,000(P/F, 8%, 12) = 20,000 + 100(34.634) - 5,000(0.3971) = $21,478 Choose Machine Y with its smaller PW of Cost. Replacement Pump A
12 years 12 years
Course Summary
EXAMPLE 14 Two alternatives have the following cash flows: Alternative Year A B 0 -$2,000 -$2,800 1 +800 +1,100 2 +800 +1,100 3 +800 +1,100 At a 5% interest rate, which alternative should be selected? Solution Solving by Present Worth analysis: Net Present Worth(NPW) NPWA NPWB To maximize NPW, choose Alternative B.
17
= PW of benefits - PW of cost = 800(P/A, 5%, 3) - 2,000 = 800(2.723) - 2,000 = $178.40 = 1,100(P/A, 5%, 3) - 2,800 = 1,100(2.723) - 2,800 = $195.30
Capitalized Cost
In the special situation where the analysis period is infinite(n = "), an analysis of the present worth of cost is called capitalized cost. There are a few public projects where the analysis period is infinity. Other examples would be permanent endowments and cemetery perpetual care. When n equals infinity, a present sum P will accrue interest of Pi for every future interest period. For the principal sum P to continue undiminished (an essential requirement for n equal to infinity), the end-of-period sum A that can be disbursed is Pi. P P + Pi P + Pi P + Pi
.
A
When n = ", the fundamental relationship between P, A, and i is: A = Pi Some form of this equation is used whenever there is a problem with an infinite analysis period.
18
Course Summary
EXAMPLE 15 In his will, a man wishes to establish a perpetual trust to provide for the maintenance of a small local park. If the annual maintenance is $7,500 per year and the trust account can earn 5% interest, how much money must be set aside in the trust? Solution When n = ", A = Pi or P = A/i Capitalized cost P = A/i = $7,500/0.05 = $150,000
FUTURE WORTH
In present worth analysis, the comparison is made in terms of the equivalent present costs and benefits. But the analysis need not be made at the present time. It could be made at any point in time: past, present, or future. Although the numerical calculations may look different, the decision is unaffected by the point in time selected. Of course, there are situations where we do want to know what the future situation will be if we take some particular course of action now. When an analysis is made based on some future point in time, it is called future worth analysis. Category Fixed Input Fixed Output Neither Input nor Output Fixed Future Worth Criterion Maximize the Future Worth of benefits or other outputs. Minimize the Future Worth of costs or other inputs. Maximize [Future Worth of benefits minus Future Worth of costs ] or, stated another way: Maximize Net Future Worth.
EXAMPLE 16 Two alternatives have the following cash flows: Alternative Year A B 0 -$2,000 -$2,800 1 +800 +1,100 2 +800 +1,100 3 +800 +1,100 At a 5% interest rate, which alternative should be selected? Solution In Example 14, this problem was solved by Present Worth analysis at Year 0. Here it will be solved by Future Worth analysis at the end of Year 3. Net Future Worth(NFW) NFWA = FW of benefits - FW of cost = 800(F/A, 5%, 3) - 2,000(F/P, 5%, 3) = 800(3.152) - 2000(1.158) = $205.60
Course Summary
NPWB To maximize NFW, choose Alternative B. = 1,100(P/A, 5%, 3) - 2,800(F/P, 5%, 3) = 1,100(3.152) - 2,800(1.158) = $224.80
19
ANNUAL COST
The annual cost method is more accurately described as the method of Equivalent Annual Cost(EAC) or, where the computation is of benefits, the method of Equivalent Annual Benefits(EAB). For each of the three possible categories of problems, there is an annual cost criterion for economic efficiency. Category Fixed Input Fixed Output Neither Input nor Output Fixed Annual Cost Criterion Maximize the Equivalent Annual Benefits. That is, maximize EAB. Minimize the Equivalent Annual Cost. That is, minimize EAC. Maximize [ EAB - EAC ]
20
Course Summary
Problems presented on examinations are often readily solved by the annual cost method. And the underlying continuing requirement is often present, so that an annual cost comparison of unequal-lived alternatives is an appropriate method of analysis. EXAMPLE 17 Consider the following alternatives: First cost Annual maintenance End-of-useful-life salvage value Useful life A $5,000 500 600 5 years B $10,000 200 1,000 15 years
Based on an 8% interest rate, which alternative should be selected? Solution Assuming both alternatives perform the same task and there is a continuing requirement, the goal is to minimize EAC. Alternative A; EAC = 5,000(A/P, 8%, 5) + 500 - 600(A/F, 8%, 5) = 5,000(0.2505) + 500 - 600(0.1705) = $1,650 Alternative B: EAC = 10,000(A/P, 8%, 15)+ 200 - 1,000(A/F, 8%, 15) = 10,000(0.1168) + 200 - 1,000(0.0368) = $1,331 To minimize EAC, select Alternative B.
RATE OF RETURN
A typical situation is a cash flow representing the costs and benefits. The rate of return may be defined as the interest rate where PW of cost = PW of benefits, EAC = EAB, or PW of cost - PW of benefits = 0.
Course Summary
EXAMPLE 18 Compute the rate of return for the investment represented by the following cash flow: Year Cash Flow 0 -$595 1 +250 2 +200 3 +150 4 +100 5 + 50 Solution
21
This declining arithmetic gradient series may be separated into two cash flows for which compound interest factors are available: 250 200 250 250 250 250 250 150 100 50 0 100 150 200
50 =
Note that the gradient series factors are based on an increasing gradient. Here, subtracting an increasing arithmetic gradient, as indicated by the diagram, solves the declining cash flow. PW of cost - PW of benefits = 0 595 - [250(P/A ,i, 5) - 50(P/G, i, 5)] = 0 Try i = 10 %: 595 - [250(3.791) - 50(6.862)] = -9.65 Try i = 12 %: 595 - [250(3.605) - 50(6.397)] = +13.60 The rate of return is between 10% and 12%. It may be computed more accurately by linear interpolation:
& 9.65 % 0 ) Rate of return = 10% $ (2%)( + # 10.83% ' 13.60 % 9.65*
22
Course Summary
Course Summary
23
Since the incremental rate of return exceeds the 5 % MARR, the increment of investment is desirable. Choose the higher-cost Alternative B. Before leaving this example problem, one should note something that relates to the rates of return on Alternative A and on Alternative B. These rates of return, if computed, are: Alternative A Alternative B Rate of Return 9.7% 8.7%
The correct answer to this problem has been shown to be Alternative B, and this is true even though Alternative A has a higher rate of return. The higher-cost alternative may be thought of as the lower cost alternative, plus the increment of investment between them. Looked at this way, the higher-cost Alternative B is equal to the desirable lower-cost Alternative A plus the desirable differences between the alternatives. The important conclusion is that computing the rate of return for each alternative does not provide the basis for choosing between alternatives. Instead, incremental analysis is required. EXAMPLE 20 Consider the following: Alternative Year A B 0 -$200.0 -$131.0 1 +77.6 +48.1 2 +77.6 +48.1 3 +77.6 +48.1 If the minimum attractive rate of return(MARR) is 10%, which alternative should be selected? Solution To examine the increment of investment between the alternatives, we will examine the higher initial cost alternative minus the lower initial-cost alternative, or A - B. Alternative Year 0 1 2 3 A -$200.0 +77.6 +77.6 +77.6 B -$131.0 +48.1 +48.1 +48.1 Increment A-B -$69.0 +29.5 +29.5 +29.5
24
Course Summary
Solve for the incremental rate of return: PW of cost = PW of benefits 69.0 = 29.5(P/A, i, 3) (P/A, i, 3) = 69.0/29.5 =2.339 From Compound Interest Tables, the incremental rate of return is between 12% and 15%. This is a desirable increment of investment hence we select the higher initial-cost Alternative A.
2. 3.
4. 5.
EXAMPLE 21 Consider the following: Alternative Year A B 0 -$200.0 -$131.0 1 +77.6 +48.1 2 +77.6 +48.1 3 +77.6 +48.1 If the minimum attractive rate of return(MARR) is 10%, which alternative, if any, should be selected?
Course Summary
Solution
25
One should carefully note that this is a three-alternative problem where the alternatives are A, B, and Do Nothing. In this solution we will skip Step 1. Reorganize the problem by placing the alternatives in order of increasing initial cost: Do Alternative Year Nothing B A 0 0 -$131.0 -$200.0 1 0 +48.1 +77.6 2 0 +48.1 +77.6 3 0 +48.1 +77.6 Examine the B - Do Nothing increment of investment: Do Year B - Nothing 0 -$131.0 0 = -$131.0 1 +48.1 0 = +48.1 2 +48.1 0 = +48.1 3 +48.1 0 = +48.1 Solve for the incremental rate of return: PW of cost = PW of benefits 131.0 = 48.1(P/A, i, 3) (P/A, i, 3) = 131.0/48.1 = 2.723 . From Compound Interest Tables, the incremental rate of return = 5%. Since the incremental rate of return is less than 10%, the B - Do Nothing increment is not desirable. Reject Alternative B. Next, consider the increment of investment between the two remaining alternatives: Do Year A - Nothing 0 -$200.0 0 = -$131.0 1 +77.6 0 = +77.6 2 +77.6 0 = +77.6 3 +77.6 0 = +77.6 Solve for the incremental rate of return: PW of cost = PW of benefits 200.0 = 77.6(P/A, i, 3) (P/A, i, 3) = 200/77.6 = 2.577
26
Course Summary
From Compound Interest Tables, the incremental rate of return is 8%. Since the rate of return on the A - Do Nothing increment of investment is less than the desired 10%, reject the increment by rejecting Alternative A. We select the remaining alternative: Do nothing! If you have not already done so, you should go back to Example 20 and see how the slightly changed wording of the problem radically altered it. Example 20 required the choice between two undesirable alternatives. Example 21 adds the Do-nothing alternative that is superior to A or B. EXAMPLE 22 Consider four mutually exclusive alternatives: Initial Cost Uniform Annual Benefit A $400.0 100.9 Alternative B C $100.0 $200.0 27.7 46.2 D $500.0 125.2
Each alternative has a five-year useful life and no salvage value. If the minimum attractive rate of return(MARR) is 6%, which alternative should be selected? Solution Mutually exclusive is where selecting one alternative precludes selecting any of the other alternatives. This is the typical textbook situation. The solution will follow the several steps in incremental analysis. 1. The rate of return is computed for the four alternatives. Alternative A B Computed rate of return 8.3% 11.9%
C 5%
D 8%
Since Alternative C has a rate of return less than the MARR, it may be eliminated from further consideration. 2. Rank the remaining alternatives in order of increasing initial cost and examine the increment between the two lowest cost alternatives. Alternative A $400.0 100.9
Initial Cost Uniform Annual Benefit $ Initial Cost $ Uniform Annual Benefit $ Computed $ rate of return
B $100.0 27.7
D $500.0 125.2
Since the incremental rate of return exceeds the 6% MARR, the increment of investment is desirable. Alternative A is the better alternative.
Course Summary
3.
27
Take the preferred alternative from the previous step and consider the next higher-cost alternative. Do another two-alternative comparison. $ Initial Cost $ Uniform Annual Benefit $ Computed $ rate of return D-A $100.0 24.3 6.9%
The incremental rate of return exceeds MARR, hence the increment is desirable. Alternative D is preferred over Alternative A. Conclusion: Select Alternative D. Note that once again the alternative with the highest rate of return(Alt. B) is not the proper choice.
BENEFIT-COST RATIO
Generally in public works and governmental economic analyses, the dominant method of analysis is called benefit-cost ratio. It is simply the ratio of benefits divided by costs, taking into account the time value of money.
B/ C # PW of benefits Equivalent Annual Benefits # PW of cost Equivalent Annual Cost
For a given interest rate, a B/C ratio > 1 reflects an acceptable project. The method of analysis using B/C ratio is parallel to that of rate of return analysis. The same kind of incremental analysis is required.
28
Course Summary
EXAMPLE 23 Solve Example 22 using Benefit-Cost analysis. Consider four mutually exclusive alternatives: Alternative A B C D Initial Cost $400.0 $100.0 $200.0 $500.0 Uniform Annual Benefit 100.9 27.7 46.2 125.2 Each alternative has a five-year useful life and no salvage value. Based on a 6% interest rate, which alternative should be selected? Solution 1. B/C ratio computed for the alternatives: Alt. A
B/ C # PW of benefits 100.9( P/ A, 6%, 5) . # # 106 400 PW of costs PW of benefits 27.7( P/ A, 6%, 5) . # # 117 100 PW of costs PW of benefits 46.2( P/ A, 6%, 5) # # 0.97 200 PW of costs
Alt. B
B/ C #
Alt. C
B/ C #
PW of benefits 125.2( P/ A, 6%, 5) . # # 105 500 PW of costs Alternative C with a B/C ratio less than 1 is eliminated.
Alt. D
B/ C #
2.
Rank the remaining alternatives in order of increasing initial cost and examine the increment of investment between the two lowest cost alternatives. Alternative A $400.0 100.9
Initial Cost Uniform Annual Benefit $ Initial Cost $ Uniform Annual Benefit Incremental B/C ratio #
B $100.0 27.7
D $500.0 125.2
The incremental B/C ratio exceeds 1.0 hence the increment is desirable. Alternative A is preferred over B.
Course Summary
3.
29
Take the preferred alternative from the previous step and consider the next higher-cost alternative. Do another two-alternative comparison. $ Initial Cost $ Uniform Annual Benefit Incremental B/C ratio # D-A $100.0 24.3
24.3( P/ A, 6%, 5) . # 102 100 The incremental B/C ratio exceeds 1.0 hence Alternative D is preferred over A.
BREAKEVEN ANALYSIS
In business, breakeven is defined as the point where income just covers the associated costs. In engineering economics, the breakeven point is more precisely defined as the point where two alternatives are equivalent. EXAMPLE 24 A city is considering a new $50,000 snowplow. The new machine will operate at a savings of $600 per day, compared to the equipment presently being used. Assume the minimum attractive rate of return(interest rate) is 12% and the machines life is 10 years with zero resale value at that time. How many days per year must the machine be used to make the investment economical? Solution This breakeven problem may be readily solved by annual cost computations. We will set the equivalent uniform annual cost of the snowplow equal to its annual benefit, and solve for the required annual utilization. Let X = breakeven point = days of operation per year. EAC = EAB 50,000(A/P, 12%, 10) = 600 X 50,000(01770 . ) X# 600 = 14.7 days/year
DEPRECIATION
Depreciation of capital equipment is an important component of many after-tax economic analyses. For this reason, one must understand the fundamentals of depreciation accounting.
30
Course Summary
Depreciation is defined in its accounting sense, as the systematic allocation of the cost of a capital asset over its useful life. Book value is defined as the original cost of an asset, minus the accumulated depreciation of the asset. In computing a schedule of depreciation charges, three items are considered: 1. 2. 3. Cost of the property, P; Depreciable life in years, n; Salvage value of the property at the end of its depreciable life, S.
Sum-Of-Years-Digits Depreciation
Depreciation charge in any year Remaining Depreciable Life at Beginning of Year Sum of Years Digits for Total Useful Life
n ! n $ 1" 2
! P % S"
Course Summary
Depreciation charge in any year n = P(percentage depreciation allowance) Recovery Periods for MACRS
Recovery Period Description of Assets Included 3-Year Tractors for over-the-road tractor/trailer use and special tools such as dies and jigs; ADR< 4 years 5-Year Cars, buses, trucks, computers, office machinery, construction equipment, and R&D equipment; 4 years - ADR < 10 years 7-Year Office furniture, most manufacturing equipment, mining equipment, And items not otherwise classified, 10 years - ADR < 16 years 10-Year Marine vessels, petroleum refining equipment, single-purpose agricultural structures, trees and vines that bear nuts or fruits; 16 years - ADR < 20 years 15-Year Roads, shrubbery, wharves, steam and electric generation and Distribution systems, and municipal wastewater treatment facilities; 20 years - ADR < 25 years 20-Year Farm buildings and municipal sewers; ADR . 25 years 27.5-Year Residential rental property 31.5-Year Non-residential real property purchased on or before 5/12/93 39-Year Non-residential real property purchased on or after 5/13/93 Note: This table is used to find the recovery period for all items under MACRS.
31
MACRS Percentages
Year 1 2 3 4 5 6 7 8 9 10 11 12 13 14 15 16 17 18 19 20 21 3-year 33.33 44.45 14.81 8.41 5-year 20.00 32.00 19.20 11.52 11.52 5.76 Recovery Period 7-year 10-year 15-year 20-year 14.29 10.00 5.00 3.750 24.49 18.00 9.50 7.219 17.49 14.40 8.55 6.677 12.49 11.52 7.70 6.177 8.93 9.22 6.93 5.713 8.92 8.93 4.46 7.37 6.55 6.55 6.56 6.55 3.28 6.23 5.90 5.90 5.91 5.90 5.91 5.90 5.91 5.90 5.91 2.95 5.285 4.888 4.522 4.462 4.461 4.462 4.461 4.462 4.461 4.462 4.461 4.462 4.461 4.462 4.461 2.231
32
Course Summary
EXAMPLE 25 A piece of construction machinery costs $5,000 and has an anticipated $1,000 salvage value at the end of its five year depreciable life. Compute the depreciation schedule for the machinery by: (a) Straight line depreciation; (b) Sum-of-years-digits depreciation; (c) Double declining balance depreciation. (d) MACRS
Solution
Straight line depreciation =
P%S 5000 % 1000 = = $800 n 5
Sum - of - years - digits depreciation: n 5 Sum-of-years-digits = ! n $ 1" # ! 6" # 15 2 2 5 1st year depreciation = (5,000 - 1,000) = $1,333 15 4 2nd year depreciation = (5,000 - 1,000) = 1,067 15 3 3rd year depreciation = (5,000 - 1,000) = 800 15 2 4th year depreciation = (5,000 - 1,000) = 533 15 1 5th year depreciation = (5,000 - 1,000) = 267 15 $4,000 Double declining balance depreciation: 2 1st year depreciation = (5,000 - 0) 5 2 2nd year depreciation = (5,000 - 2,000) 5 2 3rd year depreciation = (5,000 - 3,200) 5 2 4th year depreciation = (5,000 - 3,920) 5 2 5th year depreciation = (5,000 - 4,352) 5
Since the problem specifies a $1,000 salvage value, the total depreciation may not exceed $4,000. The double declining balance depreciation must be stopped in the 4th year when it totals $4,000.
Course Summary
MACRS
33
Since the asset is specified as construction equipment the 5-year recovery period is determined. Note that the depreciation will be over a six year period. This is because only year depreciation is allowed in the first year. The remaining year is recovered in the last year. If an asset is disposed of mid-life, than only of the disposal years depreciation is taken. Also note that the salvage value is not used in the depreciation calculations. 1st year depreciation 2nd year depreciation 3rd year depreciation 4th year depreciation 5th year depreciation 6th year depreciation = .2000(5000) = $1,000 = .3200(5000) = 1,600 = .1920(5000) = 960 = .1152(5000) = 576 = .1152(5000) = 576 = .0576(5000) = 288 $5,000
INCOME TAXES
Income taxes represent another of the various kinds of disbursements encountered in an economic analysis. The starting point in an after-tax computation is the before-tax cash flow. Generally, the before-tax cash flow contains three types of entries: 1. Disbursements of money to purchase capital assets. These expenditures create no direct tax consequence for they are the exchange of one asset(cash) for another(capital equipment). Periodic receipts and/or disbursements representing operating income and/or expenses. These increase or decrease the year-by-year tax liability of the firm. Receipts of money from the sale of capital assets, usually in the form of a salvage value when the equipment is removed. The tax consequence depends on the relationship between the residual value of the asset and its book value(cost - depreciation taken).
2. 3.
34
Course Summary
Residual Value is less than current book value equals current book value exceeds current book value exceeds initial book value Tax Consequence loss on sale no loss & no recapture recaptured depreciation recaptured depreciation & capital gain
After the before-tax cash flow, the next step is to compute the depreciation schedule for any capital assets. Next, taxable income is the taxable component of the before-tax cash flow minus the depreciation. Then, the income tax is the taxable income times the appropriate tax rate. Finally, the after-tax cash flow is the before-tax cash flow adjusted for income taxes. To organize these data, it is customary to arrange them in the form of a cash flow table, as follows: Year 0 1 Before-tax cash flow Depreciation Taxable income Income taxes After-tax cash flow
EXAMPLE 26 A corporation expects to receive $32,000 each year for 15 years from the sale of a product. There will be an initial investment of $150,000. Manufacturing and sales expenses will be $8067 per year. Assume straight-line depreciation, a 15-year useful life and no salvage value. Use a 46% income tax rate. Determine the projected after-tax rate of return. Solution Straight line depreciation = Before-tax cash flow -150,000 +23,933 +23,933 . . . +23,933
P%S 150,000 % 0 = = $10,000 per year n 15
Year 0 1 2 . . . 15
Take the after-tax cash flow and compute the rate of return at which PW of cost equals PW of benefits. 150,000 = 17,524(P/A, i, 15) (P/A, i, 15) = 150,000/17,524 = 8.559
Course Summary
From compound interest tables, i = 8%.
35
INFLATION
Inflation is a decrease in the buying power of the dollar (or peso, yen, etc.). More generally it is thought of as the increase in the level of prices. Inflation rates are usually estimated as having a constant rate of change. This type of cash flow is a geometric gradient. Future cash flows are generally estimated in constant-value terms. The assumption is made that the inflation rates for various cash flows will match the economys inflation rate. This allows for the use of a real interest rate. Another approach is the market interest rate. The market rate includes both the time value of money and an estimate of current and predicted inflation. For exact calculation of real and market interest rates the following formula is used. (1 + Market rate) = (1 + i)(1 + f) where i = the real interest rate f = the inflation rate EXAMPLE 27 Determine the market interest rate if the inflation rate is estimated to be 2.5% and the time value of money (the real interest rate) is 6%. Solution (1 + Market rate) = (1 + .06)(1 + .025) (1 + Market rate) = 1.0865 Market rate = 0.0865 = 8.65%
Probabilities must satisfy the following: 1. 2. 0 < P < 1. The sum of all probabilities must = 1.
36
Course Summary
Expected values for cash flows, useful lifes, salvage values, etc. can be found by using a weighted average calculation. EXAMPLE 28 The following table summarizes estimated annual benefits, annual costs, and end-of-life value for an asset under consideration. Determine the expected value for each of the three. p = .20 3,000 1,500 5,000 State and Associated Probability p = .40 p = .25 6,500 8,000 2,800 5,000 7,500 9,000 p = .15 11,500 5,750 10,000
Annual Benefits Annual Costs End-of-Life-Value Solution E(Annual Benefits) E(Annual Costs)
= .20(3,000) + .40(6,500) + .25(8,000) + .15(11,500) = $6,925.00 = .20(1,500) + .40(2,800) + .25(5,000) + .15(5,750) = $3,532.50
F " P(1 ! i ) n
P " F (1 ! i ) # n
) (1 ! i ) n # 1& F " A' $ i ( % ) & i A " F' $ n ( (1 ! i ) # 1% ) i (1 ! i ) n & A " P' $ n ( (1 ! i ) # 1% ) (1 ! i ) n # 1& P " A' n $ ( i (1 ! i ) % ) (1 ! i ) n # in # 1& A " G' $ n ( i (1 ! i ) # i % )1 & n or A " G ' # $ n ( i (1 ! i ) # 1% ) (1 ! i ) n # in # 1& P " G' 2 $ n ( i (1 ! i ) %
vi
P " A1 n(1 ! i ) #1
when i = g
F " P e rn
* +
P " F e # rn
* +
Uniform Series:
Compound Interest i = Interest rate per interest period*. n = Number of interest periods. P = A present sum of money. F = A future sum of money. A = An end-of-period cash receipt or disbursement in a uniform series continuing for n periods. G = Uniform period-by-period increase or decrease in cash receipts or disbursements. g = Uniform rate of cash flow increase or decrease from period to period; the geometric gradient. r = Nominal interest rate per interest period*. m = Number of compounding subperiods per periods*. _________________ *Normally the interest period is one year, but it could be something else.
vii
1 0
r. , #1 m-
where
OR ieff or ia = 21 ! i 3 # 1
m
where i = effective interest rate per period m = number of compounding periods in a year For continuous compounding: where ieff or ia = e
2 3# 1
r
Values of Interest Factors When n Equals Infinity Single Payment: (F/P, i, 4) = 4 (P/F, i, 4) = 0 Uniform Payment Series: (A/F, i, 4) = 0 (A/P, i, 4) = i (F/A, i, 4) = 4 (P/A, i, 4) = 1/i Arithmetic Gradient Series: (A/G, i, 4) = 1/i (P/G, i, 4) = 1/i2
viii
COURSE SUMMARY
This chapter is a brief review of engineering economic analysis/engineering economy. The goal is to give you a better grasp of the major topics in a typical first course. Hopefully this overview will help you put the course lectures and your reading of the textbook in better perspective. There are 26 example problems scattered throughout the engineering economics review. These examples are an integral part of the review and should be worked to completion as you come to them.
CASH FLOW
The field of engineering economics uses mathematical and economic techniques to systematically analyze situations that pose alternative courses of action. The initial step in engineering economics problems is to resolve a situation, or each alternative course in a given situation, into its favorable and unfavorable consequences or factors. These are then measured in some common unit -- usually money. Those factors that cannot readily be reduced to money are called intangible, or irreducible, factors. Intangible or irreducible factors are not included in any monetary analysis but are considered in conjunction with such an analysis when making the final decision on proposed courses of action. A cash flow table shows the money consequences of a situation and its timing. For example, a simple problem might be to list the year-by-year consequences of purchasing and owning a used car:
Car purchased now for $4,500 cash. The minus sign indicates a disbursement.
End of Year 1
End of Year 2
End of Year 3 End of Year 4
The car is sold at the end of the 4th year for $2,000. The plus sign represents a receipt of money.
Course Summary
This same cash flow may be represented graphically: Receipt of money Disbursement of money $2,000
$350
$350
$350 $350
$4,500
The upward arrow represents a receipt of money, and the downward arrows represent disbursements. The x-axis represents the passage of time. EXAMPLE 1 In January 1999, a firm purchases a used typewriter for $500. Repairs cost nothing in 1999 or 2000. Repairs are $85 in 2001, $130 in 2002, and $140 in 2003. The machine is sold in 2003 for $300. Compute the cash flow table. Solution Unless otherwise stated in problems, the customary assumption is a beginning-of-year purchase, followed by end-of-year receipts or disbursements, and an end-of-year resale or salvage value. Thus the typewriter repairs and the typewriter sale are assumed to occur at the end of the year. Letting a minus sign represent a disbursement of money, and a plus sign a receipt of money, we are able to set up this cash flow table: Year Beginning of 1999 End of 1999 End of 2000 End of 2001 End of 2002 End of 2003 Cash Flow -$500 0 0 -85 -130 +160
Notice that at the end of 1994, the cash flow table shows +160. This is the net of -140 and +300.
Course Summary
If we define Year 0 as the beginning of 1990, the cash flow table becomes: Year 0 1 2 3 4 5 Cash Flow -$500 0 0 -85 -130 +160
From this cash flow table, the definitions of Year 0 and Year 1 become clear. Year 0 is defined as the beginning of Year 1. Year 1 is the end of Year 1. Year 2 is the end of Year 2, and so forth.
EQUIVALENCE
In the preceding section we saw that money at different points in time(for example, $100 today or $100 one year hence) may be equal in the sense that they both are $100, but $100 a year hence is not an acceptable substitute for $100 today. When we have acceptable substitutes, we say they are equivalent to each other. Thus at 8% interest, $108 a year hence is equivalent to $100 today. EXAMPLE 2 At a 10% per year interest rate, $500 now is equivalent to how much three years hence? Solution $500 now will increase by 10% in each of the three years. Now = $500.00 End of 1st year = 500 + 10%(500) = 550.00 End of 2nd year = 550 + 10%(550) = 605.00 End of 3rd year = 605 + 10%(605) = 665.50
Course Summary
Thus $500 now is equivalent to $665.50 at the end of three years. Equivalence is an essential factor in engineering economic analysis. Suppose we wish to select the better of two alternatives. First, we must compute their cash flows. An example would be: Alternative A B -$2,000 -$2,800 +800 +1,100 +800 +1,100 +800 +1,100
Year 0 1 2 3
The larger investment in Alternative B results in larger subsequent benefits, but we have no direct way of knowing if Alternative B is better than Alternative A. Therefore we do not know which alternative should be selected. To make a decision we must resolve the alternatives into equivalent sums so they may be compared accurately and a decision made.
Symbols
i = = = = Interest rate per interest period. In equations the interest rate is stated as a decimal(that is, 8% interest is 0.08). Number of interest periods. A present sum of money. A future sum of money. The future sum F is an amount, n interest periods from the present, that is equivalent to P with interest rate i. An end-of-period cash receipt or disbursement in a uniform series continuing for n periods, the entire series equivalent to P or F at interest rate i. Uniform period-by-period increase in cash flows; the arithmetic gradient. Uniform rate of period-by-period increase in cash flows; the geometric gradient.
n P F
G g
= =
Course Summary
Functional Notation
To Find Single Payment
Compound Amount Factor Present Worth Factor Uniform Payment Series Sinking Fund Factor Capital Recovery Factor Compound Amount Factor Present Worth Factor Arithmetic Gradient Gradient Uniform Series Gradient Present Worth A P G G (A/G, i, n) (P/G, i, n) A A F P F P A A (A/F, i, n) (A/P, i, n) (F/A ,i ,n) (P/A , i, n) F P P F (F/P, i, n) (P/F, i, n)
Given
Functional Notation
From the table above we can see that the functional notation scheme is based on writing (To Find / Given, i, n). Thus, if we wished to find the future sum F, given a uniform series of receipts A, the proper compound interest factor to use would be (F/A, i, n).
Course Summary
The present sum P increases in n periods to P(1 + i) n . This gives us a relationship between a present sum P and its equivalent future sum F: Future Sum =(Present Sum)(1 + i) n F = P(1 + i)n This is the Single Payment Compound Amount formula. In functional notation it is written: F = P(F/P, i, n) The relationship may be rewritten as: Present Sum =(Future Sum)(l + i)-n P = F(1 + i)-n This is the Single Payment Present Worth formula. It is written: P = F(P/F, i, n) EXAMPLE 3 At a 10% per year interest rate, $500 now is equivalent to how much three years hence?
Course Summary
Solution This problem was solved in Example 2. Now it can be solved using a single payment formula. P = $500 n = 3 years F = unknown i = 10%
F = P(1 + i)n = 500(1 + 0.10)3 = $665.50 This problem may also be solved using the Compound Interest Tables. F = P(F/P, i, n) = 500(F/P, 10%, 3) From the 10% Compound Interest Table, read(F/P, 10%, 3) = 1.331. F = 500(F/P, 10%, 3) = 500(1.331) = $665.50 EXAMPLE 4 To raise money for a new business, a man asks you to loan him some money. He offers to pay you $3,000 at the end of four years. How much should you give him now if you want 12% interest per year on your money? Solution F = $3,000 n = 4 years P = unknown i = 12 %
P = F(l + i)-n = 3,000(1 + 0.12)-4 = $1,906.55 Alternate computation using Compound Interest Tables: P = F(P/F, i, n) = 3,000(P/F, 12%, 4) = 3,000(0.6355) = $1,906.50 Note that the solution based on the Compound Interest Table is slightly different from the exact solution using a hand calculator. In economic analysis, the Compound Interest Tables are always considered to be sufficiently accurate.
Course Summary
Solving this equation for A: A = P [i(1 + i)n] / [(1 + i)n - 1] Uniform Series Capital Recovery formula
Since F = P(1 + i)n, we can multiply both sides of the P/A equation by(1 + i)n to obtain: (1 + i)nP = A [(1 + i)n - 1] / i which yields F = A [(1 + i)n - 1] / i Uniform Series Compound Amount formula Solving this equation for A: A = F [i /(1 + i)n - 1] Uniform Series Sinking Fund formula
In functional notation, the uniform series factors are: Compound Amount(F/A, i, n) Sinking Fund(A/F, i, n) Capital Recovery(A/P, i, n) Present Worth(P/A, i, n)
EXAMPLE 5 If $100 is deposited at the end of each year in a savings account that pays 6% interest per year, how much will be in the account at the end of five years?
Course Summary
Solution A = $100 n = 5 years F = unknown i = 6%
F = A(F/A, i, n) = 100(F/A, 6%, 5) = 100(5.637) = $563.70 EXAMPLE 6 A woman wishes to make a uniform deposit every three months to her savings account so that at the end of 10 years she will have $10,000 in the account. If the account earns 6% annual interest, compounded quarterly, how much should she deposit each three months? Solution F = $10,000 n = 40 quarterly deposits A = unknown i = 1 % per quarter year
Note that i, the interest rate per interest period, is 1 %, and there are 40 deposits. A = F(A/F, i, n) = 10,000(A/F, 1 %, 40) = 10,000(0.0184) = $184 EXAMPLE 7 An individual is considering the purchase of a used automobile. The total price is $6200 with $1240 as a down payment and the balance paid in 48 equal monthly payments with interest at 1% per month. The payments are due at the end of each month. Compute the monthly payment. Solution The amount to be repaid by the 48 monthly payments is the cost of the automobile minus the $1,240 down payment. P = $4960 n = 48 monthly payments A = unknown i = 1% per month
A = P(A/P, i, n) = 4,960(A /P, 1 %, 48) = 4,960(0.0263) = $130.45 EXAMPLE 8 A couple sold their home. In addition to cash, they took a mortgage on the house. The mortgage will be paid off by monthly payments of $232.50 for 10 years. The couple decides to sell the mortgage to a local bank. The bank will buy the mortgage, but requires a 1% per month interest rate on their investment. How much will the bank pay for the mortgage? Solution A = $232.50 n = 120 months P = unknown i = 1% per month
10
Course Summary
Arithmetic Gradient
At times one will encounter a situation where the cash flow series is not a constant amount A. Instead it is an increasing series like: 125 150 175 200
100
n=5 This cash flow may be resolved into two components: 150 175 200 G n=5 p* p' p 2G 3G (n 1)G
100
125
=
G = 25 n=5
We can compute the value of P* as equal to P' plus P. We already have an equation for P' = P' = A(P/A, i, n) The value for P in the right-hand diagram is:
(1 + i ) n in 1 P = G i 2 1+ i n ( )
This is the Arithmetic Gradient Present Worth formula. In functional notation, the relationship is: P = G(P/G, i, n) EXAMPLE 9 The maintenance on a machine is expected to be $155 at the end of the first year, and increasing $35 each year for the following seven years. What present sum of money would need to be set aside now to pay the maintenance for the eight-year period? Assume 6 % interest.
Course Summary
Solution G = 35 155
11
P = unknown
n = 8 years i = 6%
In the gradient series, if instead of the present sum P, an equivalent uniform series A is desired, the problem becomes:
175 200 G 0 = A* A = 100 + A' 2G 3G (n-1)G
100
125
150
(1 + i ) n in A = G i (1 + i ) n 1
In functional notation, the Arithmetic Gradient (to) Uniform Series factor is: A =G(A/G, i, n) It is important to note carefully the diagrams for the two arithmetic gradient series factors. In both cases the first term in the arithmetic gradient series is zero and the last term is(n - 1)G. But we use n in the equations and functional notation. The derivations(not shown here) were done on this basis and the arithmetic gradient series Compound Interest Tables are computed this way. EXAMPLE 10 For the situation in Example 9, we wish now to know the uniform annual maintenance cost. Compute an equivalent A for the maintenance costs.
12
Solution
Course Summary
G = 35 155
n = 8 years i = 6%
Geometric Gradient
The arithmetic gradient is applicable where the period-by-period change in the cash flow is a uniform amount. There are other situations where the period-by-period change is a uniform rate, g. A diagram of this situation is: An A4 A3 A2 A1
P where An = A1(1 +g)n-1 g = Uniform rate of period-by-period change; the geometric gradient stated as a decimal(8% = 0.08). A1 = Value of A at Year 1. An = Value of A at any Year n.
1 (1 + g ) n (1 + i ) n P = A1 ig
Course Summary
13
EXAMPLE 11 It is likely that airplane tickets will increase 8% in each of the next four years. The cost of a plane ticket at the end of the first year will be $180. How much money would need to be placed in a savings account now to have money to pay a students travel home at the end of each year for the next four years? Assume the savings account pays 5% annual interest. Solution The problem describes a geometric gradient where g = 8% and i = 5 %.
Thus, $715.67 would need to be deposited now. As a check, the problem can be solved without using the geometric gradient: Year 1 2 3 4 A1 = A2 = 180.00 + 8%(180.00) = A3 = 194.40 + 8%(194.40) = A4 = 209.95 + 8%(209.95) = Ticket $180.00 194.40 209.95 226.75
P = 180.00(P/F, 5%, 1) + 194.40(P/F, 5%, 2) + 209.95(P/F, 5%, 3)+ 226.75(P/F, 5%,4) = 180.00(0.9524) + 194.40(0.9070) + 209.95(0.8638) + 226.75(0.8227) = $715.66
14
Course Summary
At beginning of year =
= = = = Amount in Account $1,000.00 1,015.00 1,030.23 1,045.68 1,061.37
The actual interest rate on the $1,000 would be the interest, $61.37, divided by the original $1,000, or 6.137%. We call this the effective interest rate. Effective interest rate =(1 + i )m - 1 , where i = Interest rate per interest period; m = Number of compoundings per year. EXAMPLE 12
A bank charges 1% per month on the unpaid balance for purchases made on its credit card. What nominal interest rate is it charging? What effective interest rate?
Solution
The nominal interest rate is simply the annual interest ignoring compounding, or 12(1%) = 18%. Effective interest rate =(1+0.015)12 -1 = 0.1956 = 19.56%
Criteria
Economic analysis problems inevitably fall into one of these categories: 1. Fixed Input The amount of money or other input resources is fixed.
Course Summary
2. Fixed Output
15
Example: A mechanical contractor has been awarded a fixed price contract to air-condition a building. 3. Neither Input nor Output Fixed This is the general situation where neither the amount of money or other inputs, nor the amount of benefits or other outputs are fixed.
Example: A consulting engineering firm has more work available than it can handle. It is considering paying the staff for working evenings to increase the amount of design work it can perform. There are five major methods of comparing alternatives: present worth; future worth; annual cost; rate of return; and benefit-cost ratio. These are presented in the following sections.
PRESENT WORTH
In present worth analysis, the approach is to resolve all the money consequences of an alternative into an equivalent present sum. For the three categories given above, the criteria are: Category Fixed Input Fixed Output Neither Input nor Output Fixed Present Worth Criterion Maximize the Present Worth of benefits or other outputs. Minimize the Present Worth of costs or other inputs. Maximize [Present Worth of benefits minus Present Worth of costs ] or, stated another way: Maximize Net Present Worth.
16
Course Summary
In situations like this, the solution is either to use some other analysis technique(generally the annual cost method) or to restructure the problem so there is a common analysis period. In the example above, a customary assumption would be that a pump is needed for 12 years and that Pump A will be replaced by an identical Pump A at the end of 6 years. This gives a 12-year common analysis period. Pump A Pump B Correct Present Worth Comparison This approach is easy to use when the different lives of the alternatives have a practical least common multiple life. When this is not true(for example, life of J equals 7 years and the life of K equals 11 years), some assumptions must be made to select a suitable common analysis period, or the present worth method should not be used. EXAMPLE 13 Machine X has an initial cost of $10,000, annual maintenance of $500 per year, and no salvage value at the end of its four-year useful life. Machine Y costs $20,000. The fast year there is no maintenance cost. The second year, maintenance is $100, and increases $100 per year in subsequent years. The machine has an anticipated $5000 salvage value at the end of its 12-year useful life. If interest is 8%, which machine should be selected? Solution The analysis period is not stated in the, problem. Therefore we select the least common multiple of the lives, or 12 years, as the analysis period. Present Worth of Cost of 12 years of Machine X = 10,000 + 10,000(P/F, 8%, 4) + 10,000(P/F, 8%, 8) + 500(P/A, 8%, 12) = 10,000 + 10,000(0.7350) + 10,000(0.5403) + 500(7.536) = $26,521 Present Worth of Cost of 12 years of Machine Y = 20,000 + 100(P/G, 8%, 12) - 5,000(P/F, 8%, 12) = 20,000 + 100(34.634) - 5,000(0.3971) = $21,478 Choose Machine Y with its smaller PW of Cost. Replacement Pump A
12 years 12 years
Course Summary
EXAMPLE 14 Two alternatives have the following cash flows: Alternative Year A B 0 -$2,000 -$2,800 1 +800 +1,100 2 +800 +1,100 3 +800 +1,100 At a 5% interest rate, which alternative should be selected? Solution Solving by Present Worth analysis: Net Present Worth(NPW) NPWA NPWB To maximize NPW, choose Alternative B.
17
= PW of benefits - PW of cost = 800(P/A, 5%, 3) - 2,000 = 800(2.723) - 2,000 = $178.40 = 1,100(P/A, 5%, 3) - 2,800 = 1,100(2.723) - 2,800 = $195.30
Capitalized Cost
In the special situation where the analysis period is infinite(n = ), an analysis of the present worth of cost is called capitalized cost. There are a few public projects where the analysis period is infinity. Other examples would be permanent endowments and cemetery perpetual care. When n equals infinity, a present sum P will accrue interest of Pi for every future interest period. For the principal sum P to continue undiminished (an essential requirement for n equal to infinity), the end-of-period sum A that can be disbursed is Pi. P P + Pi P + Pi P + Pi
.
A
When n = , the fundamental relationship between P, A, and i is: A = Pi Some form of this equation is used whenever there is a problem with an infinite analysis period.
18
Course Summary
EXAMPLE 15 In his will, a man wishes to establish a perpetual trust to provide for the maintenance of a small local park. If the annual maintenance is $7,500 per year and the trust account can earn 5% interest, how much money must be set aside in the trust? Solution When n = , A = Pi or P = A/i Capitalized cost P = A/i = $7,500/0.05 = $150,000
FUTURE WORTH
In present worth analysis, the comparison is made in terms of the equivalent present costs and benefits. But the analysis need not be made at the present time. It could be made at any point in time: past, present, or future. Although the numerical calculations may look different, the decision is unaffected by the point in time selected. Of course, there are situations where we do want to know what the future situation will be if we take some particular course of action now. When an analysis is made based on some future point in time, it is called future worth analysis. Category Fixed Input Fixed Output Neither Input nor Output Fixed Future Worth Criterion Maximize the Future Worth of benefits or other outputs. Minimize the Future Worth of costs or other inputs. Maximize [Future Worth of benefits minus Future Worth of costs ] or, stated another way: Maximize Net Future Worth.
EXAMPLE 16 Two alternatives have the following cash flows: Alternative Year A B 0 -$2,000 -$2,800 1 +800 +1,100 2 +800 +1,100 3 +800 +1,100 At a 5% interest rate, which alternative should be selected? Solution In Example 14, this problem was solved by Present Worth analysis at Year 0. Here it will be solved by Future Worth analysis at the end of Year 3. Net Future Worth(NFW) NFWA = FW of benefits - FW of cost = 800(F/A, 5%, 3) - 2,000(F/P, 5%, 3) = 800(3.152) - 2000(1.158) = $205.60
Course Summary
NPWB To maximize NFW, choose Alternative B. = 1,100(P/A, 5%, 3) - 2,800(F/P, 5%, 3) = 1,100(3.152) - 2,800(1.158) = $224.80
19
ANNUAL COST
The annual cost method is more accurately described as the method of Equivalent Annual Cost(EAC) or, where the computation is of benefits, the method of Equivalent Annual Benefits(EAB). For each of the three possible categories of problems, there is an annual cost criterion for economic efficiency. Category Fixed Input Fixed Output Neither Input nor Output Fixed Annual Cost Criterion Maximize the Equivalent Annual Benefits. That is, maximize EAB. Minimize the Equivalent Annual Cost. That is, minimize EAC. Maximize [ EAB - EAC ]
20
Course Summary
Problems presented on examinations are often readily solved by the annual cost method. And the underlying continuing requirement is often present, so that an annual cost comparison of unequal-lived alternatives is an appropriate method of analysis. EXAMPLE 17 Consider the following alternatives: First cost Annual maintenance End-of-useful-life salvage value Useful life A $5,000 500 600 5 years B $10,000 200 1,000 15 years
Based on an 8% interest rate, which alternative should be selected? Solution Assuming both alternatives perform the same task and there is a continuing requirement, the goal is to minimize EAC. Alternative A; EAC = 5,000(A/P, 8%, 5) + 500 - 600(A/F, 8%, 5) = 5,000(0.2505) + 500 - 600(0.1705) = $1,650 Alternative B: EAC = 10,000(A/P, 8%, 15)+ 200 - 1,000(A/F, 8%, 15) = 10,000(0.1168) + 200 - 1,000(0.0368) = $1,331 To minimize EAC, select Alternative B.
RATE OF RETURN
A typical situation is a cash flow representing the costs and benefits. The rate of return may be defined as the interest rate where PW of cost = PW of benefits, EAC = EAB, or PW of cost - PW of benefits = 0.
Course Summary
EXAMPLE 18 Compute the rate of return for the investment represented by the following cash flow: Year Cash Flow 0 -$595 1 +250 2 +200 3 +150 4 +100 5 + 50
21
Solution
This declining arithmetic gradient series may be separated into two cash flows for which compound interest factors are available: 250 200 250 250 250 250 250 150 100 50 0 100 150 200
50 =
Note that the gradient series factors are based on an increasing gradient. Here, subtracting an increasing arithmetic gradient, as indicated by the diagram, solves the declining cash flow. PW of cost - PW of benefits = 0 595 - [250(P/A ,i, 5) - 50(P/G, i, 5)] = 0 Try i = 10 %: 595 - [250(3.791) - 50(6.862)] = -9.65 Try i = 12 %: 595 - [250(3.605) - 50(6.397)] = +13.60 The rate of return is between 10% and 12%. It may be computed more accurately by linear interpolation:
22
Course Summary
Course Summary
23
Since the incremental rate of return exceeds the 5 % MARR, the increment of investment is desirable. Choose the higher-cost Alternative B. Before leaving this example problem, one should note something that relates to the rates of return on Alternative A and on Alternative B. These rates of return, if computed, are: Alternative A Alternative B Rate of Return 9.7% 8.7%
The correct answer to this problem has been shown to be Alternative B, and this is true even though Alternative A has a higher rate of return. The higher-cost alternative may be thought of as the lower cost alternative, plus the increment of investment between them. Looked at this way, the higher-cost Alternative B is equal to the desirable lower-cost Alternative A plus the desirable differences between the alternatives. The important conclusion is that computing the rate of return for each alternative does not provide the basis for choosing between alternatives. Instead, incremental analysis is required. EXAMPLE 20 Consider the following: Alternative Year A B 0 -$200.0 -$131.0 1 +77.6 +48.1 2 +77.6 +48.1 3 +77.6 +48.1 If the minimum attractive rate of return(MARR) is 10%, which alternative should be selected? Solution To examine the increment of investment between the alternatives, we will examine the higher initial cost alternative minus the lower initial-cost alternative, or A - B. Alternative Year 0 1 2 3 A -$200.0 +77.6 +77.6 +77.6 B -$131.0 +48.1 +48.1 +48.1 Increment A-B -$69.0 +29.5 +29.5 +29.5
24
Course Summary
Solve for the incremental rate of return: PW of cost = PW of benefits 69.0 = 29.5(P/A, i, 3) (P/A, i, 3) = 69.0/29.5 =2.339 From Compound Interest Tables, the incremental rate of return is between 12% and 15%. This is a desirable increment of investment hence we select the higher initial-cost Alternative A.
2. 3.
4. 5.
EXAMPLE 21 Consider the following: Alternative Year A B 0 -$200.0 -$131.0 1 +77.6 +48.1 2 +77.6 +48.1 3 +77.6 +48.1 If the minimum attractive rate of return(MARR) is 10%, which alternative, if any, should be selected?
Course Summary
Solution
25
One should carefully note that this is a three-alternative problem where the alternatives are A, B, and Do Nothing. In this solution we will skip Step 1. Reorganize the problem by placing the alternatives in order of increasing initial cost: Do Alternative Year Nothing B A 0 0 -$131.0 -$200.0 1 0 +48.1 +77.6 2 0 +48.1 +77.6 3 0 +48.1 +77.6 Examine the B - Do Nothing increment of investment: Do Year B - Nothing 0 -$131.0 0 = -$131.0 1 +48.1 0 = +48.1 2 +48.1 0 = +48.1 3 +48.1 0 = +48.1 Solve for the incremental rate of return: PW of cost = PW of benefits 131.0 = 48.1(P/A, i, 3) (P/A, i, 3) = 131.0/48.1 = 2.723 . From Compound Interest Tables, the incremental rate of return = 5%. Since the incremental rate of return is less than 10%, the B - Do Nothing increment is not desirable. Reject Alternative B. Next, consider the increment of investment between the two remaining alternatives: Do Year A - Nothing 0 -$200.0 0 = -$131.0 1 +77.6 0 = +77.6 2 +77.6 0 = +77.6 3 +77.6 0 = +77.6 Solve for the incremental rate of return: PW of cost = PW of benefits 200.0 = 77.6(P/A, i, 3) (P/A, i, 3) = 200/77.6 = 2.577
26
Course Summary
From Compound Interest Tables, the incremental rate of return is 8%. Since the rate of return on the A - Do Nothing increment of investment is less than the desired 10%, reject the increment by rejecting Alternative A. We select the remaining alternative: Do nothing! If you have not already done so, you should go back to Example 20 and see how the slightly changed wording of the problem radically altered it. Example 20 required the choice between two undesirable alternatives. Example 21 adds the Do-nothing alternative that is superior to A or B. EXAMPLE 22 Consider four mutually exclusive alternatives: Initial Cost Uniform Annual Benefit A $400.0 100.9 Alternative B C $100.0 $200.0 27.7 46.2 D $500.0 125.2
Each alternative has a five-year useful life and no salvage value. If the minimum attractive rate of return(MARR) is 6%, which alternative should be selected? Solution Mutually exclusive is where selecting one alternative precludes selecting any of the other alternatives. This is the typical textbook situation. The solution will follow the several steps in incremental analysis. 1. The rate of return is computed for the four alternatives. Alternative A B Computed rate of return 8.3% 11.9%
C 5%
D 8%
Since Alternative C has a rate of return less than the MARR, it may be eliminated from further consideration. 2. Rank the remaining alternatives in order of increasing initial cost and examine the increment between the two lowest cost alternatives. Alternative A $400.0 100.9
Initial Cost Uniform Annual Benefit Initial Cost Uniform Annual Benefit Computed rate of return
B $100.0 27.7
D $500.0 125.2
Since the incremental rate of return exceeds the 6% MARR, the increment of investment is desirable. Alternative A is the better alternative.
Course Summary
3.
27
Take the preferred alternative from the previous step and consider the next higher-cost alternative. Do another two-alternative comparison. Initial Cost Uniform Annual Benefit Computed rate of return D-A $100.0 24.3 6.9%
The incremental rate of return exceeds MARR, hence the increment is desirable. Alternative D is preferred over Alternative A. Conclusion: Select Alternative D. Note that once again the alternative with the highest rate of return(Alt. B) is not the proper choice.
BENEFIT-COST RATIO
Generally in public works and governmental economic analyses, the dominant method of analysis is called benefit-cost ratio. It is simply the ratio of benefits divided by costs, taking into account the time value of money.
B/ C =
For a given interest rate, a B/C ratio > 1 reflects an acceptable project. The method of analysis using B/C ratio is parallel to that of rate of return analysis. The same kind of incremental analysis is required.
28
Course Summary
EXAMPLE 23 Solve Example 22 using Benefit-Cost analysis. Consider four mutually exclusive alternatives: Alternative A B C D Initial Cost $400.0 $100.0 $200.0 $500.0 Uniform Annual Benefit 100.9 27.7 46.2 125.2 Each alternative has a five-year useful life and no salvage value. Based on a 6% interest rate, which alternative should be selected? Solution 1. B/C ratio computed for the alternatives: Alt. A
B/ C =
PW of benefits 100.9( P/ A, 6%, 5) . = = 106 400 PW of costs PW of benefits 27.7( P/ A, 6%, 5) . = = 117 100 PW of costs PW of benefits 46.2( P/ A, 6%, 5) = = 0.97 200 PW of costs
Alt. B
B/ C =
Alt. C
B/ C =
PW of benefits 125.2( P/ A, 6%, 5) . = = 105 500 PW of costs Alternative C with a B/C ratio less than 1 is eliminated.
Alt. D
B/ C =
2.
Rank the remaining alternatives in order of increasing initial cost and examine the increment of investment between the two lowest cost alternatives. Alternative A $400.0 100.9
Initial Cost Uniform Annual Benefit Initial Cost Uniform Annual Benefit Incremental B/C ratio =
B $100.0 27.7
D $500.0 125.2
The incremental B/C ratio exceeds 1.0 hence the increment is desirable. Alternative A is preferred over B.
Course Summary
3.
29
Take the preferred alternative from the previous step and consider the next higher-cost alternative. Do another two-alternative comparison. Initial Cost Uniform Annual Benefit Incremental B/C ratio = D-A $100.0 24.3
24.3( P/ A, 6%, 5) . = 102 100 The incremental B/C ratio exceeds 1.0 hence Alternative D is preferred over A.
Conclusion: Select Alternative D.
BREAKEVEN ANALYSIS
In business, breakeven is defined as the point where income just covers the associated costs. In engineering economics, the breakeven point is more precisely defined as the point where two alternatives are equivalent. EXAMPLE 24 A city is considering a new $50,000 snowplow. The new machine will operate at a savings of $600 per day, compared to the equipment presently being used. Assume the minimum attractive rate of return(interest rate) is 12% and the machines life is 10 years with zero resale value at that time. How many days per year must the machine be used to make the investment economical? Solution This breakeven problem may be readily solved by annual cost computations. We will set the equivalent uniform annual cost of the snowplow equal to its annual benefit, and solve for the required annual utilization. Let X = breakeven point = days of operation per year. EAC = EAB 50,000(A/P, 12%, 10) = 600 X 50,000(01770 . ) X= 600 = 14.7 days/year
DEPRECIATION
Depreciation of capital equipment is an important component of many after-tax economic analyses. For this reason, one must understand the fundamentals of depreciation accounting.
30
Course Summary
Depreciation is defined in its accounting sense, as the systematic allocation of the cost of a capital asset over its useful life. Book value is defined as the original cost of an asset, minus the accumulated depreciation of the asset. In computing a schedule of depreciation charges, three items are considered: 1. 2. 3. Cost of the property, P; Depreciable life in years, n; Salvage value of the property at the end of its depreciable life, S.
Sum-Of-Years-Digits Depreciation
Depreciation charge in any year Remaining Depreciable Life at Beginning of Year Sum of Years Digits for Total Useful Life
( P S)
n ( n + 1) 2
Course Summary
Depreciation charge in any year n = P(percentage depreciation allowance) Recovery Periods for MACRS
Recovery Period Description of Assets Included 3-Year Tractors for over-the-road tractor/trailer use and special tools such as dies and jigs; ADR< 4 years 5-Year Cars, buses, trucks, computers, office machinery, construction equipment, and R&D equipment; 4 years # ADR < 10 years 7-Year Office furniture, most manufacturing equipment, mining equipment, And items not otherwise classified, 10 years # ADR < 16 years 10-Year Marine vessels, petroleum refining equipment, single-purpose agricultural structures, trees and vines that bear nuts or fruits; 16 years # ADR < 20 years 15-Year Roads, shrubbery, wharves, steam and electric generation and Distribution systems, and municipal wastewater treatment facilities; 20 years # ADR < 25 years 20-Year Farm buildings and municipal sewers; ADR $ 25 years 27.5-Year Residential rental property 31.5-Year Non-residential real property purchased on or before 5/12/93 39-Year Non-residential real property purchased on or after 5/13/93 Note: This table is used to find the recovery period for all items under MACRS.
31
MACRS Percentages
Year 1 2 3 4 5 6 7 8 9 10 11 12 13 14 15 16 17 18 19 20 21 3-year 33.33 44.45 14.81 8.41 5-year 20.00 32.00 19.20 11.52 11.52 5.76 Recovery Period 7-year 10-year 15-year 20-year 14.29 10.00 5.00 3.750 24.49 18.00 9.50 7.219 17.49 14.40 8.55 6.677 12.49 11.52 7.70 6.177 8.93 9.22 6.93 5.713 8.92 8.93 4.46 7.37 6.55 6.55 6.56 6.55 3.28 6.23 5.90 5.90 5.91 5.90 5.91 5.90 5.91 5.90 5.91 2.95 5.285 4.888 4.522 4.462 4.461 4.462 4.461 4.462 4.461 4.462 4.461 4.462 4.461 4.462 4.461 2.231
32
Course Summary
EXAMPLE 25 A piece of construction machinery costs $5,000 and has an anticipated $1,000 salvage value at the end of its five year depreciable life. Compute the depreciation schedule for the machinery by: (a) Straight line depreciation; (b) Sum-of-years-digits depreciation; (c) Double declining balance depreciation. (d) MACRS
Solution
Straight line depreciation =
Sum - of - years - digits depreciation: n 5 Sum-of-years-digits = ( n + 1) = ( 6) = 15 2 2 5 1st year depreciation = (5,000 - 1,000) = $1,333 15 4 2nd year depreciation = (5,000 - 1,000) = 1,067 15 3 3rd year depreciation = (5,000 - 1,000) = 800 15 2 4th year depreciation = (5,000 - 1,000) = 533 15 1 5th year depreciation = (5,000 - 1,000) = 267 15 $4,000 Double declining balance depreciation: 2 1st year depreciation = (5,000 - 0) 5 2 2nd year depreciation = (5,000 - 2,000) 5 2 3rd year depreciation = (5,000 - 3,200) 5 2 4th year depreciation = (5,000 - 3,920) 5 2 5th year depreciation = (5,000 - 4,352) 5
Since the problem specifies a $1,000 salvage value, the total depreciation may not exceed $4,000. The double declining balance depreciation must be stopped in the 4th year when it totals $4,000.
Course Summary
MACRS
33
Since the asset is specified as construction equipment the 5-year recovery period is determined. Note that the depreciation will be over a six year period. This is because only year depreciation is allowed in the first year. The remaining year is recovered in the last year. If an asset is disposed of mid-life, than only of the disposal years depreciation is taken. Also note that the salvage value is not used in the depreciation calculations. 1st year depreciation 2nd year depreciation 3rd year depreciation 4th year depreciation 5th year depreciation 6th year depreciation = .2000(5000) = $1,000 = .3200(5000) = 1,600 = .1920(5000) = 960 = .1152(5000) = 576 = .1152(5000) = 576 = .0576(5000) = 288 $5,000
INCOME TAXES
Income taxes represent another of the various kinds of disbursements encountered in an economic analysis. The starting point in an after-tax computation is the before-tax cash flow. Generally, the before-tax cash flow contains three types of entries: 1. Disbursements of money to purchase capital assets. These expenditures create no direct tax consequence for they are the exchange of one asset(cash) for another(capital equipment). Periodic receipts and/or disbursements representing operating income and/or expenses. These increase or decrease the year-by-year tax liability of the firm. Receipts of money from the sale of capital assets, usually in the form of a salvage value when the equipment is removed. The tax consequence depends on the relationship between the residual value of the asset and its book value(cost - depreciation taken).
2. 3.
34
Course Summary
Residual Value is less than current book value equals current book value exceeds current book value exceeds initial book value Tax Consequence loss on sale no loss & no recapture recaptured depreciation recaptured depreciation & capital gain
After the before-tax cash flow, the next step is to compute the depreciation schedule for any capital assets. Next, taxable income is the taxable component of the before-tax cash flow minus the depreciation. Then, the income tax is the taxable income times the appropriate tax rate. Finally, the after-tax cash flow is the before-tax cash flow adjusted for income taxes. To organize these data, it is customary to arrange them in the form of a cash flow table, as follows: Year 0 1 Before-tax cash flow Depreciation Taxable income Income taxes After-tax cash flow
EXAMPLE 26 A corporation expects to receive $32,000 each year for 15 years from the sale of a product. There will be an initial investment of $150,000. Manufacturing and sales expenses will be $8067 per year. Assume straight-line depreciation, a 15-year useful life and no salvage value. Use a 46% income tax rate. Determine the projected after-tax rate of return. Solution Straight line depreciation = Before-tax cash flow -150,000 +23,933 +23,933 . . . +23,933
Year 0 1 2 . . . 15
Take the after-tax cash flow and compute the rate of return at which PW of cost equals PW of benefits. 150,000 = 17,524(P/A, i, 15) (P/A, i, 15) = 150,000/17,524 = 8.559
Course Summary
From compound interest tables, i = 8%.
35
INFLATION
Inflation is a decrease in the buying power of the dollar (or peso, yen, etc.). More generally it is thought of as the increase in the level of prices. Inflation rates are usually estimated as having a constant rate of change. This type of cash flow is a geometric gradient. Future cash flows are generally estimated in constant-value terms. The assumption is made that the inflation rates for various cash flows will match the economys inflation rate. This allows for the use of a real interest rate. Another approach is the market interest rate. The market rate includes both the time value of money and an estimate of current and predicted inflation. For exact calculation of real and market interest rates the following formula is used. (1 + Market rate) = (1 + i)(1 + f) where i = the real interest rate f = the inflation rate EXAMPLE 27 Determine the market interest rate if the inflation rate is estimated to be 2.5% and the time value of money (the real interest rate) is 6%. Solution (1 + Market rate) = (1 + .06)(1 + .025) (1 + Market rate) = 1.0865 Market rate = 0.0865 = 8.65%
Probabilities must satisfy the following: 1. 2. 0 < P < 1. The sum of all probabilities must = 1.
36
Course Summary
Expected values for cash flows, useful lifes, salvage values, etc. can be found by using a weighted average calculation. EXAMPLE 28 The following table summarizes estimated annual benefits, annual costs, and end-of-life value for an asset under consideration. Determine the expected value for each of the three. State and Associated Probability p = .40 p = .25 6,500 8,000 2,800 5,000 7,500 9,000
Annual Benefits Annual Costs End-of-Life-Value Solution E(Annual Benefits) E(Annual Costs)
= .20(3,000) + .40(6,500) + .25(8,000) + .15(11,500) = $6,925.00 = .20(1,500) + .40(2,800) + .25(5,000) + .15(5,750) = $3,532.50
37
38
1-2 A food processor is considering the development of a new line of product. Depending on the quality of raw material, he can expect different yields process-wise, and the quality of the final products will also change considerably. The product development department has identified three alternatives, and produced them in a pilot scale. The marketing department has used those samples for surveys to estimate potential sales and pricing strategies. The three alternatives would use existing equipment, but different process conditions and specifications, and they are summarized as follows. Indicate which alternative seems to be the best according to the estimated data, if the objective is to maximize total profit per year. Alternative 2 0.07 0.18 0.12 $0.24 1.05 1,250,000
Lbs of raw material A per unit of product Lbs of raw material B per unit of product Lbs of raw material C per unit of product Other processing costs ($/unit product) Expected wholesale price ($/unit product) Projected volume of sales (units of product) Cost of raw material A $3.45/lb Cost of raw material B $1.07/lb Cost of raw material C $1.88/lb Solution
Cost of Raw Material. A ($/unit product) Cost of Raw Material B ($/unit product) Cost of Raw Material C ($/unit product) Other processing costs ($/unit product) Total Cost ($/unit product) Wholesale price ($/unit product) Profit per unit Projected sales (units of product) Projected profits Therefore, choose alternative 2.
Alternative 2 0.2415 0.1926 0.2256 $0.24 0.8997 1.05 0.1503 1,250,000 187,875
39
1-3 Consider the previous problem. When asked about the precision of the given figures, the marketing department indicated the actual sales results could change plus/minus 10% from the forecast. Similarly, product development indicated the actual production costs may vary 3% from the pilot-based calculations. Is your choice of the best alternatives the same as you found in the previous problem? Solution Alternative 1: Cost max.=1.03 x .799 =.82297 min. =0.97 x .799 =.77503 Sales max.=1.1 x 1,000,000 =1,100,000 min. =0.9 x 1,000,000 = 900,000 Profit min. =.95-.82297=.12703 max.=.95-.77503=.17497 Sales x Profit = TOTAL PROFIT max.=1,100,000 x .17497=$192,467 min. = 900,000 x .12703=$114,327
Alternative 2: max. total profit = (1,250,000 x 1.1)(1.05 -.8997 x 0.97) = $243,775 min total profit = (1,250,000 x 0.9)(1.05 -.8997 x 1.03) = $138,723 Alternative 3: max.total profit = (800,000 x 1.1)(1.25 - 1.0865 x 0.97) = $172.564 min. total profit = (800,000 x 0.9)(1.25 - 1.0865 x 1.03) = $94,252 Although alternative 2 still gives the largest profits, the data's precision is not good enough to tell for sure that it will actually be the best choice since the max. profits for 1 & 3 are larger than the min. profits of 2.
1-4 Car A initially costs $500 more than Car B, but it consumes 0.04 gallons/mile versus 0.05 gallons/mile for B. Both last 8 years and B's salvage value is $100 smaller than A's. Fuel costs $1.70 per gallon. Other things being equal, beyond how many miles of use per year (X) does A become preferable to B? Solution -500 +100 + (.05 - .04) (1.70) (8) X = 0 -400 + 0.136X = 0 X = 400/0.136 = 2,941 miles/year
40
1-5 The following letter was a reply from Benjamin Franklin to Joseph Priestley, a friend of Franklin's. Priestley had been invited to become the librarian for the Earl of Shelburne and had asked for Franklin's advice. What engineering economy principle does Franklin suggest Priestley use to aid in making his decision? London, September 19, 1772 Dear Sir: In the affair of so much importance to you wherein you ask my advice, I cannot, for want of sufficient premises, advise you what to determine, but if you please I will tell you how. When these difficult cases occur, they are difficult chiefly because while we have them under consideration, all the reasons Pro and Con are not present to the mind at the same time; but sometimes one set present themselves, and at other times another, the first being out of sight. Hence the various purposes or inclination that alternately prevail, and the uncertainty that perplexes us. To get over this, my way is to divide a half a sheet of paper by a line into two columns; writing over the one PRO and over the other CON. Then during three or four days' consideration I put down under the different heads short hints of the different motives that at different times occur to me, for or against the measure. When I have thus got them all together in one view, I endeavour to estimate their respective weights; and where I find two (one on each side) that seem equal, I strike them both out. If I find a reason Pro equal to some two reasons Con, I strike out the three. If I judge some two reasons Con equal to three reasons Pro, I strike out the five; and thus proceeding I find at length where the balance lies; and if after a day or two of further consideration, nothing new that is of importance occurs on either side, I come to a determination accordingly. And though the weight of the reasons cannot be taken with the precision of algebraic quantities, yet when each is thus considered separately and comparatively and the whole lies before me, I think I can judge better, and am less likely to make a rash step; and in fact I have found great advantage from this kind of equation in what may be called moral or prudential algebra. Wishing sincerely that you may determine for the best, I am ever, my dear friend, your most affectionately... s/Ben Franklin Solution Decisions should be based on the differences between the alternatives. Here the alternatives are taking the job (Pro) and not taking the job (Con).
41
1-6 Assume that you are employed as an engineer for Wreckall Engineering, Inc., a firm specializing in demolition of high-rise buildings. The firm has won a bid to tear down a 30-story building in a heavily developed downtown area. The crane owned by the company only reaches to 29 stories. Your boss asks you to perform an economic analysis of buying a new crane to complete the job. How would you handle the analysis? Solution The important point of this problem is to realize that your boss may not have recognized what the true problem is in this case. To buy a new crane is only one alternative, and quite likely not the best alternative. Other alternatives: extension on current crane ramp for current crane rent a crane to remove top story explosive demolition . etc. If this is a fixed output project (e.g., fixed fee for demolishing building) we want to minimize costs. Weigh alternatives using economic criteria to choose the best alternative.
"
A # Floor area in ft 2 / floor If the total number of square feet required is 106 , what is the optimal (minimum cost) number of floors?
Solution
X* #
42
1-8 By saving and investing, wisely or luckily or both, Helen finds she has accumulated $400,000 in savings while her salaried position is providing her with $40,000 per year, including benefits, and after income taxes and other deductions. Helen's salaried position is demanding and allows her little free time, but the desire to pursue other interests has become very strong. What would be your advice to her if you were asked? Solution First, Helen should decide what annual income she needs to provide herself with the things she wants. Depending on her age, she might be able to live on the interest income (maybe 10% x $400,000 = $40,000), or a combination of interest and principal. The important thing that Helen should realize is that it may be possible for her to lead a more fulfilling lifestyle if she is fully aware of the time value of money. There are many people with large sums of money in bank checking accounts (drawing no interest) because they can write "free" checks.
1-9 Charles belongs to a square dance club that meets twice each month and has quarterly dues of $9.00 per person. The club moved its meeting place to a location with increased cost. To offset the cost each member agrees to pay 50 cents each time they attend the meeting. Later the treasurer suggests that the quarterly dues be increased to $12.00 per person as an alternative to the meeting charge. Discuss the consequences of the proposal. Do you think the club members would agree to the proposal? Solution The members who attend regularly would pay the same amount with the new dues as with the older method of $9.00 plus 50 cents per meeting. Many would like the added advantage of covering their quarterly expenses in one check. The members who attend infrequently would pay more by the new method and might oppose the action. Since the people who attend infrequently are in the minority in this club, the members voted to approve the proposal.
43
1-10 Sam decides to buy a cattle ranch and leave the big city rat race. He locates an attractive 500-acre spread in Montana for $1,000 per acre that includes a house, a barn, and other improvements. Sam's studies indicate that he can run 200 cow-calf pairs and be able to market 180 500-pound calves per year. Sam, being rather thorough in his investigation, determines that he will need to purchase an additional $95,000 worth of machinery. He expects that supplemental feeds, medications and veterinary bills will be about $50 per cow per year. Property taxes are $4000 per year, and machinery upkeep and repairs are expected to run $3,000 per year. If interest is 10% and Sam would like a net salary of $10,000 per year, how much will he have to get for each 500-pound calf? Solution Land Cost : $500 Acre x $1,000/Acre = $500,000 Machinery: Lump sum = 95,000 Total Fixed Cost $595,000 Assume lands and machinery to have a very long life At 10% Annual Cost = (.10)($595,000) = $59,500 Other Annual Costs: Feeds, medications, vet bills $50 x 200 = $ 10,000 Property taxes 4,000 Upkeep & Repairs 3,000 Salary 10,000 Total Annual Cost 86,500 Net sale price of each calf would have to be: $86,500/180 = $480.56 Note: If Sam were to invest his $595,000 in a suitable investment vehicle yielding 10% interest his salary would be almost six times greater and he could go fishing instead of punching cows.
2-2 A continuously compounded loan has what nominal interest rate if the effective interest rate is 25%? Select one of the five choices below. (a) (b) (c) (d) (e) e1.25 e0.25 log e (1.25) log e (0.25) Other (specify) _________________________
Solution er - 1 = .25 er = 1.25 log e (er) = log e (1.25) r = log e (1.25) Choose c.
45
46
2-3 A continuously compounded loan has what effective interest rate if the nominal interest rate is 25%? Select one of the five choices below. (a) (b) (c) (d) (e) e1.25 e0.25 log e (1.25) log e (0.25) Other (specify) _________________________
2-4 Given: A situation where the annual interest rate is 5%. When continuous compounding is used, rather than monthly compounding, the nominal interest rate. (Select One) (a) Increases (b) Remains the same (c) Decreases Solution The answer is b: Remains the same
2-5 A journalist for a small town newspaper has a weekly column in which he answers questions from the local populace on various financial matters. Below is one such question. Assume you are the journalist and respond to this inquiry. Be brief and specific.
Q: "I put $10,000 in a 12% six month savings certificate. When it matured, I expected to receive interest of $1200 - 12% of $10,000. Instead, I received only $600. Why? Current six month certificates now pay 9% interest. If I put $10,000 in a new six month certificate, I assume (based on my previous experience) I'll get only $450 interest - one half of 9%. Wouldn't my money earn more interest if I deposit it in a savings account paying 5 1/2 percent?"
47
A: The questioner is confused about nominal vs. effective interest rates. The 9% and 12% rates are nominal annual interest rates compounded semi-annually. The effective semi-annual interest rates are 0.09/2 = 0.045 and 0.12/2 = 0.06 hence the interest earned in 6 months would be 0.045 (10,000) = $450 and 0.06 (10,000) = $600. The corresponding effective annual interest rates are (1.045)2 - 1 = 0.092 and (1.06)2 - 1 = 0.1236 The interest rate advertised on the savings account typically is also a nominal annual rate. Most accounts pay interest quarterly or continuously, thus the effective annual interest rate would be either (1.01375)4 - 1 = 0.05614 or neither of which is as good as the six month certificates. e 0.055 - 1 = 0.05654
2-6 A drug dealer will sell goods to his regular customers for $20 immediately or $22 if the payment is deferred one week. What nominal annual interest rate is the dealer receiving? Solution i = 2/20 = 10% r = 52 x 10% = 520%
2-7 A local bank is advertising that they pay savers 6% compounded monthly, yielding an effective annual rate of 6.168%. If $2,000 is placed in savings now and no withdrawals are made, how much interest (to the penny) will be earned in one year? Solution Interest = Effective annual rate x principal = 0.06168 x 2,000 = $123.36 Monthly compounding is irrelevant when the effective rate is known.
2-8 A man decides to put $100 per month into an account paying 12% compounded monthly. Without using formulas or factors (that is, use only basic concepts) determine how much (to the penny) will be in the account immediately after the fourth deposit.
48
Solution
Month 1 2 3 4
2-9 A small company borrowed $10,000 to expand the business. The entire principal of $10,000 will be repaid in 2 years but quarterly interest of $330 must be paid every three months. What nominal annual interest rate is the company paying? Solution Since $330 is interest only for one interest period, then i = 330/10,000 = 3.3% per quarter r = 3.3 x 4 = 13.2% nominal annual
2-10 A store policy is to charge 1% interest each month on the unpaid balance. (a) What is the nominal interest? (b) What is the effective interest? Solution (a) r = mi = 12(1.25) = 15% (b) ieff = (1 + i )n - 1 = (1.0125) 12 - 1 = 16.075% 2-11 Under what circumstances are the nominal and effective annual interest rates exactly equal; or is this never true? Solution The nominal interest rate equals the effective interest rate when there is yearly (annual) compounding (i.e., m = 1).
49
2-12 A small company borrowed $10,000 to expand the business. The entire principal of $10,000 will be repaid at the end of two years but quarterly interest of $335 must be paid every three months. What nominal annual interest rate is the company paying? Solution i = 335/10,000 = 3.35% r = i m = 4 x 3.35 = 13.40%
2-13 E. Z. Marc received a loan of $50 from the S.H. Ark Loan Company that he had to repay one month later with a single payment of $60. What was the nominal annual interest rate for this loan? Solution Interest = $10 in one month i = 10/50 = 20% r = im = 20 x 12 = 240%
2-14 A local college parking enforcement bureau issues parking tickets that must be paid within one week. The person receiving the ticket may pay either $5 immediately, or $7 if payment is deferred one week. What nominal interest rate is implied in the arrangement? Solution i = (7 - 5)/5 = 40% per week r = i m = 52(40) = 2080%
2-15 A deposit of $300 was made one year ago into an account paying monthly interest. If the account now has $320.52, what was the effective annual interest rate? Give answer to 1/100 of a percent. Solution ieff = 20.52/300 = 6.84%
50
2-16 Which is the better investment, a fund that pays 15% compounded annually, or one that pays 14% compounded continuously? Solution i = 15% ; n = 1 F = P (1 + .15) 1 = 1.1500 P = 1.1503 P
2-17 For a nominal interest of 16 percent, what would the effective interest be, if interest is (a) compounded quarterly? (b) compounded monthly? (c) compounded continuously? Solution (a) ieff = [(1.04)4 - 1 ] ( 100) = 16.986% (b) ieff = [(1.01333)12 - 1 ] (100) = 17.222% (c) ieff = ern - 1 = e.16(1) - 1 = 17.35% 2-18 If compounding is weekly and the (one year = 12 months = 48 weeks for this problem) quarterly effective interest rate is 5%, (a) (b) (c) (d) What is the nominal annual interest rate? What is the weekly interest rate? What is the semi-annual effective interest rate? What is the effective interest rate for a two year period?
Solution (a) iP = (1 + r/m)P - 1 " r = m { (1+ iP)1/P - 1 } igr = i12 = .05 = (1 + r/48)12 - 1 r = 48 {1.051/12 - 1 } = 19.555% per year compounded weekly. (b) iWK = r/m = 19.555 / 48 = .4074% per week
51
2-19 If the interest rate is 10% compounded continuously, what is the semi-annual effective interest rate? Solution i t = ert - 1 i 1/2 = e(.1) (1/2) - 1 = e.05 - 1 = 1.0512711 - 1 = 5.127% per 1/2 year 2-20 A grandfather gave his grandchild $100 for his 10th birthday. The childs parents talked him into putting this gift into a bank account so that when he had grandchildren of his own he could give them similar gifts. The child lets this account grow for 50 years, and it has $100000. What was the interest rate of the account? (a) 15.0% (b) 15.8% (c) 14.8% (d) 14.0%
3-2 If you had $1,000 now and invested it at 6%, how much would it be worth 12 years from now? F = 1,000(F/P, 6%, 12) = $2,012.00
53
54
3-4 A young engineer wishes to buy a house but only can afford monthly payments of $500. Thirty year loans are available at 12% interest compounded monthly. If she can make a $5,000 downpayment, what is the price of the most expensive house that she can afford to purchase? Solution i 12%/12 = 1 % n = (30)(12) = 360
3-5 Mr. Beach deposited $200,000 in the Lawrence National Bank. If the bank pays 8% interest, how much will he have in the account at the end of 10 years? Solution F = 200,000(F/P, 8%, 10) = $413,800
3-6 Given:
A = $222
4 5
P = $800 Find: i % Solution P = A(P/A, i %, 5) 800 = 222(P/A, i%, 5) (P/A, i %, 5) = 800/222 = 3.6 From tables i = 12%
3-7 How much should Abigail invest in a fund that will pay 9%, compounded continuously, if she wishes to have $600,000 in the fund at the end of 10 years?
55
3-8 Solve for the unknown interest rate. P = $1,000 Solution A = P(A/P, i %, 10) 238.50 = 1,000(A/P, i%, n) (A/P, i%, n) = 238.50/1,000 = 0.2385 From tables, i = 20% n = 10 years A = $238.50 i=?
3-9 How much will accumulate in an Individual Retirement Account (IRA) in 15 years if $500 is deposited in the account at the end of each quarter during that time? The account earns 8% interest, compounded quarterly. What is the effective interest rate? Solution i = 8/4 = 2 % n = (4)(15) = 60
F = 500 (F/A, 3%, 60) = $57,025.50 Effective interest rate = (1 + .02)4 - 1 = 8.24%
3-10 Solve for the unknown value. Be sure to show your work. P = 1,000 Solution A = 1,000(A/P, 12%, 5) = $277.40 i = 12% n=5 A=?
56
3-11 To offset the cost of buying a $75,000 house, a couple borrowed $12,500 from their parents at 6% nominal interest, compounded monthly. The loan from their parents is to be paid off in five years in equal monthly payments. The couple has saved $11,250. Their total downpayment is therefore $12,500 + 11,250 = $23,750. The balance will be mortgaged at 9% nominal interest, compounded monthly for 30 years. Find the combined monthly payment that the couple will be making for the first five years. Solution Payment to parents: 12,500(A/P, %, 60) = $241.25 Borrowed from bank: 75,000 - 23,750 = $51,250 Payment to bank 51,250(A/P, %, 360) = $412.56 Therefore, monthly payments are $241.25 + 412.56 = $653.81
15C
10C
5C
0 100 200 300 400 Solution 20C(P/A, 12%, 4) - 5C(P/G, 12%, 4) = 100(P/A, 12%, 4) + 100(P/G, 12%, 4) 40.105C = 716.4 C = 17.86
57
3-13 Decide whether each of the three statements below is TRUE or FALSE without referring to your book, notes, or compound interest tables. (a) If interest is compounded quarterly, the interest period is four months. (b) (F/A, 12%, 30) = (F/A, 1%, 360) (c) (F/P, i %, 10) is greater than (P/F, i %, 10) for all values of i% > 0%. Solution (a) FALSE. If interest is compounded quarterly, each interest period is 3 months long. (b) FALSE. If we assume, for example, we are talking about 30 years or 360 months, the (F/A, 12%, 30) does not provide monthly compounding of interest. The (F/A, 1%, 360) does. (This is a common error among beginning students.) (c) TRUE. Since i % > 0 %, (1 + i) > 0. Thus (F/P, i %, 10) = (1+i) 10 > 1 and (P/F, i %, 10) = (1 + i) -10 < 1 for all values of i % > 0%.
3-14 A company borrowed $10,000 at 12% interest. The loan was repaid according to the following schedule. Find X, the amount that will pay off the loan at the end of year 5. Year 1 2 3 4 5 Solution 10,000 = 2,000 (P/A, 12%, 4) + X(P/F, 12%, 5) 3,926 = X(.5674) X = 3,926/.5674 = $6,919.28 Amount $2,000 2,000 2,000 2,000 X
3-15 How much will Thomas accumulate in a bank account if he deposits $3,000 at the end of each year for 7 years? Use interest = 5% per annum.
58
Solution
3-16 You need to borrow $10,000 and the following two alternatives are available at different banks: (a) Pay $2,571 at the end of each year for 5 years, starting at the end of the first year. (5 payments in total.) (b) Pay $207.58 at the end of each month, for 5 years, starting at the end of the first month. (60 payments in total.) On the basis of the interest rate being charged in each case, which alternative should you choose? Solution Alternative a: 10,000 = 2,571(P/A, i, 5) (P/A, i, 5) = 10,000/2,571 = 3.890 From tables; i "#" 9% (nominal = effective rate since compounded annually) Alternative b: n = 5 x 12 = 60 10,000 = 207.58(P/A, i, 60) (P/A, i, 60) = 10,000/ 207.58 = 48.174 From tables i = .75% The nominal annual interest rate is: 12 x .75 = 9% but the effective interest rate is: (1 + (0.09/12))12 - 1 = 9.38% Therefore, choose the first alternative.
59
3-17 Find the Uniform Equivalent for the following cash flow diagram if i = 18%. Use the appropriate gradient and uniform series factors. 0 1 2 3 4 5 6 7 8 9 10 11 12
400 Solution X1
350
300
250
200
150
150
150
150 150
(=)
3 4 5 6 7 8 9 10 11 12
1 0 0
150 X2 0 1 3
150
150
150
150 150
150
(+)
4 5 6 7 8 9 10 11 12
250 X3 0 1 3 0
250 250
(-)
4 50 5 6 7 8 9 10 11 12
100
150
200
X1 = 150 (P/A, 18%, 10) = 674.10 X2 = 250 (P/A, 18%, 5) = 781.75 X3 = 50 (P/G, 18%, 5) = 261.55 X = X1 + X2 - X3 = $1,194.30 P = 1,194.30 (P/F, 18%, 2) = 857.75 A = 857.75 (A/P, 18%, 12) = $178.93
60
3-18 Below is an equation to compute an equivalent annual cash flow (EACF). Determine the values of the net cash flow series that is implied by the equation. EACF = [-8000 - 8000(P/F, 10%,1)](A/P, 10%, 8) + 2000 +500(P/G, 10%, 4)(P/F, 10%, 1)(A/P, 10%, 8) + 750[(P/F, 10%.6) - (P/F, 10%, 8)](A/P, 10%, 8) Time 0 1 2 3 4 5 6 7 8 Solution Time 0 1 2 3 4 5 6 7 8 Net Cash Flow Series - 8,000 = - 8,000 -6,000 = - 8,000 + 2,000 2,000 = + 2,000 2,500 = + 2,000 + 500 3,000 = + 2,000 + 1,000 3,500 = + 2,000 + 1,500 2,750 = + 2,000 2,000 = + 2,000 1,250 = + 2,000 Net Cash Flow Series
+750 - 750
3-19 On July 1 and September 1, Joan placed $2,000 into an account paying 6% compounded monthly. How much was in the account on October 1?
61
Deposit 0 $2,000.00 0
3-20 Using a credit card, Ben Spendthrift has just purchased a new stereo system for $975 and will be making payments of $45 per month. If the interest rate is 18% compounded monthly, how long will it take to completely pay off the stereo? Solution i = 18%/12 = 1% 975 = 45(P/A, 1%, n) (P/A, 1%, n) = 975/45 = 21.667 From tables n is between 26 and 27 months. The loan will not be completely paid off after 26 months. Therefore the payment in the 27th month will be smaller.
3-21 An engineer on the verge of retirement has accumulated savings of $100,000 that are in an account paying 6% compounded quarterly. The engineer wishes to withdraw $6000 each quarter. For how long can she withdraw the full amount? Solution i = 6%/4 = 1% 6,000 = 100,000(A/P, 1%, n) (A/P, 1%, n) = 0.0600 From tables n = 19 quarters or 4years Note: This leaves some money in the account but not enough for a full $6,000 withdrawal
62
3-22 Charles puts $25 per month into an account at 9% interest for two years to be used to purchase an automobile. The car he selects then costs more than the amount in the fund. He agrees to pay $50 per month for two more years, at 12% interest, and also makes cash downpayment of $283.15. What is the cost of the automobile? Solution P = 283.15 + 25(F/A, %, 24) + 50(P/A, 1%, 24) = 283.15 + 654.70 + 1,062.15 = $2,000 " cost of auto
3-23 Explain in one or two sentences why (A/P, i%, infinity) = i. Solution In order to have an infinitely long A series, the principal must never be reduced. For this to happen only the interest earned each period may be removed. Removing more than the interest would deplete the principal so that even less interest is available the next period.
3-24 A bank is offering a loan of $20,000 with a nominal interest rate of 12%, payable in 48 months. (a) Calculate first the monthly payments. (b) This bank also charges a loan fee of 4% of the amount of the loan, payable at the time of the closing of the loan (that is, at the time they give the money to the borrower). What is the effective interest rate they are charging? Solution (a) The monthly payments: n = 48 ; i = 12%/12 = 1% per period (month) 20,000(A/P, 1%, 48) = $526.00
63
Recalling that A = P(A/P, i, n) 526 = 19,200(A/P, i, 48) (A/P, i, 48) = 526/19,200 = 0.02739 for i = 1% the A/P, factor @ n = 48 = 0.0278 for i = 1% the A/P factor @ n = 48 = 0.0263
by interpolation i # 1 + ((.0263 - .02739)/(.0263 - .0278)) i "# 1.1817% the effective interest rate = (1 + 0.011817)12 - 1 = 0.1514 = 15.14%
3-25 Find A if A = $3,000(A/P, 13.5%, #). Solution A=P x i when n = # A = 3,000 x .135 = $405
3-26 Henry Fuller purchases a used automobile for $4,500. He wishes to limit his monthly payment to $100 for a period of two years. What downpayment must he make to complete the purchase if the interest rate is 9% on the loan? Solution P = P' + A(P/A, %, 24) 4,500 = P' + 100(21.889) P' = 4,500 - 2,188.90 = $2,437.60 " downpayment
64
3-27 If you can make 6% interest on your money, how much is $1000 paid to you 12 years in the future worth to you now? Solution P = 1,000(P/F, 6%, 12) = = $497.00
3-28 Find the present equivalent of the following cash flow diagrams if i = 18%. 100 80
60 2
40 3
20 4 5 6 7 8 9 10
20
40
60
80
100
3-29 Downtown is experiencing an explosive population growth of 18% per year. At the end of 2002 the population was 16,000. If the growth rate continues unabated, at the end of how many years will the population have passed 75,000? Solution Use i = 18% to represent the growth rate. 75,000 = 16,000(F/P, 18%, n) (F/P, 18%, n) = 75,000/16,000 = 4.6875 From tables n is 10 Note that population would not have passed 75,000 after 9 years.
65
3-30 To start business, ECON ENGINEERING has just borrowed $500,000 at 6%, compounded quarterly, which will be repaid by quarterly payments of $50,000 each, with the first payment due in one year. How many quarters after the money is borrowed is the loan fully paid off? Solution i = 6/4 = 1% 500,000 = 50,000(P/A, 1%, n)(P/F, 1%3) (P/A, 1%, n) = 500,000/[50,000(.9563)] = 10.46 From tables n = 12 payments plus 3 quarters without payments equal 15 quarters before loan is fully paid off.
3-31 Given: 125 0 i = 10% P Solution P = [125(P/A, 10%, 4) + 25(P/G, 10%, 4)](P/F, 10%, 1) = 459.73 150 175 200 Find: P
3-32 If the interest rate is 6% compounded quarterly, how long (number of quarters) does it take to earn $100 interest on an initial deposit of $300? Solution i = 6%/4 = 1% 400 = 300(F/P, 1%, n) (F/P, 1%, n) = 400/300 = 1.333 From tables n = 20 quarters
66
3-33 A tractor is bought for $125,000. What is the required payment per year to completely pay off the tractor in 20 years, assuming an interest rate of 6%? (a) (b) (c) (d) $ 1,150 $ 5,550 $10,900 $12,750
67
68
4-2 What amount, at 3.25% annual interest, will generate annual payments of $60,00 for a period of 20 years? Use Excel to solve. Solution
4-3 Using Excel, determine the payment on a 5-year automobile loan of $15,000 with 1.9% interest and monthly payments. Solution
69
4-4 Determine the equation that must be supplied to complete the Excel spreadsheet. What is the value for B12?
70
4-5 ABC Block Inc. is considering replacing their wood shaving machine. The industrial engineer for ABC has initially selected the QuickShave machine. The machine has a first cost of $35,000 and a salvage value of $6,000 at the end of its seven year useful life. The cash flows associated with the machine are presented below. Assuming ABCs MARR is, determine if the machine should be purchased. Year Cash flow($) 1 7,000 2 7,000 3 7,000 4 5,500 5 5,500 6 4,500 7 4,500
Sum column C
71
4-6 A $10,000, 5% bond issued by Holiday Shipping is selling for $9,500. The bond pays interest annually and matures in ten years. Determine the rate of return for the bond. Solution
72
4-7 A machine costing $35,000 has associated income of $8,000 the first year with yearly decreases of $1,000 for the eight year life of the machine. (The 1st year = $8,000, the 2nd year $7,000, etc.) Using Excel determine the payback for the machine. Solution
73
4-8 Using Excel, calculate the depreciation per year and the book value each year of an asset costing $56,000 with a salvage value of $8,000 at the end of its eight year useful life. Also graph the book value vs. year. Solution
74
75
5-2 Using an interest rate of 8%, what is the capitalized cost of a tunnel to transport water through the Lubbock mountain range if the first cost is $1,000,000 and the maintenance costs are expected to occur in a 6-year cycle as shown below? End of Year: Maintenance: Solution Capitalized Cost = PW of Cost for an infinite time period. As the initial step, compute the Equivalent Annual Maintenance Cost. EAC = 35,000 + [10,000(F/A, 8%, 3) + 15,000](A/F, 8%, 6) = $41,468.80 For n = , P = A/I Capitalized Cost = 1,000,000 + (41,468.80/0.08) = $1,518,360. 1 $35,000 2 $35,000 3 $35,000 4 $45,000 5 $45,000 6 $60,000
75
76
5-3 The investment in a crane is expected to produce profit from its rental as shown below, over the next six years. Assume the salvage value is zero. What is the present worth of the investment, assuming 12% interest? Year 1 2 3 4 5 6 Solution P = 15,000(P/A, 12%, 6) - 2,500(P/G, 12%, 6) = $39,340 Profit $15,000 12,500 10,000 7,500 5,000 2,500
5-4 A tax refund expected one year from now has a present worth of $3000 if i = 6 %. What is its present worth if i = 10 %? Solution Let x = refund value when received at the end of year 1 = 3,000(F/P, 6%, 1); PWx = x(P/F, 10%, 1) Therefore the PW if i = 10% = 3,000(F/P, 6%, 1)(P/F, 10%, 1) = $2,890.94
5-5 It takes $10,000 to put on a Festival of Laughingly Absurd Works each year. Immediately before this year's FLAW, the sponsoring committee finds that it has $60,000 left in an account paying 8% interest. After this year, how many more FLAWs can be sponsored without raising more money? Think Carefully! Solution 60,000 - 10,000 = 10,000(P/A, 8%, n) (P/A, 8%, n) = 50,000/10,000 =5 Therefore n = 6 which is the number of FLAWs after this year's. There will be some money left over but not enough to pay for a 7th year.
77
5-6 An engineer is considering buying a life insurance policy for his family. He currently owes about $77,500 in different loans, and would like his family to have an annual available income of $35,000 indefinitely (that is, the annual interest should amount to $35,000 so that the original capital does not decrease). (a) He feels he can safely assume that the family will be able to get a 4% interest rate on that capital. How much life insurance should he buy? (b) If he now assumes the family can get a 7% interest rate, calculate again how much life insurance should he buy. Solution (a) If they get 4% interest rate: n= A = Pi or P = A/i = 35,000/0.04 = 875,000 Total life insurance = 77,500 + 875,000 = $952,500 (b) If they can get 7% interest rate: again n = P = A/i = 35,000/0.07 = 500,000 Total life insurance = 77,500 + 500,000 = $577,500
5-7 The winner of a sweepstakes prize is given the choice of one million dollars or the guaranteed amount of $80,000 a year for 20 years. If the value of money is taken at a 5% interest rate, which choice is better for the winner? Solution Alternative 1: P = $1,000,000 Alternative 2: P = 80,000K(P/A, 5%, 20) = 81K(7.469) = $996,960 Choose alternative 1: take $1,000,000 now
5-8 The annual income from an apartment house is $20,000. The annual expense is estimated to be $2000. If the apartment house could be sold for $100,000 at the end of 10 years, how much could you afford to pay for it now, with 10% considered a suitable interest rate?
78
Solution
P = (AINCOME - AEXPENSES)(P/A, i %, n) + FRE-SAlE(P/F, i %, n) = (20,000 - 2,000)(P/A, 10%, 10) + 100,000(P/F, 10%, 10) = $149,160 5-9 A scholarship is to be established that will pay $200 per quarter at the beginning of Fall, Winter, and Spring quarters. It is estimated that a fund for this purpose will earn 10% interest, compounded quarterly. What lump sum at the beginning of Summer quarter, when deposited, will assure that the scholarship may be continued into perpetuity? Solution P = 200(P/A, 2 1/2 %, 3) = 571.20 A' = 571.20(A/P, 2 1/2 %, 4) = 151.82 For n = , P' = A' / i = 151.82 / .025 = $6,073 deposit 5-10 Your company has been presented with an opportunity to invest in a project. The facts on the project are presented below: Investment required Salvage value after 10 years Gross income expected from the project Operating costs: Labor Materials, licenses, insurance, etc Fuel and other costs Maintenance costs $60,000,000 0 20,000,000/yr 2,500,000/yr 1,000,000/yr 1,500,000/yr 500,000/yr
The project is expected to operate as shown for ten years. If your management expects to make 25% on its investments before taxes, would you recommend this project? Solution PW = -60,000,000 + 14,500,000(P/A, 25%, 10) = -$8,220,500 Reject due to negative NPW
79
100
150
200
250
300
350
300
250
200
150
100
Solution P1 0 1 2 3 4
(=)
5 6 7 8 9 10
100
150 P2
150
150
150
150
150
150
150
150
150
(+)
1 2 50 3 4 5 6 7 8 9 10
100
150
200
250
300
P3 0 1 2 3 4
(-)
5 6 7
350 8
400 9
450 10
100
200
300
400
P1 = 100 + 150(P/A, 18%, 10) = 774.10 P2 = 50(P/G, 18%, 10) = 717.60 P3 = 100(P/G, 18%, 6)(P/F, 18%, 4) = 365.34 P = P1 + P2 + P3 = $1,126.36
500
80
5-12 A couple wants to begin saving money for their child's education. They estimate that $10,000 will be needed on the child's 18th birthday, $12,000 on the 19th birthday, $14,000 on the 20th birthday, and $16,000 on the 21st birthday. Assume an 8% interest rate with only annual compounding. The couple is considering two methods of setting aside the needed money. (a) How much money would have to be deposited into the account on the child's first birthday (note: a child's "first birthday" is celebrated one year after the child is born) to accumulate enough money to cover the estimated college expenses? (b) What uniform annual amount would the couple have to deposit each year on the child's first through seventeenth birthdays to accumulate enough money to cover the estimated college expenses? Solution
note: year zero corresponds to child's 1st birthday
10K
14K 12K 18
16K
(a)
8 10 12 14 16 20
Let F = the $s needed at the beginning of year 16 = 10,000(P/A, 8%, 4) + 2,000(P/G, 8%, 4) = 42,420 The amount needed today P = 42,420(P/F, 8%, 16) = $12,382.40 P' (b) 12,382.40 0 2 4
Year 1 indicates childs first birthday
10
12
14
16
81
5-13 Assume you borrowed $50,000 at an interest rate of 1 percent per month, to be repaid in uniform monthly payments for 30 years. In the 163rd payment, how much of it would be interest, and how much of it would be principal? Solution In general, the interest paid on a loan at time t is determined by multiplying the effective interest rate times the outstanding principal just after the preceding payment at time t - 1. To find the interest paid at time t = 163,(call it I163) first find the outstanding principal at time t = 162 (call it P162). This can be done by computing the future worth at time t = 162 of the amount borrowed, minus the future worth of 162 payments. Alternately, compute the present worth, at time 162, of the 198 payments remaining. The uniform payments are 50,000(A/P, 1%, 360) = $514.31, thus P162 = 50,000(F/P, .01, 162) - 514.31(F/A, 1%, 162) = 514.31(P/A, 1%, 198) = $44,259.78 The interest is I163 = 0.01(44,259.78) = $442.59 and the principal in the payment is $514.31 - 442.59 = $71.72
5-14 A municipality is seeking a new tourist attraction, and the town council has voted to allocate $500,000 for the project. A survey shows that an interesting cave can be enlarged and developed for a contract price of $400,000. It would have an infinite life. The estimated annual expenses of operation are: Direct Labor Maintenance Electricity $30,000 15,000 5,000
The price per ticket is to be based upon an average of 1000 visitors per month. If money is worth 8%, what should be the price of each ticket? Solution If the $100,000 cash, left over after developing the cave, is invested at 8%, it will yield a perpetual annual income of $8000. This $8000 can be used toward the $50,000 a year of expenses. The balance of the expenses can be raised through ticket sales, making the price per ticket $42,000/12,000 tickets = $3.50/ticket
82
5-15 A middle-aged couple has made an agreement with Landscapes Forever Company, a gravesite landscaping and maintenance firm. The agreement states that Landscapes Forever will provide "deluxe landscaping and maintenance" for the couple's selected gravesite forever for an annual fee of $1000. To arrange payment, the couple has set us a variable rate perpetual trust fund with their bank. The bank guarantees that the trust fund will earn a minimum of 5% per year. Assume that the services of Landscapes Forever will not be needed until after the wife has died, and that she lives to the ripe old age of 100. (a) What is the smallest amount of money that the couple would have to deposit into the trust fund? (b) Suppose that the couple made this minimum deposit on the wife's 50th birthday, and suppose that the interest rate paid by the trust fund fluctuated as follows: Wife's Age 50 - 54 55 - 64 65 - 74 75 - 84 Interest Rate 5% 10% 15% 20%
What is the largest sum of money that could be withdrawn from the trust fund on the wife's 85th birthday, and still have the perpetual payments to Landscapes Forever made?
83
Trust Fund Balance 20,000.00(F/P, 5%, 5) = 25,520.00(F/P, 10%, 10) = 66,198.88(F/P, 15%, 10) = 267,840.00(F/P, 20%, 10) = 25,520.00 66,198.88 267,840.00 1,658,469.43
Therefore the largest sum which could be withdrawn from the trust fund is 1,658,469.43 - 20,000 = $1,632,469.43
5-16 A local car wash charges $3.00 per wash or the option of paying $12.98 for 5 washes, payable in advance with the first wash. If you normally washed your car once a month, would the option be worthwhile if your minimum attractive rate of return(MARR) is 12% compounded annually? Solution First, convert the effective annual MARR to its equivalent effective monthly rate: (1.12)1/12 - 1 = 0.9489% Any measure of worth could now be used, but net present value is probably the easiest. NPV = (-12.98 + 3.00) + 3.00(P/A, .9489%, 4) = $1.74 > 0 Therefore, the option is economical.
5-17 A project has a first cost of $14,000, uniform annual benefits of $2400, and a salvage value of $3000 at the end of its 10 year useful life. What is its net present worth at an interest rate of 12%? Solution PW = -14,000 + 2,400(P/A, 20%, 10) + 3,000(P/F, 20%, 10) = $526.00
84
5-18 A person borrows $5,000 at an interest rate of 18%, compounded monthly. Monthly payments of $180.76 are agreed upon. (a) What is the length of the loan? (Hint: it is an integral number of years.) (b) What is the total amount that would be required at the end of the sixth month to payoff the entire loan balance? Solution (a) P = A(P/A, i%, n) 5,000 = 180.76(P/A, 1 %, n) (P/A, %, n) = 5,000/180.76 = 27.66 From the 1% interest table n = 36 months = 6 years. (b) 180.762 + 180.762(P/A, 1%, 30) = $4,521.91
5-18 A $50,000 30-year loan with a nominal interest rate of 6% is to be repaid in payments of $299.77 per month (for 360 months). The borrower wants to know how many payments, N*, he will have to make until he owes only half of the amount he borrowed initially. His minimum attractive rate of return (MARR) is a nominal 10% compounded monthly. Solution The MARR is irrelevant in this problem. The outstanding principal is always equal to the present worth of the remaining payments when the payments are discounted at the loan's effective interest rate. Therefore, let N' be the remaining payments. (50,000) = 299.77(P/A, %, N) (P/A, %, N) = 83.397 N = 108.30 108 So, N* = 360 - N = 252 payments
5-19 A project has a first cost of $10,000, net annual benefits of $2000, and a salvage value of $3000 at the end of its 10 year useful life. The project will be replaced identically at the end of 10 years, and again at the end of 20 years. What is the present worth of the entire 30 years of service if the
85
5-20 The present worth of costs for a $5,000 investment with a complex cash flow diagram is $5265. What is the capitalized cost if the project has a useful life of 12 years, and the MARR is 18%? Solution Capitalized Cost = 5,265(A/P, 18%, 12)(P/A, 18%, ) = 5,265(.2086)(1/.18) = $6,102 5-21 A used car dealer tells you that if you put $1,500 down on a particular car your payments will be $190.93 per month for 4 years at a nominal interest rate of 18%. Assuming monthly compounding, what is the present price you are paying for the car? Solution A P = 190.93 per period, i = .18/12 = .015, n = 4 x 12 = 48
5-22 What is the price of a 3-year Savings Certificate worth $5,000 three years hence, at 12 % interest, compounded continuously, with loss of interest if taken out before three years? Solution P = Fe - r n = $5,000e -(0.12) 3 = 5,000e -0.36 = $3,488.50
86
5-23 If the current interest rate on bonds of a certain type is 10% nominal, compounded semiannually, what should the market price of a $1,000 face value, 14 percent bond be? The bond will mature (pay face value) 6-1/2 years from today and the next interest payment to the bondholder will be due in 6 months. Solution Bi-yearly interest payment = .07(1,000) = $70 PV = $70(P/A, 5%, 13) + $1,000(P/F, 5%, 13) = $1,187.90
5-24 What is the Present Worth of a series that decreases uniformly, by $20 per year, from $400 in Year 11 to $220 in Year 20, if i equals 10 %? Solution PW = [400(P/A, 10%, 10) - 20(P/G, 10%, 10)](P/F, 10%, 10) = $770.91
5-25 Many years ago BigBank loaned $12,000 to a local homeowner at a nominal interest rate of 4.5%, compounded monthly. The terms of the mortgage called for payments of $60.80 at the end of each month for 30 years. BigBank has just received the 300th payment, thus the loan has five more years to maturity. The outstanding balance is now $3,261.27. Because BigBank currently charges a nominal 13% compounded monthly on home mortgages, it could earn a better return on its money if the homeowner would pay off the loan now; however, the bank realizes the homeowner has little economic incentive to do that with such a low interest rate on the loan. Therefore, BigBank plans to offer the homeowner a discount. If the homeowner will pay today an amount of $3,261.27 - D, where D is the dollar amount of the discount, BigBank will consider the loan paid in full. If for BigBank the minimum attractive rate of return(MARR) is 10% (effective annual rate), what is the maximum discount, D, it should offer the homeowner?
87
Any measure of worth could be used. The appropriate discount rate is the effective monthly MARR: (1.1)1/12 - 1 = .00797 Therefore, using NPV = 0 = -3261.27 + D + 60.80(P/A, .797%, 60) D=$370.60
5-26 A resident will give money to his town to purchase a Vietnam veteran memorial statue and to maintain it at a cost of $500 per year forever. If an interest rate of 10% is used, and the resident gives a total of $15,000; how much can be paid for the statue? Solution Capitalized Cost = 15,000 = P + 500(P/A, 10%, ) P = 15,000 - 500(1/.1 ) = $10,000
5-27 A rich widow decides on her 70th birthday to give most of her wealth to her family and worthy causes, retaining an amount in a trust fund sufficient to provide her with an annual end of year payment of $60,000. If she is earning a steady 10% rate of return on her investment, how much should she retain to provide these payments until she is 95(the last payment the day before she is 96)? If she dies on her 85th birthday, how much will remain in the trust fund? Solution P = 60K(P/A, 10%, 26) = 60K(9.161) = $549,660 P' = 60K(P/A, 10%, 11) = 60K(6.495) = $389,700
88
5-28 J.D. Homeowner has just bought a house with a 20-year, 9%, $70,000 mortgage on which he is paying $629.81 per month. (a) If J.D. sells the house after ten years, how much must he give the bank to completely pay off the mortgage at the time of the 120th payment? (b) How much of the first $379.33 payment on the loan is interest? Solution (a) P = 629.81 + 629.81(P/A, %, 120) = $49,718.46 (b) $70,000 x 0.0075 = $525
5-29 Dolphin Inc. trains mine seeking dolphins in a 5-mine tank. They are considering purchasing a new tank. The U.S. Navy will pay $105,000 for each dolphin trained and a new tank costs $750,000 and realistic dummy mines cost $250,000. The new tank will allow the company to train 3 dolphins per year and will last 10 years costing $50,000 per year to maintain. Determine the net present value if the MARR equals 5%? Solution NPV = -Cost - Cost of Mines - Annual Maintenance(P/A, 5%, 10) + Income(P/A, 5%, 10) = -750,000 - 250,000(5) - 50,000(P/A, 5%, 10) + 105,000(3)(P/A, 5%, 10) = $46,330
6-2 Suppose you wanted to buy a $100,000 house. You have $20,000 cash to use as the down payment. The bank offers to loan you the remainder at 6% nominal interest. The term of the loan is 20 years. Compute your monthly loan payment assuming the payment is the same for all months. Solution Amount of loan: $100,000 - $20,000 = $80,000 i = 6%/12 = % per month A = 80,000(A/P, %, 240) = $572.80 per month n = 20 x 12 = 240 periods
89
90
6-3 Lester Peabody decides to install a fuel storage system for his farm that will save him an estimated 6.5 cents/gallon on his fuel cost. He uses an estimated 20,000 gallons/year on his farm. Initial cost of the system is $10,000 and the annual maintenance is a uniform gradient amount of $25. After a period of 10 years the estimated salvage is $3,000. If money is worth 12%, is it a wise investment? Solution EAC =(10,000 - 3,000)(A/P, 12%, 10) + 3,000(.12) + 25(A/G, 12%, 10) = $1,688.63 EAB = 20,000(.065) = $1,300 EAW = -$388.63 ! not a wise investment
6-4 The returns for a business for five years are as follows: $8,250, $12,600, $9,750, $11,400, and $14,500. If the value of money is 12%, what is the equivalent annual benefit for the five-year period? Solution PW = 8,250(P/F, 12%, 1) + 12,600(P/F, 12%, 2) + 9,750(P/F, 12%, 3) + 11,400(P/F, 12%, 4) + 14,500(P/F, 12%, 5) = $39,823 EAB = 39,823(A/P, 12%, 5) = $11,047
6-5 The local loan shark has loaned you $1000. The interest rate you must pay is 20%, compounded monthly. The loan will be repaid by making 24 equal monthly payments. What is the amount of each monthly payment? Solution A = 1,000(A/P, 20%/12, 24) There is no 1-2/3% compound interest table readily available. Therefore the capital recovery factor must be calculated. (A/P, 1.666%, 24) = [0.01666(1.01666)24] / [(1.01666)24 - 1] = 0.050892 A = 1,000(0.050892) = $50.90
91
6-7 Several companies offer "instant cash" plans to holders of their credit cards. A typical plan permits card holders to "draw" cash up to a preset limit. At the time the cash is drawn, a special charge of 4% of the amount drawn is charged to the card holder's account. Then the card holder repays the debt (the original amount drawn plus the special charge) by making a series of equal monthly payments. Each month the company adds a finance charge of 1% of the previous month's unpaid balance to the account balance. If the card holder "draws" $150, a $6 special charge will be made and the card holder will make a series of monthly payments of $9.95. (a) How many payments will be required? (b) What "true" (effective) annual interest rate does the card holder pay? Solution (a) 9.95(P/A, 1%, n) = 156 (P/A, 1%, n) = 15.678 From compound interest tables n = 18 + a very slight amount PW of payments = 9.95(P/A, 1%, 18) = $155.95 for 18 payments FW of balance = 0.05(F/P, 1%, 19) = $0.07 So there are 18 payments of $9.95 and a final payment of 7 cents. (b) 150 = 9.95(P/A, i%, 18) + 0.07(P/F, i%, 19) solve for i, solution using tables: try i = 1% NPW = -150 + 9.95(P/A 1%, 18) + 0.07(P/F, 1%, 19) = $2.55 try i = 2% NPW = -150 + 9.95(P/A, 2%, 18) + 0.07(P/F, 2%, 19) = -$0.78
interpolate: i = 1% + [(2.55 - 0)/(2.55 (-0.78)](%) = 1.9414% Effective annual interest rate = (1 + 0.019414)12 - 1 = 0.2595 = 25.95 %
6-8 If $15,000 is deposited into a savings account that pays 4% interest compounded quarterly, how much can be withdrawn each quarter for five years? Solution i = 4%/4 = 1% per quarter A = 15,000(A/P, 1%, 20) = $831.00 per quarter n = 5 x 4 = 20 periods
92
6-9 What uniform annual payment for 12 years is equivalent to receiving all of these: $ 3,000 20,000 4,000 800 10,000 at the end of each year for 12 years today at the end of 6 years at the end of each year forever at the end of 15 years
Use an 8% interest rate. Solution A1 = $3,000 A2 = 20,000(A/P, 8%, 12) = $2,654 A3 = 4,000(P/F, 8%, 6)(A/P, 8%, 12) = $334.51 A4 =(800/.08)(A/P, 8%, 12) = $1,327 A5 = 10,000(P/F, 8%, 15)(A/P, 8%, 12) = $418.27
# Ai
i "1
6-10 For the following cash flow diagram, which equation properly calculates the uniform equivalent? (a) (b) (c) (d) (e) A = 100(A/P, i, 3) + 100(A/F, i, 3) A = 100(A/P, i, 15) A = 100(A/F, i, 3) + 100(A/P, i, 15) A = 100(A/F, i, 3) + 100(A/F, i, 15) A = 100(A/F, i, 3)
12
15
100 Solution
100
100
100
100
100
93
6-11 A project has a first cost of $75,000, operating and maintenance costs of $10,000 during each year of its 8 year life, and a $15,000 salvage value. What is its equivalent annual cost (EAC) if the interest rate is 12%? Solution EAC = 75,000(A/P, 12%, 8) + 10,000 - 15,000(A/F, 12%, 8) = $23,878.00
6-12 A land surveyor just starting in private practice needs a van to carry crew and equipment. He can lease a used van for $3,000 per year, paid at the beginning of each year, in which case maintenance is provided. Alternatively, he can buy a used van for $7,000 and pay for maintenance himself. He expects to keep the van three years at which time he could sell it for $1,500. What is the most he should pay for uniform annual maintenance to make it worthwhile buying the van instead of leasing it, if his MARR is 20%? Solution Lease: EAC = 3,000(F/P, 20%, 1) = 3,000(1.20) = 3,600 Buy: EAC = 7,000(A/P, 20%, 3) + M - 1,500(A/F, 20%, 3) M = 3,600 - 2,910.85 = $ 689.15
6-13 A foundation supports an annual seminar on campus by using the earnings of a $50,000 gift. It is felt that 10% interest will be realized for 10 years, but that plans should be made to anticipate an interest rate of 6% after that time. What uniform annual payment may be established from the beginning, to fund the seminar at the same level into infinity? Solution A=?
n = 10 i = 10% P P'
n=! i = 6%
94
6-14 A consumer purchased new furniture by borrowing $1,500 using the store's credit plan which charges 18% compounded monthly. (a) What are the monthly payments if the loan is to be repaid in 3 years? (b) How much of the first payment is interest? (c) How much does the consumer still owe just after making the 20th payment? Solution (a) i = 18%/12 = 1% per month, A = 1,500(A/P, 1%, 36) = $54.30 per month (b) Int. = Principal x Monthly interest rate Int. = 1,500 x 0.015 = $22.50 (c) P = 54.30(P/A, 1%, 16) = $767.31 n = 12 x 3= 36 periods
6-15 A 30-year mortgage of $100,000 at a 6% interest rate had the first payment made on September 1, 1999. What amount of interest was paid for the 12 monthly payments of 2002? Solution Monthly payment A = 100,000(A/P, %, 360) = $599.55 Interest periods remaining Jan 1, 2002 = 331 Jan 1, 2003 = 319
95
6-16 A grateful college graduate makes a donation of $2,000 now and will pay $37.50 per month for 10 years to establish a scholarship. If interest in the fund is computed at 9%, what annual scholarship may be established? Assume the first scholarship will be paid at the end of the first year. Solution P = 2,000 + 37.50(P/A, %, 120) = 4,960.33 A = Pi = 4,960.33(.09) = $446.43 scholarship
6-17 A rich folk singer has donated $500,000 to endow a university professorial chair in Bohemian Studies. If the money is invested at 8.5%, how much can be withdrawn each year, ad infinitum, to pay the Professor of B.S.? Solution A = 500,000(A/P, 8.5%, $) = 500,000(.085) =$42,500
6-17 For the cash flow shown below, what would an equivalent sum of money be (a) Now? (b) Two years from now? (c) As a five year annuity, starting at the end of the first year? $1,000 i = 20% 0 Solution (a) PV = $1,000(P/F, 20%, 5) = $401.90 (b) $401.90(F/P, 20%, 2) = $578.74 (c) A = $1,000(A/F, 20%, 5) = $134.40 1 2 3 4 5
96
6-18 A project requires an initial investment of $10,000 and returns benefits of $6,000 at the end of every 5th year thereafter. If the minimum attractive rate of return (MARR) is 10%, should the project be undertaken? Show supporting calculations. Solution 6,000 6,000 6,000 6,000 6,000
!
5 10,000 EAW = 6,000(A/F, 10%, 5) - 10,000(A/P, 10%, $) = - $17.20 The project should not be undertaken. 10 15 20 25
6-19 On January 1st an engineering student projects a need of $2,400 on December 31st. What amount must be deposited in the credit union each month if the interest paid is 3%, compounded monthly? Solution A = 2,400(A/F, , 12) = $197.28
6-20 Given:
97
6-21 Assuming monthly payments, which would be the better financing plan on the same $9,000 car? (a) 6% interest on the full amount for 48 months, compounded monthly. (b) A $1,000 rebate(discount) and 12% interest on the remaining amount for 48 months, compounded monthly. Solution (a) A = 9,000(A/P, %, 48) = $211.50/mo. (b) A = 8,000(A/P, 1%, 48) = $210.40/mo. Choose plan b.
6-22 The initial cost of a pickup truck is $11,500 and will have a salvage value of $4,000 after five years. Maintenance is estimated to be a uniform gradient amount of $150 per year (first year maintenance = zero), and the operation cost is estimated to be 30 cents per mile for 300 miles per month. If money is worth 12%, what is the equivalent annual cost (EAC) for the truck expressed as a monthly cost? Solution EAC = [(11,500 - 4,000)(A/P, 1%, 60) + 4,000(.01) + 150(A/G, 12%, 5)]/12 + 300(.30) = $318.69 per month
6-23 Twenty five thousand dollars is deposited in a bank trust account that pays 18% interest, compounded semiannually. Equal annual withdrawals are to be made from the account, beginning one year from now and continuing forever. Calculate the maximum amount of the equal annual withdrawal. Solution i =18%/2 = 9% per semi-annual period A = Pi = 25,000(.09) = 2,250 W = 2,250(F/A, 9%, 2) = $4,702.50
98
6-24 A truck, whose price is $18,600, is being paid for in 36 uniform monthly installments, including interest at 10 percent. After making 13 payments, the owner decides to pay off the remaining balance of the purchase price in one lump sum. How much is the lump sum? Solution In problems like this the lump sum is the present worth of all the future(unpaid) payments. So to solve the problem compute the payment and then compute the PW of the unpaid payments at the stated interest rate. A = 18,600(A/P, .83%, 36) = 18,600[(.00833(1 + .00833)36)/((1 + .00833)36 - 1)] = $600.22 After 13 months: 36 - 13 = 23 payments remain P = 600.22(P/A, .83%, 23) = 600.22[((1 + .00833)23 - 1)/(.00833(1 + .00833)23)] = $12,515.45
6-25 If the interest rate is 10% and compounding is semiannual, what series of equal annual transactions is equivalent to the following series of semiannual transactions? The first of the equal annual transactions is to occur at the end of the second year and the last at the end of the fourth year.
Time (yr) Period Cash Flow 0 0 $0 1 600 1 2 500 3 400 2 4 300 5 200 3 6 100 7 300 4 8 500 9 700 5 10 900 5 11 1100
Solution P = 600(P/A, 5%, 5) - 100(P/G, 5%, 5) + [100(P/A, 5%, 6) + 200(P/G, 5%, 6)](P/F, 5%, 5) = 4,046.80
Effective i =(1 + 0.10/2)2 - 1 = 10.25% Sum at end of Year 1: F = P(F/P, 10.25%, 1) = 4,461.60 Equal Annual Payments: A = 4,461.60(A/P, 10.25%, 3) = $1,802.04
99
6-26 A tractor costs $12,500 and will be used for five years when it is estimated a salvage value of $4,000 will be appropriate. Maintenance cost is estimated to be a $100 the first year and increase by $100 each year thereafter. If a 12% interest rate is used, what is the equivalent annual cost (EAC) for the tractor? Solution EAC = 12,500(A/P, 12%, 5) + 100 + 100(A/G, 12%, 5) - 4,000(A/F, 12%, 5) = $3,115.40
6-27 If Zoe won $250,000 the last week in February, 1996 and invested it by March 1, 1996 in a "sure thing" which pays 8% interest, compounded annually, what uniform annual amount can he withdraw on the first of March for 15 years starting in 2004? Solution
0 2 4 6
n = 15
98
00
02
04
$250,000
P'
P' = 250K(F/P, 8%, 7) = $428,500 A = 250K(F/P, 8%, 7)(A/P, 8%, 15) = $50,048.80
6-28 A machine, with a first cost of $20,000, is expected to save $1,500 in the first year of operation and the savings should increase by $200 every year until (and including) the ninth year, thereafter the savings will decrease by $150 until (and including) the 16th year. Using equivalent annual worth, is this machine economical? Assume a minimum attractive rate of return of 10%. Solution There are a number of equations that could be written. Here's one: EAW = -20,000(A/P, 10%, 16) + [1,500(P/A, 10%,9) + 200(P/G, 10%, 9)](A/P, 10%, 16) + [1,450(P/A, 10%, 7) - 150(P/G, 10%, 7)](P/F, 10%, 9)(A/P, 10%, 16) = -$676.79, the machine is not economical
100
6-29 Calculate the equivalent uniform annual cost of the following schedule of payments. 400 500 100
3 4 5 6
100
0 1
200
2
300
200
7
300
400
300
400 500
10
11
13
14
15
Solution Since payments repeat every five years, analyze for 5 years only. A = 100 + 100(A/G, 8%, 5) = $284.60
6-30 A college is willed $100,000 to establish a permanent scholarship. If funds are invested at 6% and all funds earned are disbursed yearly, what will be the value of the scholarship in the 6th year of operation? Solution A = Pi = 100,000(0.06) = $6,000 for any year
6-31 The UNIFORM EQUIVALENT of the cash flow diagram shown is given by which one of the following five answers? (a) (b) (c) (d) (e) 50(A/G, i, 8) 50(A/G, i, 9) 50(A/G, i, 10) 50(A/G, i, 9)(F/A, i, 9)(A/F, i, 10) 50(P/G, i, 8)(P/F, i, 1)(A/P, i, 10) A
1
A 0
A 10
2 ..
50
100
150
200
250
300
350
400
101
6-32 To get started, Econ Engineering has just borrowed $500,000 that will be paid off in 20 end-ofquarter payments. If interest is 18% compounded monthly, what will be the size of each loan payment? Solution i = 18%/12 = 1% n = 60 monthly interest periods
6-33 The cost of an automobile is $9,000 and after a period of three years it will have an estimated salvage value of $5,200. A downpayment of $1,000 will be used to purchase the car. It is desired to make the monthly payments, at 12% interest, a value such as to reduce the unpaid balance to exactly the amount of the salvage value after three years. What is the amount of the monthly payment? Solution PW of Salvage = 5,200(P/F, 1%, 36) = $3,634.28 P = 9,000 - 3,634.28 = $5,365.72 A = 5,365.72(A/P, 1%, 36) = $178.14 monthly payment
6-34 Joyce and Bill purchased a four unit apartment house and as part of the financing obtained a $100,000 loan at 9% nominal annual interest, with equal monthly payments for 20 years. What is their monthly payment?
102
Solution
n = 20 x 12 = 240 periods
6-35 The initial cost of a van is $12,800 and will have a salvage value of $5,500 after five years. Maintenance is estimated to be a uniform gradient amount of $120 per year (with no maintenance costs the first year), and the operation cost is estimated to be 36 cents/mile for 400 miles/month. If money is worth 12%, what is the equivalent annual cost (EAC) for the van, expressed as a monthly cost? Solution EAC = (12,800 - 5,500)(A/P, 12%, 5) + (5,500)(.12) + 120(A/G, 12%, 5) + .36(400)(12) = 4,626/12 = $385.50/month
6-36 An engineering student purchased a 2-year-old car that sold new for $8,000. The car depreciated 25% per year. The student made a downpayment of $1000 and obtained a 36 month loan at 15% interest, compounded monthly. What were the monthly payments? Solution year 1 depreciation = (.25)(8,000) = 2,000 year 2 depreciation = (.25)(6,000) = 1,500 if downpayment = 1,000 ; loan = 3,500 i = 15%/12 = 1% n = 12 x 3 = 36 periods 8,000 - 2,000 = $6,000 6,000 - 1,500 = $4,500 sale price
103
$200
6-38 A(n)(fill in your major) student bought a car at a used car lot on 4th street for $2,000, including tax and insurance. He was to pay for the car by making 19 equal monthly payments, with the first payment to be made when the car was delivered (a down payment). Interest on the loan was charged at the rate of 12% compounded monthly. After 11 payments (the down payment and 10 end-of-month payments) were made, a second buyer agreed to buy the car from the student and to pay a cash amount to pay off the loan in full at the time the next payment was due. If there is no pay off penalty for the early pay off, what amount will be required to pay off the loan? Solution P = A(P/A, 1%, 18) + A 2,000 = A(17.398269) A = 2,000/17.398269 = $114.95 Payoff = 114.95(P/A, 1%, 7) + 114.95 = 114.95(6.728) + 114.95 = $888.33
7-2 The heat loss through the exterior walls of a processing plant is estimated to cost the owner $3,000 next year. A salesman from Superfiber, Inc. claims he can reduce the heat loss by 80 % with the installation of $ 15,000 of Superfiber now. If the cost of heat loss rises by $200 per year, after next year (gradient), and the owner plans to keep the building ten more years, what is his rate of return, neglecting depreciation and taxes? Solution NPW = 0 at the rate of return Try 12% NPW = -15,000 + .8(3,000)(P/A, 12%, 10) + .8(200)(P/G, 12%, 10) = $1,800.64 Try 15% NPW = -$237.76 By interpolation i = 14.7%
105
106
7-3 Does the following project have a positive or negative rate of return? Show how this is known to be true. Investment cost Net benefits Salvage Useful life Solution Year 1 2 3 4 5 Benefits 300 500 700 900 50 Total = 2,450 < Cost $2,500 $300 in Year 1, increasing by $200/year $50 4 years
Total Benefits obtained are less than the investment, so the "return" on the investment is negative
7-4 At what interest rate would $1000 at the end of 1995 be equivalent to $2000 at the end of 2002? Solution (1 + i)7 = 2 ; i = (2)1/7 - 1 = 0.1041 or 10.41%
7-5 A painting, purchased one month ago for $1000, has just been sold for $1700. What nominal annual rate of return did the owner receive on his investment? Solution i = 7,000/1,000 = 70% IRR = 70 x 12 = 840%
7-8 Find the rate of return for a $10,000 investment that will pay $1000/year for 20 years. Solution 10,000 = 1,000(P/A, i%, 20) (P/A, i%, 20) = 10 From tables: 7% < i < 8% $ interpolate i = 7.77%
107
7-9 A young engineer has a mortgage loan at a 15% interest rate, which he got some time ago, for a total of $52,000. He has to pay 120 more monthly payments of $620.72 each. As interest rates are going down, he inquires about the conditions under which he could refinance the loan. If the bank charges a new loan fee of 4% of the amount to be financed, and if the bank and the engineer agree on paying this fee by borrowing the additional 4% under the same terms as the new loan, what percentage rate would make the new loan attractive, if the conditions require him to repay it in the same 120 payments? Solution The amount to be refinanced: a) PW of 120 monthly payments left P = A(P/A, i, n) = 620.72(P/A, 15%/12, 120) = $38,474.09
b) New loan fee (4 %) 38,474.09 x 0.04 = $1,538.96 ! Total amount to refinance = 38,474.09 + 1,538.96 = 40,013.05 The new monthly payments are: ANEW = 40,013.05 (A/P, i, 120) The current payments are: AOLD = 620.72 We want ANEW < AOLD Substituting ! 40,013.05(A/P, i, 120) < 620.72 (A/P, i, 120) < 620.72/40,013.05 = 0.0155 for i = 1% for i = 1% (A/P, 1%, 120) = 0.0143 (A/P, 1%, 120) = 0.0161
1% < i < 1% $ interpolate i = 1.1667 % This corresponds to a nominal annual percentage rate of 12 x 0.011667 = 14 % Therefore, she has to wait until interest rates are less than 14%.
108
7-10 Your company has been presented with an opportunity to invest in a project. The facts on the project are presented below: Investment required $60,000,000 Annual Gross income 20,000,000 Annual Operating costs: Labor 2,500,000 Materials, licenses, insurance, etc. 1,000,000 Fuel and other costs 1,500,000 Maintenance costs 500,000 Salvage value after 10 years 0 The project is expected to operate as shown for ten years. If your management expects to make 25% on its investments before taxes, would you recommend this project? Solution Net income = 20M (2.5M + 1M + 1.5M + .5M) = $14,500,000 NPW = 0 at the rate of return 0 = - 60,000,000 + 14,500,000(P/A, i, 10) (P/A, i, 10) = 60/14.5 = 4.138 @20% @25% P/A = 4.192 P/A = 3.571
20% < i < 25% $ interpolate i = 20.43% IRR < 25 % $ Reject Project
7-11 Consider the following investment in a piece of land. Purchase price $10,000 Annual maintenance: 100 Expected sale price after 5 years: $20,000 Determine: (a) A trial value for i (b) The rate of return ( to 1/100 percent) (c) What is the lowest sale price the investor should accept if she wishes to earn a return of 10% after keeping the land for 10 years?
109
7-12 Calculate the rate of return of the following cash flow with accuracy to the nearest 1/10 percent. A = $1,000
$3,100 Solution NPW = 0 at the rate of return 0 = -3,100 + 1,000(P/A, i, 5) (P/A, i, 5) = 3.1 (P/A, 20%, 5) = 2.991 (P/A, 18%, 5) = 3.127 18% < i < 20% $ interpolate i = 18.4%
110
7-13 An investment that cost $1000 is sold five years later for $1261. What is the nominal rate of return on the investment (interest rate)? Solution F = P (F/P, i %, 5) 1,261 = 1,000 (F/P, i %, 5) (F/P, i %, 5) = 1,261/1,000 = 1.2610 From tables: (F/P, 4%, 5) = 1.246 (F/P, 5%, 5) = 1.276 4% < i < 5% $ interpolate i = 4.75%
7-14 Elizabeth made an initial investment of $5000 in a trading account with a stock brokerage house. After a period of 17 months the value of the account had increased to $8400. What is the nominal annual interest rate realized on the initial investment if it is assumed there were no additions or withdrawals from the account? Solution F = P(F/P, i, 17) F/P = 8,400/5,000 = 1.68 (1 + i)17 = 1.68 1 + i = (1.68)1/17 = 1.031 Annual interest rate = 3.1 x 12 = 37.2%
7-15 You borrowed $25 from a friend and after five months repaid $27. What interest rate did you pay (annual basis)?
111
7-16 You have a choice of $3,000 now, or $250 now with $150 a month for two years. What interest rate will make these choices comparable? Solution 3,000 = 250 + 150 (P/A, i, 24) P/A = 18.33 @ 2% @ 2% P/A = 18.914 P/A = 17.885
7-17 Sain and Lewis Investment Management Inc., is considering the purchase of a number of bonds to be issued by Southeast Airlines. The bonds have a face value of $10,000 and a face rate of 7.5% payable annually. The bonds will mature ten years after they are issued. The issue price is expected to be $8,750. Determine the yield to maturity (IRR) for the bonds. If SLIM Inc. requires at least a 10% return on all investments should they invest in the bonds? Solution
Year 0 1-10 10
112
7-18 A 9.25% coupon bond issued by Gurley Gears LLC is purchased January 1, 2002 and matures December 31, 2010. The purchase price is $1079 and interest is paid semi-annually. If the face value of the bond is $1,000, determine the effective internal rate of return. Solution n = 9 x 2 = 18 year periods Year 0 1-18 18 IRR = (1 + .04)2 1 = 8.16% 4% -1,079.00 +585.48 +493.60 $.08
First Cost Interest 46.25(P/A, i%, 18) Maturity 1000(P/F, i%, 18) NPW =
7-19 Joes Billiards Inc. stock can be purchased for $14.26 per share. If dividends are paid each quarter at a rate of $0.16 per share, determine the effective i if after 4 years the stock is sold for $21.36 per share. Year 0 1-16 16 3% -14.26 +2.01 +13.31 $1.06 3 -14.26 +1.93 +12.32 -$.01
Cost Dividends .16(P/A, i%, 16) Sale 21.36(P/F, i%, 16) NPW =
7-20 A bond with a face value of $1,500 can be purchased for $800. The bond will mature five years from now and the bond dividend rate is 12%. If dividends are paid every three months, what effective interest rate would an investor receive if she purchases the bond?
113
A = $45
$1,500 + $45
NPW = 0 at IRR 0 = -800 + 45(P/A, i%, 20) + 1,500(P/F, i%, 20) By trial and error: Try 7% Try 8% NPW = +$64.33 NPW = -$36.44
7% < i < 8% $ interpolate i = 7.64 % Effective interest rate = (1+ i) -1 = (1+0.07638) - 1 = 0.3423 = 34.23%
m 4
7-21 How many positive rates of return does the following cash flow have? Year 0 1 2 3 4 5 6 7 Cash Flow -$50,000 +25,000 +25,000 0 -50,000 +25,000 +25,000 +25,000
114
Solution
Year 0 1 2 3 4 5 6 7
3 sign changes in cash flow Cash Flow Rule of Signs: There may be as many as 3 positive ROR Accumulated Cash Flow: A 7 % 0, one sign change in accumulated cash flow, $one positive rate of return
Note: also check to see if there is any external investments during project life.
2,950,000 $2
= 2.33
8-2 A proposed bridge on the interstate highway system is being considered at the cost of $2 million. It is expected that the bridge will last 20 years. Operation and maintenance costs are estimated to be $180,000 per year. The federal and state governments will pay these construction costs. Benefits to the public are estimated to be $900,000 per year. The building of the bridge will result in an estimated cost of $250,000 per year the general public. The project requires a 10% return. Determine the B/C ratio for the project. State any assumption made about benefits or costs.
115
116
Solution
$250,000 cost to general public is disbenefit. AWBENEFITS = 900,000 - 250,000 = $650,000 AWCOSTS = 2,000,000(A/P, 10%, 20) + 180,000 = $415,000 B/C =
AWBENEFITS 650,000 ! AWCOSTS 415,000
= 1.57
8-3 The town of Podunk is considering the building of a new downtown parking lot. The land will cost $25,000 and the construction cost of the lot is estimated to be $150,000. Each year costs associated with the lot are estimated to be $17,500. The income from the lot is estimated to be $18,000 the first year and increase by $3,500 each year for the twelve year expected life of the garage. Determine the B/C ratio if Podunk uses a cost of capital of 4%. Solution PWBENEFITS = 18,000(P/A, 4%, 12) + 3500(P/G, 4%, 12) = $334,298 PWCOSTS = 175,000 + 17,500(P/A, 4%, 12) = 339,238 B/C =
PWBENEFITS 334,298 ! PWCOSTS 339,238
= 0.99
8-4 Flyin Ryans Tire Sales is considering the purchase of new tire balancing equipment. The machine will cost $12,699 and have an annual savings of $1,500 with a salvage value at the end of 12 yrs. of $250. If MARR is 6%, use B/C analysis to determine whether or not Ryan should purchase the machine. Solution PWBENEFITS = $1500(P/A, 6%, 12) + $250(P/F, 6%, 12) = $12,700.25 PWCOSTS = $12,699 B/C =
PWBENEFITS 12,700 ! PWCOSTS 12,699
= 1.00
117
8-5 Sim City wants to build a new bypass between two major roads that will cut travel time for commuters. The road will cost $2,000,000 and save 7,500 people $100/yr in gas. The road will need to be resurfaced every year at a cost of $7,500. The road is expected to be in use for 20 years. Determine if Sim City should build the road using B/C analysis. MARR is 8%. Solution PW of Costs 2,000,000 + 7,500(P/A, 8, 20) = $2,073,635 PW of Benefits (7,500)($100)(P/A, 8, 20) = $7,363,500 B/C = 7,363,500/2,073,635 = 3.551 Conclusion: Yes, Sim City should build the bypass
FUTURE WORTH
8-6 A mortgage of $50,000 for 30 years, with monthly payments at 10% interest is contemplated. At the last moment you receive news of a $25,000 gift from you parents to be applied to the principal. Leaving the monthly payments the same, what amount of time will now be required to pay off the mortgage and what is the amount of the last payment (assume any residual partial payment amount is added to the last payment)? Solution i = 6%/12 = % per month A = 50,000(A/P, %, 360) = $299.77 monthly payment After reduction of P to 25,000 25,000 = 299.77(P/A, i%, n) (P/A, i%, n) = 83.40 Try n = 104 periods; P/A = 80.942 Try n = 120 periods: P/A = 90.074 Interpolate, n = 108.31 periods = 9.03 years At 9 years (108 periods): P = 299.77 (P/A, %, 108) = 299.77 (83.2934) = 24,968.87 n = 30 x 12 = 360 periods
1 P/A
118
Residual = 25,000 - 24,968.87 = $31.13 Last Payment = Value of residual at time of last payment + last payment = 31.13(F/P, %, 108) + 299.77 = $353.12
8-7 A woman deposited $10,000 into an account at her credit union. The money was left on deposit for 10 years. During the first five years the woman earned 9% interest (nominal), compounded monthly. The credit union then changed its interest policy so that the second five years the woman earned 6% interest (nominal), compounded quarterly. (a) How much money was in the account at the end of the 10 years? (b) Calculate the rate of return that the woman received.
Solution (a) at the end of 5 years: F = 10,000 (F/P, %, 60)* = $15,660.00 at the end of 10 years: F = 15,660(F/P, %. 20)** = $17,304.30 (b) 10,000(F/P, i, 10) = 17,304.30 (F/P, i, 10) = 1.7304 try i = 5% try i = 6% (F/P, i, 10) = 1.629 (F/P, i, 10) = 1.791
8-8 A woman deposited $100 per month in her savings account for 24 months at 6% interest, compounded monthly. Then for five years she made no deposits or withdrawals. How much is the account worth after seven years?
119
8-9 On January 1st a sum of $1,200 is deposited into a bank account that pays 12% interest, compounded monthly. On the first day of each succeeding month $100 less is deposited (so $1,100 is deposited February 1st, $1,000 on March 1st, and so on). What is the account balance immediately after the December 1st deposit is made? Solution P = 1,200(P/A, 1%, 12) - 100 (P/G, 1%, 12) = 7,449.1 F = 7,449.10(F/P, 1%, 12) = $8,395.14
8-10 A 25-year-old engineer named Newt begins working for a salary of $50,000 per year when he graduates from college. From his first monthly paycheck, he notices 7% of his salary is deducted and paid into Social Security, and his employer pays a like amount. In effect, Newt finds that 14% of his salary is being taken by the government for this mandatory program. Assuming that Newt's contribution to Social Security is 14% of $50,000 = $7,000 per year, and that Newt works for the same salary until he is 65, how much will he have effectively contributed into the Social Security program. Assume a 5% interest rate. Solution F = 7,000(F/A, 5%, 40) = $845,593.00
8-11 How much money would be in an account if $1,000 is deposited in a bank at 4% interest, compounded semiannually, for 3 years? Solution i = 4%/2 = 2% per year; F = $1,000 (F/A, 2%, 6) = $1126 n = 2 x 3 = 6 periods
120
8-12 Harry was a big winner in the New Hampshire sweepstakes. After paying the income taxes he had $80,000 left to invest in an investment fund that will pay 10% interest for the next 20 years. How much money will Harry receive at the end of the 20 years? Solution F = 80,000(F/P, 10%, 20) = $538,160.00
8-13 David has received $20,000 and wants to invest it for 12 years. There are three plans available to him. (a) A Savings Account. It pays 3% per year, compounded daily. (b) A Money Market Certificate. It pays 6% per year, compounded semiannually. (c) An Investment Account. Based on past experience it is likely to pay 8% per year. If David does not withdraw the interest, how much will be in each of the three investment plans at the end of 12 years? Solution a) F = P(1 + i)n
r' $ i eff ! & 1 # ) % m(
m 365
* 1 ! 3.82%
Choose plan C since this plan yields the highest return at the end of 12 years.
8-14 How long will it take for $300 to triple at a 5% per year interest rate?
121
8-15 An annuity is established by the payment of $150 per month for eight years with interest to be calculated at 7%. The company retains these funds in an account from which they propose to pay you $1,530 per year for life (an actuarial period of 18 years for your age). If interest is assumed to continue at 7%, what is the lump-sum profit to the company at the end of the pay-in period? Assume monthly compounding for the payment period. Solution FW = 150(F/A,
12
* 1 ! 7.76%
PW of payment = 1,530(P/A, 7.76%, 18) = 1,530(9.530) = $14,581 Profit at end of pay-in period = 19,649 - 14,581 = $5,068
8-16 A person would like to retire 10 years from now. He currently has $32,000 in savings, and he plans to deposit $300 per month, starting next month, in a special retirement plan. The $32,000 is earning 8% interest, while the monthly deposits will pay him 6% nominal annual interest. Once he retires, he will collect the two sums of money, and being conservative in his calculations, he expects to get a 4% annual interest rate after year 10. Assuming he will only spend the interest he earns, how much will he collect in annual interest, starting in year 11?
122
Solution Savings:
F = 300(F/A,
FT = 69,086 + 49,164 = $118,250 The interest to collect per year = 118,250 x 0.04 = 4,730
8-17 Mary wants to accumulate a sum of $20,000 over a period of 10 years to use as a downpayment for a house. She has found a bank that pays 6% interest compounded monthly. How much must she deposit four times a year (once each three months) to accumulate the $20,000 in ten years? Solution In this problem Mary's deposits do not match the interest period. One solution is to compute what her monthly deposit would need to be, and then the equivalent deposit each 3 months. Monthly deposit (A) = F(A/F, i %, n) - 20,000(A/F, %, 120) = 20,000 (0.00610) = $122 Equivalent deposit at end of each 3-months: = A(F/A, %, 3) = 122(3.015) = $368 Note that the compound interest table only provided two digit accuracy. Hand Calculator solution:
. 1 0.005 3 ! 122.041 monthly deposit (A) = 20,0000 120 0 / ,1 # 0.005- * 13 2
8-18 If $5,000 is deposited into a savings account that pays 8% interest, compounded quarterly, what will the balance be after 6 years? What is the effective interest rate? Solution F = 5,000(F/P, 2%, 24) = $8,042.19
123
8-19 Starting now, deposits of $100 are made each year into a savings account paying 6% compounded quarterly. What will be the balance immediately after the deposit made 30 years from now? Solution F = 100 (A/F, 1%,4) (F/A, 1%, 120) + 100 (F/P, 1%, 120) = $8,693.58
8-20 You invest $1,000 in a bank at 4% nominal interest. Interest is continuously compounded. What will your investment be worth (a) at the end of 1 year? (b) at the end of 36 months? Solution (a) F = Pe = 1,000e (b) F = Pe = 1,000e
rn rn (0.04)(1) (0.04)(3)
= $1,040.81 = $1,127.50
8-21 Using the tables for Uniform Gradients, solve for the Future Value at the end of year 7. $500 $400 $300 $200 $100
0 1 2 3 4 5 6 7
$100 $100 Solution PV = 100(P/G, 10%, 7) - 100(P/A, 10%, 7) - 100 = $689.50 FV = 689.5(F/P, 10%, 7) = $1,343.84
124
8-22 If Jane invests $125,000 in a fund that will pay 8%, compounded quarterly, how much will she have in the fund at the end of 10 years? What effective interest rate is she earning on her investment? Solution i = 8%/4 = 2% per quarter n = 10 x 4 = 40 periods
8-23 As a tax shelter, Joan's father has set up a trust fund into which he puts $1000 every year on Joan's birthday beginning on her first birthday. How much will be in the fund on Joan's 21st birthday if the account pays 6% compounded quarterly? Solution i = 6%/4 = 1 per quarter A = 1,000(A/F, 1%, 4) = $244.40 F = 244.40(F/A, 1%, 84) = $40,613.
PAYBACK PERIOD
8-24 What is the major advantage of using payback period to compare alternatives? Solution Payback period is easy and rapid. Also, it provides information on investment recovery time that may be important for companies with cash flow problems.
8-25 For calculating payback period, when is the following formula valid?
Payback period ! First Cost Annual Benefits
125
Valid when: a) There is a single first cost at time zero. b) Annual Benefits = Net annual benefits after subtracting any annual costs c) Net Annual Benefits are uniform
8-26 Is the following statement True or False? If two investors are considering the same project, the payback period will be longer for the investor with the higher minimum attractive rate of return (MARR). Solution Since payback period is generally the time to recover the investment, and ignores the MARR, it will be the same for both investors. The statement is False.
8-27 What is the payback period for a project with the following characteristics, if the minimum attractive rate of return (MARR) is 10%? First Cost Annual Benefits Annual Maintenance Salvage Value Useful Life Solution Payback occurs when the sum of net annual benefits is equal to the first cost. Time value of money is ignored. Year Benefits 1 8,000 2 8,000 2,500 = 2,000 = 5,500 11,500 Costs = 6,000 6,000 Net Benefits Total Net Benefits $20,000 8,000 2,000 in year 1, then increasing by $500/year 2,000 10 years
126
3
8-28 Determine the payback period (to the nearest year) for the following project. First Cost Annual Maintenance Annual Income Salvage Value Useful Life MARR Solution Year 1 2 3 4 5 Net Income 2,500 2,300 2,100 1.900 1,700 Sum 2,500 4,800 6,900 8,800 10,500 > 10,000 $10,000 500 in year 1, increasing by $200/year 3,000 4,000 10 years 10%
8-29 Determine the payback period (to the nearest year) for the following project: Investment cost Annual maintenance costs Annual benefits Overhaul costs $22,000 1,000 6,000 7,000 every 4 years
127
Solution
BREAKEVEN
8-30 A machine that produces a certain piece must be turned off by the operator after each piece is completed. The machine "coasts" for 15 seconds after it is turned off, thus preventing the operator from removing the piece quickly before producing the next piece. An engineer has suggested installing a brake that would reduce the coasting time to 3 seconds. The machine produces 50,000 pieces a year. The time to produce one piece is 1 minute 45 seconds, excluding coasting time. The operator earns $8.00 an hour and other direct costs for operating the machine are $4.00 an hour. The brake will require servicing every 500 hours of operation. It will take the operator 30 minutes to perform the necessary maintenance and will require $44.00 in parts and material. The brake is expected to last 7500 hours of operation (with
128
proper maintenance) and will have no salvage value. How much could be spent for the brake it the minimum attractive rate of return is 10% compounded annually? Solution
$ 2' Annual cost w/o the brake = 50,000 & ) (12) = $20,000 % 60 (
, -
7,500
5,000(1.8) 60
! 5 years
8-31 A road can be paved with either asphalt or concrete. Concrete costs $15,000/km and lasts 20 years. What is the maximum that should be spent on asphalt if it only lasts 10 years? Annual maintenance costs for both pavements are $500/km. MARR = 12%. Solution Since maintenance is the same for both, it doesn't affect the answer. However, there is nothing wrong with including it. 15,000(A/P, 12%, 20) = PASPHALT(A/P, 12%, 10) 15,000(.1339) = PASPHALT(.1770) PASPHALT = $11,347 8-32 A proposed building may be roofed in either galvanized steel sheet or composition roofing. The composition roof costs $20,000 and must be replaced every 5 years at the same cost. The steel roof costs $28,000 but the useful life is unknown. Neither roof has any salvage value and no maintenance is needed. If the minimum attractive rate of return (MARR) equals 15%, what is the minimum life that the steel roof must have to make it the better alternative? (Report to the nearest whole year; don't bother interpolating.)
129
8-33 What is the breakeven capital cost for Project B compared to Project A if interest equals 10%? Year 0 1-5 Solution NPW of A = - 1,000 + 300 (P/A, 10%, 5) = 137.3 NPW of B = NPW of A 137.3 = PB + 200(P/A, 10, 5) PB = -$620.90 8-34 What is the smallest acceptable annual income from a project that has a $70,000 investment cost and a $70,000 salvage value if the life is 15 years and the minimum attractive rate of return (MARR) is 20%? Solution Income = 70,000(A/P, 20%, 15) - 70,000(A/F, 20%, 15) = 70,000[(A/P, 20%, 15) (A/F, 20%, 15)] = $14,000 A -1,000 +300/year B ? +200/year
8-35 A car rental agency has a contract with a garage to have them do major repairs (specified in the contract) for $450/car every six months. The car rental agency estimates that for $150,000, amortized at 8% interest for 20 years, and a salvage value of $60,000, they could have their own facility. They estimate that they could take care of their own car repairs in this facility at a cost of $200/car every six months. Ignoring taxes and other economic factors, what is the minimum
130
Solution
Let N = number of autos needed 450N = (150,000 - 60,000)(A/P, 4%, 40) + 60,000(.04) + 200N 450N = 90,000 x .0505 + 2,400 + 200N 250N = 6,945 N = 27.78 or 28 autos needed
8-36 Assume you need to buy some new automobile tires and you are considering purchasing either the "Econo-Ride", which costs $33.95 per tire, or the "Road King", which costs $65.50. Both tires are alike except that the "Road King" is more durable and will last longer. Regardless of which tire is purchased, balancing and installation costs are $1.50 per tire. The salesman says the "EconoRide" will last 20,000 miles. Assume a minimum attractive rate of return (MARR) of 6% and that you drive 10,000 miles per year. (a) How many miles would the "Road King" have to last to make you indifferent in your choice? (b) The salesman says the "Road King" will be on sale next week. If he also says the tire will last 30,000 miles, what would the sale price have to be to make you indifferent in your choice? Solution (a) 4(1.5 + 33.95)(A/P, 6%,
20,000 10,000
(A/P, 6%, N) = .28859 From tables N = 4, or 40,000 miles (b) 4(1.5 + 33.95)(A/P, 6%,
20,000 10,000
30,000 10,000
8-37 A soft drink company has researched the possibility of marketing a new low-calorie beverage, in a study region. The expected profits depend largely on the sales volume, and there is some uncertainty as to the precision of the sales-forecast figures. The estimated investment is $173,000 while the anticipated profits are $49,500 per year for the next 6 years. If the company's MARR = 15%, is the decision to invest sensitive to the uncertainty of the sales forecast, if it is estimated that in the worst case the profits will be reduced to $40,000 per year? What is the minimum volume of sales for the project to breakeven, if there is a profit of $6.70 per unit volume? Solution
131
b) For an annual profit of $40,000 NPW = 40,000 (P/A, 15%, 6) - 173,000 = -21,640 (not attractive) Therefore the decision is sensitive to the expected variations in sales or profits.
The breakeven: NPW = 0 NPW = 0 = X(P/A, 15%, 6) - 173,000 where X = min $ profit
X!
173,000 ( P/A, 15%, n )
! $45,718.80
$45,718.80 $6.70/ unit
in volume units =
8-38 A machine, costing $2,000 to buy and $300 per year to operate, will save labor expenses of $650 per year for 8 years. If the interest rate is 10%, what is the minimum salvage value (after 8 years) at which the machine is worth purchasing? Solution NPW 0 = - 2,000 + 350(P/A, 10%, 8) + S(P/F, 10%, 8) 0 = - 132.75 + .4665S S = $284.57
8-39 The PARC Company can purchase gizmoes to be used in building whatsits for $90 each. PARC can manufacture their own gizmoes for $7,000 per year overhead cost plus $25 direct cost for each gizmoe, provided they purchase a gizmoe maker for $100,000. PARC expects to make whatsits using gizmoes for 10 years. The gizmoe maker should have a salvage value of $20,000 after 10 years. PARC uses 12% as its minimum attractive risk rate. At what annual production rate should PARC make their own gizmoes? Solution Equivalent Cost Solution: EACBUY = $90N where N = annual quantity EACMAKE = 100,000(A/P, 12%, 10) + 7,000 + 25N -20,000(A/F, 12%,10)
132
For breakeven: EACBUY = EACMAKE 90N = 23,560 + 25N N = 362.5 This indicates they should be bought at 362/year or less and made at 363/year or more.
PWFIRST COST
PWCF = (4)(15,000)(P/A, 6%, 5) - 1000(P/A, 6%, 5) - [10,000(P/A, 6%, 4)+1,000(P/G, 6%, 4)] + 10,000(P/F, 6%, 5) = $216,386 PWFC = $200,000 PWI =
216,386 = 1.08 200,000
PWI > 1
8-41 A construction co. wants to purchase a new skid steer. The first cost is $30,000. The yearly savings is $10,800. Operating and maintenance is $3,600 the first year and increases $500 per year. The salvage value is expected to be $1,000. The estimated life is 10 years. The companys MARR is 7%. Find the present worth index.
133
PWCF = 10,800(P/A, 7%, 10) - [3,600(P/A, 7%, 10) + 500(P/G, 7%, 10)] - 1,000(P/F, 7%, 10) = $37,223 PWFC = $30,00 PWI =
PWCF 37,223 = = 1.24 30,000 PWFC
8-42 Taylor Technologies is considering buying new circuit analyzing software. The software license sells for $12,500 for a five-year usage period. The software speeds up the circuit design process and is estimated to yield savings of $6,000 per year. Upgrades and updates will cost the company $2,000 annually. Taylor uses the present worth index to make all purchasing decisions. The companys MARR is 7%. Should Taylor Tech. purchase the software? Solution PWCF = 6,000(P/A, 7%, 5) - 2,000(P/A, 7%, 5) = $16,400 PWFC = $12,500 PWI =
PWCF 16,400 = = 1.31 12,500 PWFC
9-2 Three purchase plans are available for a new car. Plan A: $5,000 cash immediately Plan B: $1,500 down and 36 monthly payments of $116.25 Plan C: $1,000 down and 48 monthly payments of $120.50 If a customer expects to keep the car five years and her minimum attractive rate of return (MARR) is 18% compounded monthly, which payment plan should she choose?
135
136
Solution
Note that in all cases the car is kept 5 years that is the common analysis period. Therefore PWC is the easiest method. i = 18%/12 = 1% PWCA = $5,000 PWCB = 1,500 + 116.25(P/A, 1%, 36) = $4,715,59 PWCC = 1,000 + 120.50(P/A, 1%, 48) = $5,102.18 Therefore Plan B is best
9-3 Given the following three mutually exclusive alternatives. Alternative A Initial Cost Annual Benefits Useful Life(years) What alternative is preferable if i = 10%? Solution Using simplest method (NPW): A Initial Cost Annual Benefits Useful Life(years) Present Worth Benefits Present Worth Costs Net Present Worth = PWB - PWC Choose C 50.00 15.00 5 56.87 50.00 6.87 B 30.00 10.00 5 37.91 30.00 7.91 C 40.00 12.00 5 45.49 40.00 5.49 $50 15 5 B $30 10 5 C $40 12 5
137
If this were your money, and you want to earn at least 12% interest on your money, which investment would you make, if any? What nominal interest rate do you earn on the investment you choose? Solve by present worth analysis. Solution PW Analysis i = 12%/12 = 1% per month Alternative 1: Alternative 2: NPW = 110(P/A, 1%, 10) - 1,000 = $41.81 NPW = 130(P/A, 1%, 10) - 1,200 = $31.23
Choose Alternative 1 Maximum NPW Nominal Interest: NPW = 0 = -1,000 + 110(P/A, i%, 10) (P/A, i%, 10) = 9.1 From tables: i 1.75%
9-5 A farmer has just purchased a tractor for which he had to borrow $20,000. The bank has offered the following choice of payment plans determined using an interest rate of 8%. If the farmer's minimum attractive rate of return (MARR) is 15%, which plan should he choose? Plan A: $5,010 per year for 5 years Plan B: $2,956 per year for 4 years plus $15,000 at end of 5 years Plan C: Nothing for 2 years, then $9048 per year for 3 years Solution PWCA = 5,010(P/A, 15%, 5) = $16,794 PWCB = 2,956(P/A, 15%, 4) + 15,000(P/F, 15%, 5) = $15,897. PWCC = 9,048(P/A, 15%, 3)(P/F, 15%, 2) = $15,618 Plan C is lowest cost plan
138
9-6 Projects A and B have first costs of $5,000 and $9,000, respectively. Project A has net annual benefits of $2,500 during each year of its 5 year useful life, after which it can be replaced identically. Project B has net annual benefits of $3,300 during each year of its 10 year life. Use present worth analysis, an interest rate of 30% per year, and a 10 year analysis period to determine which project to select. Solution PWA PWB = -5,000[1 +(P/F, 30%, 5)] + 2,500(P/A, 30%, 10) = $1,382.20 = - 9,000 + 3,300(P/A, 30%, 10) = $ 1,202.08
9-7 The lining of a chemical tank in a certain manufacturing operation is replaced every 5 years at a cost of $5,000. A new type lining is now available which would last 10 years, but costs $13,000. The tank needs a new lining now and you intend to use the tank for 40 years, replacing linings when necessary. Compute the present worth of costs of 40 years of service for the 5-year and 10year linings if i = 10%. Solution PW 5 yr Lining: PW = [5,000(A/P, 10%, 5)](P/A, 10%, 40) = $12,898.50 PW 10 yr Lining: PW = [13,000(A/P, 10%, 10)](PA, 10%, 40) = $20,683.50
9-8 A manufacturing firm has a before-tax minimum attractive rate of return (MARR) of 12% on new investments. What uniform annual benefit would Investment B have to generate to make it preferable to Investment A? Year 0 1-6 Investment A - $60,000 +15,000 Investment B - $45,000 ?
139
9-9 The city council wants the municipal engineer to evaluate three alternatives for supplementing the city water supply. The first alternative is to continue deep well pumping at an annual cost of $10,500. The second alternative is to install an 18" pipeline from a surface reservoir. First cost is $25,000 and annual pumping cost is $7000. The third alternative is to install a 24" pipeline from the reservoir at a first cost of $34,000 and annual pumping cost of $5000. Life of all alternatives is 20 years. For the second and third alternatives, salvage value is 10% of first cost. With interest at 8%, which alternative should the engineer recommend? Use present worth analysis. Solution Fixed output, therefore minimize cost. Year 0 1-20 20 Deepwell: -10,500 DEEPWELL 18" PIPELINE -25,000 -7,000 +2,500 PWC = - 10,500(P/A, 8%, 20) -$103,089 24" PIPELINE -34,000 -5,000 +3,400
18" Pipeline: PW of Cost = -25,000 - 7,000(P/A, 8%, 20) + 2,500(P/F, 8%, 20) = -$493,190 24" Pipeline: PW of Cost = -34,000 - 5,000(P/A, 8%, 20) + 3,400(P/F, 8%, 20) = -$82,361 Choose 24" Pipeline
9-10 A magazine subscription is $12 annually, or $28 for a 3-year subscription. If the value of money is 12%, which choice is best?
140
Solution
28 <?=?> 12 + 12(P/A, 12%, 2) = 12 + 12(1.69) = 32.28 Choose 3 yr subscription because 28 < 32.28
9-11 Two alternatives are being considered for recovering aluminum from garbage. The first has a capital cost of $100,000, a first year maintenance cost of $5000, with maintenance increasing by $1500 per year for each year after the first. The second has a capital cost of $120,000, a first year maintenance cost of $3000, with maintenance increasing by $1000 per year after the first. Revenues from the sale of aluminum are $20,000 in the first year, increasing $2000 per year for each year after the first. Life of both alternatives is 10 years. There is no salvage value. The before-tax Minimum Attractive Rate of Return is 10%. Using Present Worth Analysis determine which alternative is preferred. Solution Alternative 1: NPW = -100,000 +(20,000 - 5,000)(P/A, 10%, 10) = $3,620.50 Alternative 2: NPW = -120,000 +(20,000 - 3,000)(P/A, 10%, 10) = $7,356.00 Choose Alternative 2 Maximum. NPW 9-12 A brewing company is deciding between two used filling machines as a temporary measure, before a plant expansion is approved and completed. The two machines are: (a) The Kram Filler. Its initial cost is $85,000, and the estimated annual maintenance is $8000. (b) The Zanni Filler. The purchase price is $42,000, with annual maintenance costs of $8000. The Kram filler has a higher efficiency, compared with the Zanni, and it is expected that the savings will amount to $4000 per year if the Kram filler is installed. It is anticipated that the filler machine will not be needed after 5 years, and at that time, the salvage value for the Kram filler would be $25,000, while the Zanni would have little or no value. Assuming a minimum attractive rate of return (MARR) of 10%, which filling machine should be purchased?
141
9-13 Two technologies are currently available for the manufacture of an important and expensive food and drug additive. The two can be described as follows: Laboratory A. Is willing to release the exclusive right to manufacture the additive in this country for $50,000 payable immediately, and a $40,000 payment each year for the next 10 years. The production costs are $1.23 per unit of product. Laboratory B. This laboratory is also willing to release similar manufacturing rights. They are asking for the following schedule of payments: On the closing of the contract, $10,000. From years 1 to 5, at the end of each year, a payment of $25,000 each. From years 6 to 10, also at the end of each year, a payment of $20,000. The production costs are $1.37 per unit of product. Neither lab is to receive any money after 10 years for this contract. It is anticipated there will be an annual production of 100,000 items for the next 10 years. On the basis of analyses and trials, the products of A and B are practically identical in quality. Assuming a MARR of 12%, which lab should be chosen? Solution Laboratory A: The annual production cost = 1.23 x 100K = $123K
142
Laboratory B:
PWC = 10,000 + [25,000 + 137,000](P/A, 12%, 5) + [20,000 + 137,000](P/A, 12%, 5)(P/F, 12%, 5) = $915,150 Therefore choose Laboratory B.
9-14 An engineering analysis by net present worth (NPW) is to be made for the purchase of two devices A and B. If an 8% interest rate is used, recommend the device to be purchased. Uniform Cost Device A Device B Solution Device A: NPW = 100(P/A, 8%, 10) + 250((P/F, 8%, 10) - 600 - [600 - 250](P/F, 8%, 5) = -$51.41 Device B: NPW = 100(P/A, 8%, 10) + 180(P/F, 8%, 10) - 700 = $54.38 Select device B $600 700 Annual Benefit $100 100 Salvage $250 180 Useful Life 5 years 10 years
9-15 A company decides it must provide repair service for the equipment it sells. Based on the following, which alternative for providing repair service should be selected? Alternative A B C Solution None of the alternatives look desirable, but since one of the alternatives must be chosen (the do nothing alternative is not available), choose the one that maximizes NPW. Thus the best of the three alternatives is B. Net Present Worth -$15,725 -6,657 -8,945
143
9-16 A firm is considering the purchase of a new machine to increase the output of an existing production process. Of all the machines considered, the management has narrowed the field to the machines represented by the cash flows shown as follows: Machine 5 2 4 1 3 Initial Investment $50,000 60,000 75,000 80,000 100,000 Annual Operating Cost $22,500 20,540 17,082 15,425 11,374
If each of these machines provides the same service for 3 years and the minimum attractive rate of return is 12%, which machine should be selected? Solution Minimize the PW of Cost: Machine 5 2 4 1 3 Select machine 5 Initial Investment -50,000 -60,000 -75,000 -80,000 -100,000 Operating Costs x (P/A, 12%, 3) + 22,500 (2.402) + 20,540(2.402) + 17,082(2.402) + 15,425(2.402) + 11,374(2.402) PW of Costs = 104,045 = 109,337 = 116,031 = 117,051 = 127,320
9-17 Data for tractors A and B are listed below. With interest of 12%, which tractor would be selected based on equivalent annual cost(EAC)? A First cost Annual maintenance Salvage value Useful life $30,000 1,500 5,000 6 years B $36,000 2,000 8,000 6 years
144
Solution
S = Salvage value A = Annual maintenance P = First cost i = 12 % n=6 EAC = P(A/P, i %, n) - S(A/F, i %, n) + Other Costs TRACTOR A: EAC = 30,000(A/P, 12%, 6) - 5,000(A/F, 12%, 6) + 1,500 = $8,180 check: EAC = (P - S)(A/P, i %, n) + Si + Other Costs = $8,180 TRACTOR B: EAC = 36,000(A/P, 12%, 6) - 8,000(A/F, 12%, 6) + 2,000 = $9,770 check: EAC = (P - S)(A/P, 12%, 6) + S(.12) + Other Costs = $9,770 Since criteria is to minimize EAC select tractor A
9-18 According to the manufacturers' literature, the costs of running automatic grape peelers, if maintained according to the instruction manuals, are: Manufacturer: First Cost Maintenance Slippery $500 $100 at end of years 2, 4, 6 and 8 Grater $300 Year 1 - $ 0 2350 75
145
9-19 A semiconductor manufacturer has been ordered by the city to stop discharging acidic waste liquids into the city sewer system. Your analysis shows you should select one of the following three systems. Installed System CleanH2O AcidFree Evergreen Cost $30,000 35,000 80,000 Annual Operating Cost $6,000 5,000 1,000 Salvage value End of 20 yrs $2,000 5,000 40,000
If the system is expected to last and be used 20 years and money is worth 8%, which system should be purchased? Solution CleanH2O AcidFree Evergreen EAC EAC EAC = 6,000 + 30,000(A/P, 8%, 20) - 2,000(A/F, 8%, 20) = $9,013 = 5,000 + 35,000(A/P, 8%, 20) - 5,000(A/F, 8%, 20) = $8,456 = 1,000 + 80,000(A/P, 8%, 20) - 40,000(A/F, 8%, 20) = $8,276
146
9-20 The following alternatives describe possible projects for the use of a vacant lot. In each case the project cost includes the purchase price of the land. Parking Lot Investment Cost Annual Income Annual Operating Expenses $50,000 35,000 25,000 Gas Station $100,000 85,000 70,000 in Year 1, then increasing by 1,000/yr Salvage Value Useful Life 10,000 5 years 10,000 10 years
(a) If the minimum attractive rate of return (MARR) equals 18%, what should be done with the land? (b) Is it possible the decision would be different if the MARR were higher than 18%? Why or why not? (No calculations necessary.) Solution (a) EAWP.L. = (35,000 - 25,000) - 50,000(A/P, 18%, 5) + 10,000(A/F, 18%, 5) = -$4,592 EAWG.S. = (85,000 - 70,000) - 100,000(A/P, 18%, 10) + 10,000(A/F, 18%, 10) - 1,000(A/G, 18%, 10) = -$10,019 Since both EAWs are negative, leave lot vacant. (b) No. Higher MARR favors lower cost projects and the lowest cost project(null) has already been chosen.
9-21 Given the following information about possible investments, what is the best choice at a minimum attractive rate of return (MARR) of 10%? A Investment Cost Annual Benefits Useful Life $5,000 1,200 5 years B $8,000 800 15 years
147
9-22 You are considering purchasing the Press-o-matic or Steam-it-out model automatic ironing system to allow you to handle more dry cleaning business. Either machine will cost the same amount, $5,000. The Press-o-matic will generate a positive cash flow of $1,300 per year for 5 years and then be of no service or salvage value. The Steam-it-out will generate a positive cash flow of $800 per year for 10 years and then be of no service or salvage value. You plan to be in the dry cleaning business for the next 10 years. How would you invest the $5,000 you have in your hand if you feel the time value of money is worth the same as your high interest bank account offers, which is (a) 8%? (b) 12%? Solution (a) Press EAW = 1,300 - 5,000(A/P, 8%, 5) = $47.50 Steam EAW = 800 - 5,000(A/P, 8%, 10) = $55.00 Choose highest EAB, Steam-it-out (b) Press EAW = 1,300 - 5,000(A/P, 12%, 5) = -$87.00 Steam EAW = 800 - 5,000(A/P, 12%, 10) = -$85.00 Choose neither option because both have a negative annual worth.
148
9-23 Data for Machines X and Y are listed below. With an interest rate of 8%, which machine would be selected based upon equivalent annual cost (EAC)? X First cost Annual maintenance Salvage value Useful life Solution Method 1 Method 2 Machine X: Method 1 Method 2 Machine Y: Method 1 Method 2 EAC = 10,000(A/P, 8%, 15) - 1,000(A/F, 8%, 15) + 200 = $1331.20 EAC = (10,000 - 1,000)(A/P, 8%, 5) + 1,000(.08) + 200 = $1331.20 EAC = 5,000(A/P, 8%, 5) - 600(A/F, 8%, 5) + 500 = $1,650.20 EAC = (5,000 - 600)(A/P, 8%, 5) + 600 .08) + 500 = $1,650.20 EAC = P(A/P, i %, n) - S(A/F, i %, n) + other costs EAC = (P - S)(A/P, i %, n) + Si + other costs $5,000 500 600 5 years Y $10,000 200 1,000 15 years
9-24 Consider Projects A and B. Which project would you approve, if the income to both were the same. The expected period of service is 15 years, and the interest rate is 10%. Project A Initial cost Annual operating costs Annual repair costs Salvage value after 15 years Solution Project A: EACA = 50,000(A/P, 10%, 15) + 20,000 5,000(A/F, 10%, 15) = $26,417.50 $50,000 15,000 5,000 5,000 Project B $75,000 10,000 3,000 10,000
149
9-25 Assuming a 10% interest rate, determine which alternative should be selected. A First Cost Uniform Annual Benefit Useful Life Salvage Value Solution Alternative A: EAW = (5,300 - 0)(A/P, 10%, 4) - 1,800 = $127.85 Alternative B: EAW = (10,700 - 200)(A/P, 10%, 8) + 200(.1) - 2,100 = $112.30 Choose alternative A $5,300 1,800 4 years 0 B $10,700 2,100 8 years 200
9-26 A company must decide whether to buy Machine A or Machine B. After 5 years A will be replaced with another A. Machine A First Cost Annual Maintenance Salvage Value Useful Life $10,000 1,000 10,000 5 years Machine B $20,000 0 10,000 10 years
With the minimum attractive rate of return (MARR) = 10%, which machine should be purchased?
150
Solution
EAWA = - 10,000(A/P, 10%, 5) - 1,000 + 10,000(A/F, 10%, 5) = -$2,000 EAWB = - 20,000(A/P, 10%, 10) + 10,000(A/F, 10%, 10) = -$2,627 Therefore Machine A should be purchased.
9-27 The construction costs and annual maintenance costs of two alternatives for a canal are given below. Using equivalent annual cost (EAC) analysis, which alternative would you recommend? Assume 7% interest and infinite life. What is the capitalized cost of maintenance for the alternative you choose? Alternative A Construction cost Annual Maintenance costs Solution (a) Alternative A Alternative B EAC = A + P i = 3.5M + 25M(0.07) = $5,250,000 EAC = A + P i = 2.0M + 50M(0.07) = $5,500,000 $25,000,000 3,500,000 Alternative B $50,000,000 2,000,000
9-28 Two alternatives are being considered by a food processor for the warehousing and distribution of its canned products in a sales region. These canned products come in standard cartons of 24 cans per carton. The two alternatives are: Alternative A. To have its own distribution system. The administrative costs are estimated at $43,000 per year, and other general operating expenses are calculated at $0.009 per carton. A warehouse will have to be purchased, which costs $300,000. To sign an agreement with an independent distribution company, which is asking a payment of $0.10 per carton distributed.
Alternative B.
Assume a study period of 10 years, and that the warehouse can be sold at the end of this period for $200,000. (a) Which alternative should be chosen, if they expect that the number of cartons to be distributed will be 600,000 per year?
151
(b) Find the minimum number of cartons per year which will make the alternative of having a distribution system (Alt A.) more profitable than to sign an agreement with the distribution company (Alt B.) Solution (a) For 600,000 cartons/yr. Alternative A Administration: Annual Costs: Operating Expenses: Capital Expenses* Total = *EAC = (P - S)(A/P, i, n) + S i = (300,000 - 200,000)(A/P, 10%, 10) + 200,000(0.1) = $36,270 Total annual costs = $ 84, 670.00 Alternative B: Total annual costs = 0.10 x 600,000 =$ 60,000 Sign an agreement for distribution (Alt. B) (b) Let M = number of cartons/yr. - The annual cost for alternative B (agreement) = EACAGREEMENT = 0.10M - The annual cost for alternative A (own system) = EACOWN = 43,000 + 0.009M + 36,270 We want EACOWN < EACAGREEMENT 43,000 + .009M + 36,270 < 0.10M 79,270 < (0.10 - 0.009)M 79,270/0.091 < M 871,099 < M Own distribution is more profitable for 871,100 or more cartons/year. .009 x 600,000 = $43,000 5,400 36,270 $84,670
152
9-29 The plant engineer of a major food processing corporation is evaluating alternatives to supply electricity to the plant. He will pay $3 million for electricity purchased from the local utility at the end of this first year and estimates that this cost will increase at $300,000 per year. He desires to know if he should build a 4000 kilowatt power plant. His operating costs (other than fuel) for such a power plant are estimated to be $130,000 per year. He is considering two alternative fuels: (a) WOOD. Installed cost of the power plant is $1200/kilowatt. Fuel consumption is 30,000 tons per year. Fuel cost for the first year is $20 per ton and is estimated to increase at a rate of $2 per ton for each year after the first. No salvage value. (b) OIL. Installed cost is $1000/kw. Fuel consumption is 46,000 barrels per year. Fuel cost is $34 per barrel for the first year and is estimated to increase at $1/barrel per year for each year after the first. No salvage value. If interest is 12%, and the analysis period is 10 years, which alternative should the engineer choose? Solve the problem by equivalent annual cost analysis (EAC). Solution Do Nothing First Cost Annual Oper. Costs Annual Energy Costs Gradient Do Nothing: EAC = 3,000K + 300K(A/G, 12%, 10) = $4,075,500 Wood: EAC = 4,800K(A/P, 12%, 10) + 130K + 600K + 60K(A/G, 12%, 10) = $1,794,700 Oil: EAC = 4,000K(A/P, 12%, 10) + 130K + 1,564K + 46K(A/G, 12%, 10) = $2,566,190 Minimize EAC choose wood 0 0 3,000,000 300,000 30,000 x 2 = Wood 4,000 x 1,200 = 4,800,000 130,000 30,000 x 20 = 600,000 60,000 46,000 x 1 = Oil 4,000 x 1,000 = 4,000,000 130,000 46,000 x 34 =1,564,000 46,000
153
9-30 The manager of F. Roe, Inc. is trying to decide between two alternative designs for an aquacultural facility. Both facilities produce the same number of fish for sale. The first alternative costs $250,000 to build, and has a first-year operating cost of $110,000. Operating costs are estimated to increase by $10,000 per year for each year after the first. The second alternative costs $450,000 to build, and has a first-year operating cost of $40,000 per year, escalating at $5000 per year for each year after the first. The estimated life of both plants is 10 years and each has a salvage value that is 10% of construction cost. Assume an 8% interest rate. Using equivalent annual cost (EAC) analysis, which alternative should be selected? Solution Alternative 1 First Cost Uniform Annual Cost for 10 years Gradient Salvage in year 10 $250,000 110,000 10,000 25,000 Alternative 2 $450,000 40,000 5,000 45,000
Alternative 1: EAC = 250,000(A/P, 8%, 10) - 25,000(A/F, 8%, 10) + 110,000 + 10,000(A/G, 8%, 10) = $184,235 Alternative 2: EAC = 450,000(A/P, 8%, 10) - 45,000(A/F, 8%, 10) + 40,000 + 5,000(A/G, 8%, 10) = $123,300 Fixed Output (same amount of fish for sale) Minimize EAC Choose Alternative 2
9-31 Two alternative investments are being considered. What is the minimum uniform annual benefit that will make Investment B preferable over Investment A? Assume interest is 10%. Year 0 1-5 A -$500 +150 B -$700 ?
154
Solution
NPWA = NPWB -500 + 150(P/A, 10%, 5) = -700 + X(P/A, 10%, 5) X = $202.76 Alternate Solution: EAWA = EAWB -500(A/P, 10%, 5) + 150 = -700(A/P, 10%, 5) + X X = $202.76
9-32 Consider two investments: (1) Invest $1000 and receive $110 at the end of each month for the next 10 months. (2) Invest $1200 and receive $130 at the end of each month for the next 10 months. If this were your money, and you want to earn at least 12% interest on your money, which investment would you make, if any? Solve the problem by annual cash flow analysis. Solution EAW Analysis: Alternative 1: Alternative 2: EAB - EAC = 110 - 1,000(A/P, 1%, 10) = $4.40 EAB - EAC = 130 - 1,200(A/P, 1%, 10) = $3.28
9-33 An engineer is considering the purchase of a new set of batteries for a tractor. Given the cost, annual benefit and useful life, conduct a net future worth (NFW) analysis to decide which alternative to purchase if i = 12%. A Cost Annual benefit Useful life $150 40 6 yrs B $90 40 3 yrs
155
9-34 Two mutually exclusive alternatives are found to be acceptable but A lasts twice as long as B. With no additional information given, which alternative is likely to have the shorter payback period? Why? Solution Alternative B is likely to have the shorter payback period. Since both are acceptable, they must return benefits greater than costs within the useful life. But B has only half as long a life, so it will almost certainly return benefits greater than cost before A and will therefore have a shorter payback period.
9-35 A cannery is considering different modifications to some of their can fillers in two plants that have substantially different types of equipment. These modifications will allow better control and efficiency of the lines. The required investments amount to $135,000 in Plant A and $212,000 for Plant B. The expected benefits(which depend on the number and types of cans to be filled each year) are as follows: Plant A Benefits $ 73,000 73,000 80,000 80,000 80,000 Plant B Benefits $ 52,000 85,000 135,000 135,000 135,000
Year 1 2 3 4 5
(a) Assuming MARR = 10%, which alternative is should be chosen? (b) Which alternative should be chosen based on payback period?
156
Solution
a) May be solved in various ways. Use PW method. NPWA = -135K + 73K(P/A, 10%, 2) + 80K(P/A, 10%, 3)(P/F, 10%, 2) = $156,148.50 NPWB = -212K + 52K(P/F, 10%, 1) + 85K(P/F, 10%, 2) + 135K(P/A, 10%, 3)(P/F, 10%, 2) = $182,976.80 Therefore, modifications to plant B are more profitable b) PLANT A CUMMULATIVE BENEFITS BENEFITS 73,000 73,000 73,000 146,000* 80,000 226,000 *The PBP of A is less than 2 years (1.85 years) PLANT B CUMMULATIVE BENEFITS BENEFITS 52,000 52,000 85,000 137,000 135,000 272,000** **The PBP of B is less than 3 years (2.55 years)
YEAR 1 2 3
Therefore, although not the most profitable, alternative A has the shortest payback period and should be chosen.
9-36 In this problem the minimum attractive rate of return is 10%. Three proposals are being considered. PROPOSAL A A = $1000
0 1 2
PROPOSAL B A = $1,000
0 1 2 3 0 1
PROPOSAL C A = $1,000
2 3 4 5 6
$1,700 $2,100 $3,750 (a) Which proposal would you choose using Future Value analysis? (b) How many years for Payback for each alternative?
157
1,700 = 17 . years 1,000 2,100 = 2.1 years 1,000 3,750 = 3.75 years 1,000
Proposal B
Proposal C
9-37 The construction firm of Cagle, Owens, and Wells (COW) is considering investing in newer, more productive forming equipment. The data concerning the three best alternatives is presented below. Initial Investment Annual Net Income Computed IRR $50,000 5,093 8% $22,000 2,077 7% $15,000 1,643 9%
Each alternative has a twenty year useful life with no salvage value. If the MARR for COW is 7%, which alternative should be chosen? (Show ranges for IRR were appropriate.)
158
Solution
Incremental analysis is required for IRR. A-B NPW = 0 at IRR. 0 = [-50,000 - (-22,000)] + (5,093 - 2,077)(P/A, i%, 20) (P/A, i%, 20) = 28,000/3,016 = 9.284 8% < IRR < 9% Choose most expensive, A A-C (P/A, i%, 20) = 35,000/3,450 = 10.145 7% < IRR < 8% Choose most expensive, A
9-38 Barber Brewing is considering investing in one of the following opportunities. First Cost Annual Income Annual Cost A $100.00 100.00 73.62 B $130.00 90.78 52.00 C $200.00 160.00 112.52 D $330.00 164.55 73.00 E $000.00 00.00 00.00
Each alternative has a five year useful life. The firms MARR is 8%. Which alternative should be selected? Solution NPW = 0 at IRR. 0 = -First cost + net income (P/A, i%, n) IRRA IRRB IRRC IRRD IRRE P/A = 100/26.38 = 3.791 P/A = 130/38.78 = 3.352 P/A = 200/47.48 = 4.212 P/A = 330/91.55 = 3.605 Therefore (P/A, i%, n) = First cost/net income IRR = 10% IRR = 15% IRR = 6% IRR = 12% IRR = 0%
Eliminate, does not meet MARR Eliminate, does not meet MARR
Incremental analysis required IRRD-B P/A = 200/52.77 = 3.790 IRRD-A P/A = 230/65.17 = 3.529 IRR = 10% 12% < IRR < 15% Choose D Choose D
159
9-39 The construction firm of Burns, Owens, Wells, and Lowrance LLC must replace a piece of heavy earth moving equipment. The information concerning the two best alternatives is summarized below. Each alternative is expected to last six years. If BOWL LLC has a cost of capital of 11%, which alternative should be chosen? Use IRR analysis. Big Bite $15,000 3,000 2,000 Dip Big $22,500 1,500 4,000
Since individual IRRs cannot be calculated perform incremental analysis. DD - BB Year 0 1-6 6 First Cost Cost Savings 1500(P/A, i%, 6) Salvage Value 2000(P/F, i%, 6) 10% -7,500.00 6,532.50 1,129.00 $161.50 12% -7,500.00 6,166.50 1,013.20 -$320.30
9-40 Horizon Wireless must rebuild a cell tower recently destroyed on a tornado. If made of normal steel, the tower will cost $30,000 to construct and should last for 15 years. Maintenance will cost $1,000 per year. If a more corrosion resistant steel is used, the tower will cost $36,000 to build and the annual maintenance cost will be reduced to $250 per year. Determine the IRR of building the corrosion resistant bridge. If Horizon requires a return of 9% on its capital projects, which tower should they build? Solution NPWNS = -30,000 - 1000(P/A, i%, 15) NPWCRS = -36,000 - 250(P/A, i%, 15) -30,000 - 1000(P/A, i%, 15) = -36,000 - 250(P/A, i%, 15) 6000 - 750(P/A, i%, 15) = 0 (P/A, i%, 15) = 8.000
160
From tables
Build the tower using corrosion resistant steel (the most expensive)
9-41 A new electric generation plant is expected to cost $43,250,000 to complete. The revenues generated by the new plant are expected to be $3,875,000 per year while operational expenses are estimated to $2,000,000 per year. If the plant is expected to last forty years and the electric authority uses 3% as its cost of capital, determine if the plant should be built? (Use B/C ratio analysis.) Solution AWBENEFITS = $3,875,000 AWCOSTS = 43,500,000(A/P, 3%, 40) + 2,000,000 = $3,872,725 B/C =
Build Plant
9-42 Sugar-N-Spice, a cookie factory, needs a new cookie cutter machine. They have narrowed their choices to three different types of machines, which are presented below. Cost Annual Net Savings IRR A $18,000 1,055 7% B $25,000 2,125 9% C $15,000 1,020 8%
Each choice has a twenty-five year useful life with no salvage value. If the MARR for Sugar-NSpice is 8%, which choice should be chosen? Solution B-A 25,000-18,000 = P/A = 6.5421 = 12%~15% 2125-1055 B-C 25,000-15,000 = P/A = 9.0498 = 8%~9% 2125-1020 Choose most expensive, B Choose most expensive, B
161
162
9-44 A new processor is needed. The two best alternatives are presented below. If the cost of capital is 6%, which alternative should be chosen? Alternative A B First Cost $75,000 $50,000 Annual Benefits $8,000 $6,000 Useful Life 8 yrs 8 yrs
Solution Using Annual Worth Alternative A EAW = -75,000(A/P, 6%, 8) + 8,000 = -$4,075 Alternative B EAW = -50,000(A/P, 6%, 8) + 6,000 = -$2,050 Choose B
Year 0 1 2 3 4
Both the old and new ovens have similar productivities and energy costs. Should the oven be replaced this year, if the MARR equals 10%? Solution The old oven (defender) EAC Capital Recovery (P-S)x(A/P,10%,n) + Si --5,000.00 4,857.20 4,718.90 4,801.50 EAC Maintenance = 9,500 + 100(A/G,10%,n) -9,500.00 9,547.60 9,593.70 9,638.10
Year 0 1 2 3 4
163
164
The new oven (challenger) EAC Capital Recovery (P-S)x(A/P,10%,n) + Si --13,000.00 12,762.00 12,229.40 11,879.00 EAC Maintenance = 9,500 + 100(A/G,10%,n) -0 0 302.10a 883.55b
Year 0 1 2 3 4
a b
1,000(A/F, 10%, 3) = $302.10 [1,000(F/P, 10%, 1) + 3000](A/F, 10%, 4) = $883.55 *Economic life = 3 years, with EAC = $12,531.50 Since EAC defender > EAC challenger (14,312.6 > 12,531.5) replace oven this year.
10-2 The cash flow diagram below indicates the costs associated with a piece of equipment. The investment cost is $5000 and there is no salvage. During the first 3 years the equipment is under warranty so there are no maintenance costs. Then the estimated maintenance costs. Then the estimated maintenance costs over 15 years follow the pattern shown. To show you can do the calculations required to find the most economic useful life, determine the equivalent annual cost (EAC) for n = 12 if the minimum attractive rate of return (MARR) = 15%. You must use gradient and uniform series factors in your solution.
0 1 2 3 4 5 6 7 8 9 10 11 12 13 14 15
Solution EAC = 5,000(P/A, 15%, 12) + 150(F/A, 15%, 9)(A/F, 15%, 12) + 100(P/G, 15%, 7)(P/F, 15%, 5)(A/P, 15%, 12) = $1,103
165
10-3 A hospital is considering buying a new $40,000 diagnostic machine that will have no salvage value after installation, as the cost of removal equals any sale value. Maintenance is estimated at $2000 per year as long as the machine is owned. After ten years the radioactive ion source will have caused sufficient damage to machine components that safe operation is no longer possible and the machine must be scrapped. The most economic life of this machine is: (Select one) (a) One year since it will have no salvage after installation. (b) Ten years because maintenance doesnt increase. (c) Less than ten years but more information is needed to determine it. Solution The correct answer is (b).
10-4 A petroleum company, whose minimum attractive rate of return is 10%, needs to paint the vessels and pipes in its refinery periodically to prevent rust. Tuff-Coat, a durable paint, can be purchased for $8.05 a gallon while Quick-Cover, a less durable paint, costs $3.25 a gallon. The labor cost of applying a gallon of paint is $6.00. Both paints are equally easy to apply and will cover the same area per gallon. Quick-Cover is expect to last 5 years. How long must Tuff-Coat promise to last to justify its use? Solution This replacement problem requires that we solve for a breakeven point. Let N represent the number of years Tuff-Coat must last. The easiest measure of worth to use in this situation is equivalent annual cash flow (EACF). Although more computationally cumbersome, others could be used and if applied correctly they would result in the same answer. Find N such that EACFTC = EACFQC 14.05(A/P, 10%, N) = 9.25 (A/P, 10%, 5) (A/P, 10%, N) = 0.17367 Therefore N = 9 years Tuff-Coat must last at least 9 years. Notice that this solution implicitly assumes that the pipes need to be painted indefinitely (i.e., forever) and that the paint and costs of painting never change (i.e., no inflation or technological improvements affecting the paint or the cost to produce and sell paint, or to apply the paint).
166
10-5 Ten years ago Hyway Robbery, Inc. installed a conveyor system for $8000. The conveyor system has been fully depreciated to its zero salvage value. The company is considering replacing the conveyor because maintenance costs have been increasing. The estimated end-of-year maintenance costs for the next five years are as follow: Year 1 2 3 4 5 Maintenance $1,000 1,250 1,500 1,750 2,000
At any time the cost of removal just equals the value for scrap metal. The replacement the company is considering has an equivalent annual cost (EAC) of $1028 at its most economic life. The company requires a minimum attractive rate of return (MARR) of 10%. (a) Should the conveyor be replaced now? Show the basis used for the decision. (b) Now assume the old conveyor could be sold at any time as scrap metal for $500 more than the cost of removal. All other data remain the same. Should the conveyor be replaced? Solution a) Since the current value ($0.00) is not changing but maintenance costs are increasing, the most economic life is one year. Year 0 1 S Cash Flow 0 -1,000 0
Defender uniform equivalent cost: EACD = $1,000 Since EACD < EACC , keep the old conveyor for now. b) Year 0 1 S Cash Flow -500 -1,000 +500
EACD = 1,000 + 500(A/P, 10%, 1) - 500(A/F, 10%, 1) = $1,050. Since EACD > EACC , replace the old conveyor.
167
10-6 Ten years ago, the Cool Chemical Company installed a heat exchanger in its plant for $10,000. The company is considering replacing the heat exchanger because maintenance cost have been increasing. The estimated maintenance costs for the next 5 years are as follow: Year 1 2 3 4 5 Maintenance $1,000 1,200 1,400 1,600 1,800
Whenever the heat exchanger is replaced, the cost of removal will be $1,500 more than the heat exchanger is worth as scrap metal. The replacement the company is considering has an equivalent annual cost (EAC) = $900 at its most economic life. Should the heat exchanger be replaced now if the companys minimum attractive rate of return (MARR) is 20%? Solution Since the current value ($-1,500) is not changing but maintenance costs are increasing, the most economic life is one year. Year 0 1 S Cash Flow +1,500 -1,000 -1,500
Equivalent annual cost of the defender: EACD = 1,000 + 1,500(A/F, 20%, 1) - 1,500(A/P, 20%, 1) = $700 Since EACD < EACC , keep the old heat exchanger for now. 10-7 An engineer is trying to determine the economic life of new metal press. The press costs $10,000 initially. First year maintenance cost is $1,000. Maintenance cost is forecast to increase $1,000 per year for each year after the first. Fill in the table below and determine the economic life of the press. Consider only maintenance and capital recovery in your analysis. Interest is 5%.
168
Solution Year 1 2 3 4 5 6 7 8 Maintenance Cost $1,000 2,000 3,000 4,000 5,000 6,000 7,000 8,000 EAC of Capital Recovery $11,500 6,151 4,380 3,503 2,983 2,642 2,404 2,229 EAC of Maintenance $1,000 1,465 1,907 2,326 2,723 3,097 3,450 3,781 Total EAC $12,500 7,616 6,287 5,829 5,706 5,739 5,854 6,010
Economic Life = 5 yrs (EAC = minimum) EAC of Capital Recovery = $10,000 (A/P, 15%, n) EAC of Maintenance = $1,000 + 1,000 (A/G, 15%, n)
10-8 A manufacturer is contemplating the purchase of an additional forklift truck to improve material handling in the plant. He is considering two popular models, the Convair T6 and the FMC 340. The relevant financial data are shown below. His minimum attractive rate of return (MARR) is 12%. First Salvage Annual Operating Model Cost Life Value Cost Convair T6 $20,000 6 years $2,000 $8,000 FMC 340 29,000 7 years 4,000 4,000 (a) Which model is more economical? (b) List two important assumptions that are implicit in your computations in (a).
169
The FMC is more economical. b) That either truck can be (1) repeated identically into the indefinite and (2) the service to be provided (material handling) is required forever.
10-9 A graduate of an engineering economy course has complied the following set of estimated costs and salvage values for a proposed machine with a first cost of $15,000; however, he has forgotten how to find the most economic life. Your task is to show him how to do this by calculating the equivalent annual cost (EAC) for n = 8, if the minimum attractive rate of return (MARR) is 15%. You must show how to use gradients and uniform series factors in your solution. Life (n) Years 1 2 3 4 5 6 7 8 9 10 Estimated End-of-Year Maintenance $ 0 $ 0 300 300 800 1,300 1,800 2,300 2,800 3,300 Estimated Salvage if sold in Year n $10,000 9,000 8,000 7,000 6,000 5,000 4,000 3,000 2,000 1,000
Remember: Calculate only one EAC (for n = 8). You are not expected to actually find the most economical life.
170
Solution
EAC = 15,000(A/P, 15%, 8) = $3,344 EAC = -3,000(A/F, 15%, 8) = -$219 EAC = 300(F/A, 15%, 6)(A/F, 15%, 8) + 500(P/G, 15%, 5)(P/F, 15%, 3)(A/P, 15%,8) = $615 EAC8 = $3,740
(A complete analysis would show that the most economic life is 7 years, with EAC = $3,727)
10-10 An existing machine has operating costs of $300 per year and a salvage value of $100 (for all years). A new replacement machine would cost $1,000 to purchase, and its operating cost over the next period of t years (not per year) is M = 200t + 10t2 . Assume i = zero percent. (a) What is the most economic life t* for the new machine? (b) Should the old machine be replaced with the new one?
b) AC* = AC (10) =
+ 200 + 10 (10)
= 400 > Annual cost of old machine for any number of years $No, keep old one.
10-11 The computer system used for production and administration control at a large cannery is being considered for replacement. Of the available replacement system, Challenger I has been considered the best. However, it is anticipated that after one year, the Challenger II model will become available, with significant technological modifications. The salvage value projections for these three systems are summarized below. Assuming that their performance would otherwise be comparable, should we replace the existing system either this year or next year? Assume the MARR equals 12%, and a useful life of 5 years on all alternatives.
171
Challenger I Year 0 1 2 3 4 5 Salvage value at end-of-year P = 25K 22K 21K 20K 16K 10K EAC Capital Recovery = (P - S)(A/P, 12%, n) + Si -6,000.00 4,886.80 4,481.50* 4,882.80 5,361.00
3.
Challenger II Year1 0 1 2 3 4 5 Salvage value at end-of-year P = 24K 23K 23K 22K 16K 10K EAC Capital Recovery = (P - S)(A/P, 12%, n) + Si -3,880.00 3,351.70* 3,472.60 4,553.60 5,083.60
*Economic life = 2 yrs EAC = $4,910.40 Year numbers do not refer to same time scale as for Challenger 1
1
172
Note EACChallenger II < EACChallenger I < EACDefender , but should we replace it now or should we wait one year for the Challenger II? Alternative A: Dont wait
EACB = [6,400(P/A, 12%, 1) + (3,351,7(P/A, 12%, 2)(P/F, 12%, 1)](A/P,12%,3) cost of keeping cost of challenger II at its one more year best (2 yrs economic life) = $4,484.49 Since EACA ! EACB we should preferably wait one year, although strictly speaking we can choose either option.
10-12 A truck salesman is quoted as follows: Even though our list price has gone up to $42,000, Ill sell you a new truck for the old price of only $40,000, an immediate savings of $2,000, and give you a trade-in allowance of $21,000, so your cost is only ($40,000 - 21,000) = $19,000. The book value of your old truck is $12,000, so youre making an additional ($21,000 - 12,000) = $9,000 on the deal. The salesman adds, Actually I am giving you more trade-in for your old truck than the current market value of $19,500, so you are saving an extra ($21,000 - 19,500) = $1,500. (a) In a proper replacement analysis, what is the first cost of the defender? (b) In a proper replacement analysis, what is the first cost of the challenger?
Solution (a) $19,500 The defender 1st cost is always the current market value, not trade-in or book value. (b) $38,500 With an inflated trade-in of $1,500 (21,000 - 19,500), the new truck can be purchased for $40,000. Therefore, the appropriate value for a replacement analysis is: $40,000 $1,500 = $38,500.
The owner of the company has $5000 of savings currently invested in securities yielding 8% that could be used for the company. a) Assuming his funds are limited to his savings, what is the apparent MARR? b) If he can borrow money at 10%, how much should be borrowed? Solution a) Alt A E D C B Investment 1,000 500 3,000 3,000 2,500 Cumulative Investment 1,000 1,500 4,500 7,500 10,000 IRR 30% 25% 15% 11% 9%
! MARR = 15%
b) Do all projects with a rate of return > 10%. Thus Alternatives A, E, D, & C with a total initial cost of $7,500 would be selected. Since only $5,000 is available, $2,500 would need to be borrowed.
173
174
11-2 The capital structure of a firm is given below. Source of Capital Loans Bonds Common Stock Percent of Capitalization 35 30 35 Interest Rate 17% 8% 12%
The combined state and federal income tax rate for the firm is 42%. What is the after-tax and before-tax cost of capital to the firm? Solution Before Tax Cost of Capital 0.35 x 17% + 0.30 x 8% + 0.35 x 12% = 12.55% After Tax Cost of Capital 0.35 x 17% (1-0.42) + 0.30 x 8% (1-0.42) + 0.35 x 12% = 9.04%
11-3 A small construction company identifies the following alternatives that are independent, except where noted. Alternative Repair bulldozer Replace Backhoe With Model A With Model B Buy new dump truck Model X Model Y Buy computer Model K Model L Initial Cost $5,000 20,000 25,000 20,000 30,000 5,000 10,000 Incremental Rate of Return 30.0% 15.0% 10.5% 20.0% 14.0% 12.0% 9.5% On Investment Over 0 0 2A 0 3X 0 4K
1. 2.
3.
4.
(a) Assuming the company has $55,000 available for investment and it is not able to borrow money, what alternatives should be chosen, and what is MARR? (b) If the company can also borrow money at 10%, how much should be borrowed, and which alternatives should be selected?
175
"IRR 30.0% 20.0% 15.0% 14.0% 12.0% 10.5% 9.5% 1 Repair bulldozer 2A Backhoe model A 3Y Dump truck model Y No computer
Minimum Attractive-of Rate-of-Return = 14% b) Borrow $10,000. choose Projects: 1 2B 3Y 4K Repair bulldozer Backhoe model B Dump truck model Y Computer model K
11-4 The following independent and indivisible investment opportunities are available: Investment A B C D E F Initial Cost $200 100 50 100 50 Unlimited Rate of Return 20% 22% 19% 18% 15% 7%
(a) Which investment(s) should be selected if the minimum attractive rate of return (MARR) is greater then or equal to 18% assuming an unlimited budget? (b) Which investment(s) should be selected if the available budget is $400 and the MARR is greater than or equal to 14%?
176
Solution (a) A, B, C, D ! Choose all IRRs > 18% since budget is unlimited. (b) A, B, D ! Choose D instead of C because it yields a greater overall return and fully invests the $400 budget. Investment A B C D Initial Cost $200 100 50 100 Return$ 20% 22% 19% 18% Return in $ $40.00 22.00 9.50 18.00
Total $ return on A + B + C = 40 + 22 + 9.50 +50(.14)* = $78.50 Total $ return on A + B + D = 40 + 22 +18 = $80.00 *Assumes the remaining $50 can be invested at the MARR. This is not always true. Thus yielding an even lower $ return.
11-5 A city engineer calculated the present worth of benefits and costs of a number of possible projects, based on 10% interest and a 10 year analysis period. Costs and Benefits in 1000s Project: Present Worth of Costs Present Worth of Benefits A 75 105 B 70 95 C 50 63 D 35 55 E 60 76 F 25 32 G 70 100
If 10% is a satisfactory minimum attractive rate of return (MARR), which project(s) should be selected if $180,000 is available for expenditure? Solution Project: Present Worth of Costs Present Worth of Benefits NPW NPW/C A 75 105 30 .400 B 70 95 25 .357 C 50 63 13 .260 D 35 55 20 .571 E 60 76 16 .267 F 25 32 7 .280 G 70 100 30 .428
177
11-6 Barber Brewing is in the process of determining the capital budget for the coming year. The following projects are under consideration. A First Cost Annual Income Annual Cost $10,000 10,000 7,362 B $13,000 9,078 5,200 C $20,000 16,000 11,252 D $33,000 16,455 7,300
All projects have a five year useful life. Which alternative(s) should be selected if Barbers budget is set at $50,000? Solution NPW = 0 at IRR. 0 = -First cost + net income (P/A, i%, n) Therefore (P/A, i%, n) = First cost/net income Rank IRRA IRRB IRRC IRRD P/A = 10,000/2,638 = 3.791 P/A = 13,000/3,878 = 3.352 P/A = 20,000/4,748 = 4.212 P/A = 33,000/9,155 = 3.605 IRR = 10% IRR = 15% IRR = 6% IRR = 12% 3 1 4 2
11-7 P&J Brewing has the following capital structure: Type Mortgages Bonds Common Stock Preferred Stock Retained Earnings Amount $25,000,000 180,000,000 100,000,000 50,000,000 120,000,000 Average Minimum Return 7% 9% 10% 8% 10%
178
11-8 ABC Builders has asked each of its four regional managers to submit requests for capital outlays for the following fiscal year. The CEO of ABC has decided to fund the top request from each region and to fund two additional requests with the provision that no region has more than two projects funded. Using the information provided below determine which projects should be funded. Region Southern (S) Project A B C D A B C D A B C A B Cost $ 90,000 40,000 60,000 120,000 50,000 120,000 75,000 50,000 50,000 80,000 75,000 60,000 50,000 Annual Benefit $16,400 15,000 20,400 27,600 10,000 36,700 21,600 16,200 16,700 23,500 26,100 16,900 15,300 Life (Years) 15 5 5 20 20 15 5 5 20 5 10 15 10
Midwest (MW)
Pacific (P)
Solution Region Southern (S) Project A B C D A B C D A B C Cost $ 90,000 40,000 60,000 120,000 50,000 120,000 75,000 50,000 50,000 80,000 75,000 IRR 16.3% 25.4 20.8 22.6 19.4 30.0 13.5 18.6 33.3 14.4 32.8
Midwest (MW)
179
Choose (S)B, (MW)B, (NE)A, (P)B, (NE)C, and (P)A Capital Budget = 40,000 + 120,000 + 50,000 + 50,000 + 75,000 + 60,000 = $395,000
Chapter 12 Depreciation
12-1 Some seed cleaning equipment was purchased in 1995 for $8,500 and is depreciated by the double declining balance (DDB) method for an expected life of 12 years. What is the book value of the equipment at the end of 2000? Original salvage value was estimated to be $2,500 at the end of 12 years. Solution Book Value = P(1 2 N
)n
2 6 ) 12
= 8,500(1 -
= $2,846.63
This can be checked by doing the year-by-year computations: YEAR 1995 1996 1997 1998 1999 2000 DDB (8,500-0) = $1,416.67 (8,500-1,416.67) = 1,180.56 (8,500-2,597.23) = 983.80 (8,500-3,581.03) = 819.83 (8,500-4,400.86) = 683.19 (8,500-5,084.05) = 569.32
12-2 Suds-n-Dogs just purchased new automated wiener handling equipment for $12,000. The salvage value of the equipment is anticipated to be $1,200 at the end of its five year life. Using MACRS, determine the depreciation schedule. Solution Three year class is determined. Year 1 2 3 4 Depreciation $3,999.60 5,334.00 1,777.20 889.20
179
180
Chapter 12 Depreciation
12-3 An asset will cost $1,750 when purchased this year. It is further expected to have a salvage value of $250 at the end of its five year depreciable life. Calculate complete depreciation schedules giving the depreciation charge, D(n), and end-of-year book value, B(n), for straight line (SL), sum of the years digits (SOYD), double declining balance (DDB), and modified accelerated cost recovery (MACRS) depreciation methods. Assume a MACRS recovery period of 5 years. Solution SL n 0 1 2 3 4 5 6 D(n) 300 300 300 300 300 B(n) 1,750 1,450 1,150 850 550 250 SOYD D(n) B(n) 1,750 500 1,250 400 850 300 550 200 350 100 250 DDB D(n) B(n) 1,750 700 1,050 420 630 252 378 128 250 0 250 MACRS D(n) B(n) 1,750.00 350.00 1,400.00 560.00 840.00 336.00 504.00 201.60 302.40 201.60 100.80 100.80 0.00
12-4 Your company is considering the purchase of a second-hand scanning microscope at a cost of $10,500, with an estimated salvage value of $500 and a projected useful life of four years. Determine the straight line (SL), sum of years digits (SOYD), and double declining balance (DDB) depreciation schedules. Solution Year 1 2 3 4 SL 2,500 2,500 2,500 2,500 SOYD 4,000 3,000 2,000 1,000 DDB 5,250.00 2,625.00 1,312.50 656.25
12-5 A piece of machinery costs $5,000 and has an anticipated $ 1,000 resale value at the end of its five year useful life. Compute the depreciation schedule for the machinery by the sum-of-years-digits method.
Chapter 12 Depreciation
Solution Sum - of - years - digits = 1st year depreciation = 2 year depreciation = 3 year depreciation = 4 year depreciation = 5th year depreciation =
th rd nd
181
n 2
(n + 1) =
5 2
(6) = 15
5 15 4 15 3 15 2 15 1 15
(5,000 - 1,000) = $1,333 (5,000 - 1,000) = 1,067 (5,000 - 1,000) = (5,000 - 1,000) = (5,000 - 1,000) = 800 533 267
12-6 A new machine costs $12,000 and has a $1200 salvage value after using it for eight years. Prepare a year-by-year depreciation schedule by the double declining balance (DDB) method. Solution DDB Deprecation = Year Deprecation
2 N
(P - !D) 2 2,250 3 1,688 4 1,266 5 949 6 712 7 534 8* 401 Total $10,800
1 3,000
*Book value cannot go below declared salvage value. Therefore the full value of year eights depreciation cannot be taken.
12-7 To meet increased sales, a large dairy is planning to purchase 10 new delivery trucks. Each truck will cost $18,000. Compute the depreciation schedule for each truck, using the modified accelerated cost recovery system (MACRS) method, if the recovery period is 5 years. Solution Year 1 2 3 4 5 6 Depreciation $3,600.00 5,760.00 3,456.00 2,073.60 2,073.60 1,036.80
182
Chapter 12 Depreciation
12-8 The XYZ Research Company purchased a total of $215,000 worth of assets in the 2003. Determine the MACRS depreciation schedule using the maximum allowable 179 expenses. Solution Maximum 179 = $25,000 Assets purchased = $215,000, the maximum allowable 179 = 25,000 (215,000 200,000) = 15,000 Research equipment is five year recovery class Basis = 215,000 - 15,000 = 210,000 Year 1 2 3 4 5 6 Depreciation $41,000 65,600 39,360 23,616 23,616 11,808
12-9 A used piece of depreciable property was bought for $20,000. If it has a useful life of 10 years and a salvage value of $5,000, how much will it be depreciated in the 9th year, using the 150% declining balance schedule? Solution Depreciation =
1.5P 1.5 n #1 1.5(20,000) 1.5 9 #1 1# 1# $ 10 N N 10
! "
! "
= $817.50
! "
1.5 N
$ 20,000 1#
! "
1.5 10
= $5,449.80
Because the salvage value is $5,000 in the 9th year you can only depreciate $449.80 (5,449.80 - 5,000). The $817.50 would have brought the book value below the salvage value of $5,000.
Chapter 12 Depreciation
183
12-10 A front-end loader cost $70,000 and has an estimated salvage value of $10,000 at the end of 5 years useful life. Compute the depreciation schedule, and book value, to the end of the useful life of the tractor using MACRS depreciation. Solution Five year recovery period is determined. Year 1 2 3 4 5 6 Depreciation $14,000 22,400 13,440 8,064 8,064 4,032 Book Value 70,000 - 14,000 = $56,000 56,000 - 22,400 = 33,600 33,600 - 13,440 = 20,160 20,160 - 8,064 = 12,096 12,096 - 8,064 = 4,032 4,032 - 4,032 = 0
12-11 A machine costs $5000 and has an estimated salvage value of $1000 at the end of 5 years useful life. Compute the depreciation schedule for the machine by (a) Straight line (SL) (b) Double declining balance (DDB) (c) Sum of years digits (SOYD) Solution (a) SL =
P #S 5,000#1,000 $ N 5
= $800/year
(b) DDB =
2 P # ' Dc !t" N
&
Depreciation = $2,000 $2,000 = 1,200 1,200 = 720 720 = 432 " 80 = 259.20" 0 $4,611.20 $4,000
184
Chapter 12 Depreciation
N 5 (N ( 1) $ (6) $ 15 2 2
5 15 4 15 3 15 2 15 1 15
(c) SOYD =
12-12 A lumber company purchased a tract of timber for $70,000. The value of the 25,000 trees on the tract was established to be $50,000. The value of the land was established to be $20,000. In the first year of operation, the lumber company cut down 5000 trees. What was the depletion allowance for the year? Solution For standing timber only cost depletion (not percentage depletion) is permissible. Five thousand of the trees were harvested therefore 5,000/25,000 = 0.20 of the tract was depleted. Land is not considered depletable, only the timber, which is valued at a total of $50,000. Therefore, the first years depletion allowance would be = 0.20($50,000) = $10,000.
12-13 A machine was purchased two years ago for $50,000 and had a depreciable life of five years and no expected salvage value. The owner is considering an offer to sell the machine for $25,000. For each of the depreciation methods listed, fill in the table below to determine the deprecation for year 2, and the book value at the end of year 2. (Assume a five year MARCS recovery period.) Depreciation For Year 2 Sum-Of-Years-Digits (SOYD) Straight Line (SL) Double Declining Balance (DDB) Modified Accelerated Cost Recovery System (MACRS) End of Year 2 Book Value
Chapter 12 Depreciation
Solution Depreciation For Year 2 $13,333 10,000 12,000 16,000
185
Sum-Of-Years-Digits (SOYD) Straight Line (SL) Double Declining Balance (DDB) Modified Accelerated Cost Recovery System (MACRS) SOYD
N 5 (N (1) $ (6) $15 2 2 depreciation year 1 = 5 (50,000# 0) 15 depreciation year 2 = 4 (50,000# 0) 15
SOYD =
= $16,666 = $13,333
cumulative depreciation = 16,666 + 13,333 = $30,000 book value = P - cumulative depreciation = 50,000 - 30,000 = $20,000 SL depreciation = (P - S)/N = (50,000 - 0)/5 = $10,000 per year cumulative depreciation = 2 x 10,000 = $20,000 book value = 50,000 - 20,000 = $30,000 DDB: depreciation year 1 = depreciation year 2 =
2 (Book Value) = 2 (50,000) = N 5 2 ( 50,000 - 20,000) = $12,000 5
$20,000
cumulative depreciation = 20,000 + 12,000 = $32,000 book value = 50,000 - 32,000 = $18,000 MACRS: depreciation year 1 = (20%)(50,000) = $10,000 depreciation year 2 = (32%)(50,000) = $16,000 cumulative depreciation = 10,000 + 16,000 = $26,000
186
Chapter 12 Depreciation
book value = 50,000 - 26,000 = $24,000 12-14 In the production of beer, a final filtration is given by the use of Kieselguhr or diatomaceous earth, which is composed of the fossil remains of minute aquatic algae, a few microns in diameter and composed of pure silica. A company has purchased a property for $840,000 that contains an estimated 60,000 tons. Compute the depreciation charges for the first three years, if a production (or extraction) of 3000 tons, 5000 tons, and 6000 tons are planned for years 1, 2, and 3, respectively. Use the cost-depletion methods, assuming no salvage value for the property. Solution Total diatomaceous earth in property = 60,000 tons Cost of property = $480,000 Then,
depletion allowance $840,000 $ $ $14 / ton tons extracted 60,000 tons
Year 1 2 3
12-15 A pump cost $1,000 and has a salvage value of $100 after a life of five years. Using the double declining balance depreciation method, determine: (a) The depreciation in the first year. (b) The book value after five years. (c) The book value after five years if the salvage was only $50. Solution a) Rate =
200% 5
= 40% = .4
)n
B.V. = maximum of {s.v.; 1,000(1-.4)5} = maximum of {100, 77.76} = $100 c) B.V. = maximum of {s.v.; 1,000(1-.4)5} = maximum of {50, 77.76} = $77.76
Chapter 12 Depreciation
187
12-16 Adventure Airlines recently purchased a new baggage crusher for $50,000. It is expected to last for 14 years and have an estimated salvage value of $8,000. Determine the depreciation charge on the crusher for the third year of its life and the book value at the end of 8 years, using sum-ofdigits depreciation. Solution SOYD depreciation for 3rd year Sum of Years digits =
n 14 (n ( 1) $ (14 + 1) $ 105 2 2
remaining life at beginning of year ' years digits
12 105
(P - S)
14+13+12+11+10+9+8+7 105
(50,000 - 8,000)
(42,000) = $33,600
12-17 A small wood chipping company purchases a new chipping machine for $150,000. The total cost of assets purchased for the tax year 2001 is $210,000. If the company chooses to use their entire 179 expense on the wood chipper, determine the MACRS depreciation schedule. (ADR of the chipper is 12 years.) Solution Maximum 179 = $24,000 (for tax year 2001) Assets purchased = $210,000, the maximum allowable 179 = 24,000 (210,000 200,000) = 14,000 ADR 12 years " 7 year recovery period Basis = 150,000 - 14,000 = 136,000
188
Year 1 2 3 4 5 6 7 8
Chapter 12 Depreciation
Depreciation $19,434.40 33,306.40 23,786.40 16,986.40 12,144.80 12,131.20 12,144.80 6,065.60
Year 0 1 2 3
Taxes
ATCF
The tool must be depreciated over 3 years according to the sum of the years digits method. The tax rate is 35%. (a) Fill in the four columns in the table. (b) What is the internal rate of return after tax? Solution a) SOYD Depreciation 150 100 50 Taxable Income -50 +50 +150
Year 0 1 2 3
b) NPW = -300 + 117.50(P/A, i%, 3) + 15(P/G, i%, 3) = 0 By trial and error. Try i = 12% Try i = 15% NPW = $15.55 NPW = -$ 0.68
189
190
13-2 A company, whose earnings put them in the 35% marginal tax bracket, is considering purchasing a piece of equipment for $25,000. The equipment will be depreciated using the straight line method over a 4 year useful life to a salvage value of $5,000. It is estimated that the equipment will increase the companys earnings by $8,000 for each of the 4 years. Should the equipment be purchased? Use an interest rate of 10%. Solution Year 0 1 2 3 4 BTCF -25,000 8,000 8,000 8,000 8,000 5,000 Depreciation 5,000 5,000 5,000 5,000 T.I 3,000 3,000 3,000 3,000 Taxes 1,050 1,050 1,050 1,050 ATCF -25,000 6,950 6,950 6,950 6,950 5,000
StL Depreciation = (25,000 - 5,000) = $5,000 Find net present worth: if > 0 than good deal NPV = -25,000 + 6,950(P/A, 10%,4) + 5,000(P/F, 10%, 4) = $446.50 Purchase equipment
13-3 By purchasing a truck for $30,000, a large profitable company in the 34% income tax bracket was able to save $8000 during year 1, $7000 during year 2, $6000 during year 3, $5000 during year 4, and $4000 during year 5. The company depreciated the truck using the sum-of-years-digits depreciation method over its four year depreciable life, while assuming a zero salvage value for depreciation purpose. The company wants to sell the truck at the end of year 5. What resale value will yield a 12% after-tax rate of return for the company? Solution SOYD = Year 1 2 3 4
4 (5) 2
= 10 SOYD Depreciation 4/10(30,000 - 0) = 12,000 3/10(30,000 - 0) = 9,000 2/10(30,000 - 0) = 6,000 1/10(30,000 - 0) = 3,000
191
Solve for Resale (R): 30,000 = 9,360(P/A, 12%, 5) - 1,680(P/G, 12%, 5) + .66R(P/F, 12%, 5) 30,000 = 22,995.84 + .374484R R = $18,703.50
13-4 A company has purchased a major piece of equipment, which has a useful life of 20 years. An analyst is trying to decide on a maintenance program and has narrowed the alternatives to two. Alternative A is to perform $1000 of major maintenance every year. Alternative B is to perform $5000 of major maintenance only every fourth year. In either case, maintenance will be performed during the last year so that the equipment can be sold for an established $10,000. If the MARR is 18%, which maintenance plan should be chosen? The analyst computed the solution as: Equivalent Annual CostA = $1000 Equivalent Annual CostB = $5000 (A/F, 18%, 4) = $958.50 Therefore choose Alternative B. Is it possible the decision would change if income taxes were considered? Why or Why not? Solution No. Both cash flow, which distinguish between the alternative, would be reduced by the same percentage (i.e. taxes) so EACA > EACB would still be true. If we assume a 45% tax rate, for example, the computations as follows:
192
Alternative B
Year BTCF T.I. 1-3 0 0 4 -5,000 -5,000 EACB = 2,750(A/F, 18%, 4) = 527.20
Taxes 0 -2,250*
ATCF 0 -2,750
13-5 A large company must build a bridge to have access to land for expansion of its manufacturing plant. The bridge could be fabricated of normal steel for an initial cost of $30,000 and should last 15 years. Maintenance will cost $1000/year. If more corrosion resistant steel were used, the annual maintenance cost would be only $100/year, although the life would be the same. In 15 years there will be no salvage value for either bridge. The company pays a combined state and federal income tax rate of 48% and uses straight line depreciation. If the minimum attractive after tax rate of return is 12%, what is the maximum amount that should be spent on the corrosion resistant bridge? Solution Steel Year 0 1-15 BTCF -30,000 -1,000 Depreciation 2,000 T.I. -3,000 Taxes -1,440 ATCF -30,000 +440
Corrosion Resistant Steel Year 0 1-15 BTCF -P -100 Depreciation P/15 T.I. -100 - P/15 Taxes (-48 - .032P) ATCF -P -52 + .032P
NPWA = NPWB for breakeven 440(P/A, 12%, 15) - 30,000 = (-52 + .032P)(P/A, 12%, 15) - P -27,003 = -354 + .0281P - P P = $34,078
193
13-6 An unmarried, recent engineering graduate earned $52,000 during the previous tax year. The engineer claims one $3000 exemption and establishes itemized deductions of $5,200. If single people are automatically allowed $4,700 of deductions, what is the students federal income tax? Tax Rate: Tax Rate 10% 15% 27% Solution Adjusted Gross Income Exemption Itemized deductions (5,200 - 4,700) Taxable Income $52,000 - 3,000 - 500 $ 48,500 Income < $6,000 $6,000 to $27,950 $27,950 to $67,700
13-7 An asset with five year MACRS* life will be purchased for $10,000. It will produce net annual benefits of $2000 per year for six years, after which it will have a net salvage value of zero and will be retired. The companys incremental tax is 34%. Calculate the after tax cash flows. *The annual percentages to use are 20, 32, 19.20, 11.52, 11.52, and 5.76 for years 1 through 6. Solution Year 0 1 2 3 4 5 6 BTCF -10,000 2,000 2,000 2,000 2,000 2,000 2,000 Depreciation 2,000 3,200 1,920 1,152 1,152 576 T.I. 0 -1,200 80 848 848 1,424 Taxes 0 -408.00 +27.20 +288.32 +288.32 +484.16 ATCF -10,000.00 2,000.00 2,408.00 1,972.80 1,711.68 1,711.68 1,515.84
13-8 A state tax of 10% is deductible from the income taxed by the federal government (Internal Revenue Service). The federal tax is 34%. What is the combined effective tax rate?
194
Solution
13-9 For engineering economic analysis a corporation uses an incremental state income tax rate of 7.4% and an incremental federal rate of 34%. Calculate the effective tax rate. Solution Effective rate = S + (1 - S)F = .074 + (1 - .074)(.34) = .074 + (.926).34 = .074 + .3148 = .3888 = 38.88%
13-10 A company bought an asset at the beginning of 2001 for $100,000. The company now has an offer to sell the asset for $60,000 at the end of 2002. For each of the depreciation methods shown below determine the capital loss or recaptured depreciation that would be realized for 2002. Depreciation Method SL SOYD DDB 150%DB Depreciable Life 10 years 5 4 15 Salvage Value* $ 1,000 25,000 0 0 Recaptured Depreciation Capital Loss
*This was assumed for depreciation purposes Solution Depreciation Method SL SOYD DDB 150%DB Depreciable Life 10 years 5 4 15 Salvage Value* $ 1,000 25,000 0 0 Recaptured Depreciation $ 5,000 35,000 21,000 Capital Loss $20,200
195
Book Value = 100,000 - 75,000 = 25,0000 Recaptured Depreciation = 60,000 - 25,000 = $35,000 150%DB: Depreciation Year 1 = Depreciation Year 2 = Book Value Capital Loss
1.5 15 1.5 15
(100,000) = 10,000 (100,000 - 10,000) = 9,000 = 100,000 - 19,000 = 81,000 = 81,000 - 60,000 = $35,000
13-11 A corporations tax rate is 34%. An outlay of $35,000 is being considered for a new asset. Estimated annual receipts are $20,000 and annual disbursements $10,000. The useful life of the asset is 5 years and it has no salvage value. (a) What is the prospective internal rate of return (IRR) before income tax? (b) What is the prospective internal rate of return (IRR) after taxes, assuming straight line depreciation for writing off the asset for tax purpose? Solution DC(SL) =
1 N
(P S) =
25,000# 0 5
= 7,000/year.
196
a)
= 3.500
= 3.898
13-12 A large and profitable company, in the 34% income tax bracket, is considering the purchase of a new piece of machinery that will yield benefits of $10,00 for year 1, $15,000 for year 2, $20,000 for year 3, $20,000 for year 4, and $20,000 for year 5. The machinery is to be depreciated using the modified accelerated cost recovery system (MACRS) with a three year recovery period. The percentages are 33.33, 44.45, 14.81, 8.41, respectively, for years 1, 2, 3, and 4. The company believes the machinery can be sold at the end of five years of use for 25% of the original purchase price. What is the maximum purchase cost the company can pay if it requires a 12% after-tax rate of return?
197
ATCF -P 6,600 + .1133P 9,900 + .1511P 13,200 + .0504P 13,200 + .0286P 13,200 .165P
P = 6,600(P/A 12%, 3) + 3,300(P/G, 12%, 3) + 13,200(P/A, 12%, 2)(P/F, 12%, 3) + .1133P(P/F, 12%, 1) + .1511P(P/F, 12%, 2) + .0504P(P/F, 12%, 3) + .0285P(P/F, 12%, 4) + .165P(P/F, 12%, 5) P = $61,926.52
13-13 A manufacturing firm purchases a machine in January for 100,000. The machine has an estimated useful life of 5 years, with an estimated salvage value of $20,000. The use of the machine should generate $40,000 before-tax profit each year over its 5-year useful life. The firm pays combined taxes at the 40% and uses sum of the years digits depreciation. PART 1 Year 0 1 2 3 4 5 PART PART 2 3. Complete the following table. BTCF Depreciation T.I. Taxes ATCF
Does the sum of digits depreciation represent a cash flow? Calculate the before-tax rate of return and the after-tax rate of return.
198
Solution
PART 1: Before Tax Cash Flow -In January you must pay $100,000. At the end of the first year, and in each subsequent year you realize a cash income of $40,000. This income less depreciation allowance is taxable. In year 5 you also realize $20,000 from salvage of the equipment. This amount is not taxable as it represents a capital expense that was never allocated as depreciation. Sum of Years Digits Depreciation -Year 1 = Year 2 = Year 3 = Year 4 = Year 5 =
5 15 4 15 3 15 2 15 1 15
Year 0 1 2 3 4 5 5 PART 2:
The sum of digits depreciation is a bookkeeping allocation of capital expense for purposes of computing taxable income. In itself it dose not represent a cash flow.
199
By interpolation: IRR = 31.5% After Tax Rate of Return -0 = -100,000 + 34,667(P/A, i%, n) - 2,133(P/G, i%, n) + 20,000(P/F, i%, n) Try 20%* NPW = +1,263 Try 25% NPW = -9,193 By interpolation: IRR = 20.60% *Caution should be exercised when interpolating to find an interest rate. Linear interpolation is being imposed on a curvilinear function. Therefore the solution is approximate. A maximum range over which to interpolate and achieve generally good results is usually considered to be three percentage points. Often times the interest tables you have at your disposal will force you to use a larger range. This is the case in this problem.
13-14 PARC, a large profitable firm, has an opportunity to expand one of its production facilities at a cost of $375,000. The equipment is expected to have an economic life of 10 years and to have a resale value of $25,000 after 10 years of use. If the expansion is undertaken, PARC expects that their income will increase by $60,000 for year 1, and then increase by $5000 each year through year 10 ($65,000 for year 2, $70,000 for year 3, etc.). The annual operating cost is expected to be $5000 for the first year and to increase by 250 per year ($5250 year 2, $5500 for year 3, etc.). If the equipment is purchased, PARC will depreciate it using straight line to a zero salvage value at the end of year 8 for tax purposes. Since PARC is a large and profitable firm their tax rate is 34%. If PARCs minimum attractive rate of return (MARR) is 15%, should they undertake this expansion?
200
Solution
Year 1 2 3 4 5 6 7 8 9 10 Year 0 1 2 3 4 5 6 7 8 9 10 10
Income 60,000 65,000 70,000 75,000 80,000 85,000 90,000 95,000 100,000 105,000 BTCF -375,000 55,000 59,750 64,500 69,250 74,000 78,750 83,500 88,250 93,000 97,750 25,000
Cost 5,000 5,250 5,500 5,750 6,000 6,250 6,500 6,750 7,000 7,250 Depreciation 46,875 46,875 46,875 46,875 46,875 46,875 46,875 46,875 ----
BTCF 55,000 59,750 64,500 69,250 74,000 78,750 83,500 88,250 93,000 97,750 T.I. 8,125 12,875 17,625 22,375 27,125 31,875 36,625 41,375 93,000 97,750 25,000 Taxes 2,762.50 4,377.50 5,992.50 7,607.50 9,222.50 10,837.50 12,452.50 14,067.50 31,620.00 31,620.00 8,500.00 ATCF -375,000.00 52,237.50 55,372.50 58,507.50 61,642.50 64,777.50 67,912.50 71,047.50 74,182.50 61,380.00 61,380.00 16,500.00
NPW = -375,000 + [52,237.50(P/A, 15%, 8) + 3,135(P/G, 15%, 8)] + 61,380(P/F, 15%, 9) + 77,880(P/F, 15%, 10) NPW = -$64,780.13 PARC should not undertake the expansion.
13-15 A young man bought a one-year savings certificate for $10,000, which pays 15%. Before he buys the certificate he has a taxable income that puts him in the 27% incremental income tax rate, what is his after tax rate of return on this investment?
201
13-16 A project can be summarized by the data given in the table below. The company uses straight line depreciation, pays an incremental income tax rate of 30% and requires an after-tax rate of return of 12%. First Cost Annual Revenues Annual Costs Salvage Value Useful Life $75,000 26,000 13,500 15,000 30 years
(a) Using Net Present Worth, determine whether the project should be undertaken. (b) If the company used Sum-Of-Years-Digits depreciation, is it possible the decision would change? (No computations needed.) Solution a) Year 0 1-30 30 BTCF -75,000 12,500 15,000 Depreciation 2,000 -T.I. 10,500 -Taxes 3,150 -ATCF -75,000 9,350 15,000
NPW = -75,000 + 9,350(P/A, 12%, 30) + 15,000(P/F, 12%, 30) = $815 Yes, take project since NPW > 0 b) No. Although total depreciation is the same, SOYD is larger in the early years when it is worth more. Therefore the NPW would increase with SOYD making the project even more desirable.
13-17 A corporation expects to receive $32,000 each year for 15 years if a particular project is undertaken. There will be an initial investment of $150,000. The expenses associated with the project are expected to be $7530 per year. Assume straight line depreciation, a 15-year useful life and no salvage value. Use a combined state and federal 48% income tax rate and determine the project after-tax rate of return.
202
Solution
Year 0 1-30
Depreciation 10,000
T.I. 14,470
Taxes 6,946
Take the ATCF and compute the rate of return at which PWCOSTS equals PWBENEFITS. 150,000 = 17,524(P/A, i%, 15) (P/A, i%, 15) = 150,000 = 8.559
17 ,524
13-18 A project will require the investment of $108,000 in capital equipment (SOYD with a depreciable life of eight years and zero salvage value) and $25,000 in other non-depreciable materials that will be purchased during the first year. The annual project income (net) is projected to be $28,000. At the end of the eight years, the project will be discontinued and the equipment sold for $24,000. Assuming a tax rate of 28% and a MARR of 10%, should the project be undertaken? Solution Cash expenses are multiplied by (1 - tax rate) Incomes are multiplied by (1 - tax rate) Depreciation is multiplied by tax rate Recaptured depreciation is multiplied by (1 - tax rate) Year 0 1 1-8 1-8 8
First Cost Other Costs 25,000(P/F, 10%, 1)(.72) Depr. [24,000(P/A, 10%, 8) - 3,000(P/G, 10%, 8)](.28) Income 28,000(P/A, 10%, 8) Recaptured Depr. 24,000(P/F, 10%, 8)(.72) NPW
203
13-19 The Salsaz-Hot manufacturing company must replace a machine used to crush tomatoes for its salsa. The industrial engineer favors a machine called the Crusher. Information concerning the machine is presented below. First Cost Annual Operating Costs Annual Insurance Cost* Annual Productivity Savings
*Payable at the beginning of each year
Depreciable Salvage Value Actual Salvage Value Depreciable Life Actual Useful Life Depreciation Method
Property taxes equal to 5% of the first cost are payable at the end of each year. Relevant financial information for Salsaz-Hot: Marginal Tax Rate MARR Determine net present worth. Solution Cash expenses are multiplied by (1 - tax rate) Incomes are multiplied by (1 - tax rate) Depreciation is multiplied by tax rate Capital recovery is not taxable therefore multiplied by 1 Recaptured depreciation is multiplied by (1 - tax rate)
34% 10%
First Cost Net Savings 13,000(P/A, 10%, 10)(.66) Property Taxes .05(95,000)(P/A, 10%, 10)(.66) Insurance [1,750 + 1,750(P/A, 10%, 9)](.66) Depr. 14,167(P/A, 10%, 6)(.34) Capital Recovery 10,000(P/F, 10%, 10) Recaptured Depr. 4,000(P/F, 10%, 10)(.66) NPW
204
13-19 Momma Mias Pizza must replace its current pizza baking oven. The two best alternatives are presented below. Crispy Cruster $24,000 9,000 6,000 6,000 3 yrs 5 yrs SL Easy Baker $33,000 6,000 5,000 8,000 4 yrs 5 yrs SOYD
Initial Cost Annual Costs Depreciable Salvage Value Actual Salvage Value Depreciable Life Actual Useful Life Depreciation Method
Assume Momma pays taxes at the 34% rate and has a MARR of 8%. Which oven should be should be chosen? Solution CC Year 0 1-5 1-3 5
First Cost Annual Costs 9,000(P/A, 8%, 5)(.66) Depr. 6,000(P/A, 8%, 3)(.34) Capital Recovery 6,000(P/F, 8%, 5) NPW
First Cost Annual Costs 6,000(P/A, 8%, 5)(.66) Depr. [11,200(P/A, 8%, 4) - 2,800(P/G, 8%, 4)](.34) Capital Recovery 5,000(P/F, 8%, 5) Recaptured Depreciation 3,000(P/F, 8%, 5)(.66) NPW
205
13-19 A heat exchanger purchased by Hot Spot Manufacturing cost $24,000. The exchanger will save $4,500 in each of the next 10 years. Hot Spot will use SOYD depreciation over a six year depreciable life. The declared salvage value is $3,000. It is expected the exchanger can actually be sold for the declared value. Hot Spot pays taxes at a combined rate of 42% and has a MARR of 8%. Was the purchased justified? Solution Year 0 1-10 1-6 10
First Cost Annual Savings 4,500(P/A, 8%, 10)(.58) Depr. [6,000(P/A, 8%, 6) - 1,000(P/G, 8%, 6)](.42) Capital Recovery 3,000(P/F, 8%, 10) NPW
13-20 Pinion Potato Chip Inc., must purchase new potato peeling equipment for its Union City, Tennessee plant. The plant engineer, Abby Wheeler, has determined that there are three possible set-ups that could be purchased. Relevant data are presented below. Naked Peel $45,000 6,000 12% of FC 6 years 6,500 Skinner $52,000 5,000 5,500 6 years 5,500 Peel-O-Matic $76,000 5,000 10,000 6 years 12,000
First Cost Annual Costs Declared Salvage Value Useful Life Actual Salvage Value Part A
All assets are depreciated using the SL method. Determine which set-up should be chosen if P2C Inc. has a MARR of 10% and pays taxes at the 34% marginal rate. Due to economic considerations P2C Inc. must eliminate from consideration the Peel-OMatic set-up and because of a change in the tax laws they must use MACRS depreciation. If all other information concerning the other two alternatives remains the same, which should be chosen?
Part B
206
Solution Part A NP
First Cost Annual Costs 6,000(P/A, 10%, 6)(.66) Depr. 6,600(P/A, 10%, 6)(.34) Capital Recovery 5,400(P/F, 10%, 6) Recaptured Depr. 1,100(P/F, 10%, 6)(.66) NPW
First Cost Annual Costs 5,000(P/A, 10%, 6)(.66) Depr. 7,750(P/A, 10%, 6)(.34) Capital Recovery 5,500(P/F, 10%, 6) NPW
First Cost Annual Costs 5,000(P/A, 10%, 6)(.66) Depr. 11,000(P/A, 10%, 6)(.34) Capital Recovery 10,000(P/F, 10%, 10) Recaptured Depr. 2,000(P/F, 10%, 10)(.66) NPW
207
First Cost Annual Costs 6,000(P/A, 10%, 6)(.66) Depr. (.3333)(45,000)(P/F, 10%, 1)(.34) Depr. (.4445)(45,000)(P/F, 10%, 2)(.34) Depr. (.1481)(45,000)(P/F, 10%, 3)(.34) Depr. (.0741)(45,000)(P/F, 10%, 4)(.34) Recaptured Depr. 6,500(P/F, 10%, 6)(.66) NPW
S Year 0 1-6 1 2 3 4 6
First Cost Annual Costs 5,000(P/A, 10%, 6)(.66) Depr. (.3333)(52,000)(P/F, 10%, 1)(.34) Depr. (.4445)(52,000)(P/F, 10%, 2)(.34) Depr. (.1481)(52,000)(P/F, 10%, 3)(.34) Depr. (.0741)(52,000)(P/F, 10%, 4)(.34) Recaptured Depr. 5,500(P/F, 10%, 6)(.66) NPW
If interest is 12%, which alternative should the engineer select? Assume no salvage value. Use incremental benefit/cost analysis. Solution PWCOST PWBENEFITS B/C B-A PWCOST !Annual Benefit = +2,590 PWBENEFITS 2,590(P/A, 12%, 10) = 14,634 -10,000 A -15,000 5,310(P/A, 12%, 10) = 30,000 30,000/15,000 = 2 B -25,000 7,900(P/A, 12%, 10) = 44,635 44,635/25,000 = 1.79
!B/C = 14,634/10,000 = 1.46 1.46 > 1 Therefore choose B, higher cost alternative
209
210
14-2 With interest at 10 %, what is the benefit-cost ratio for this government project? Initial cost Additional costs at end of years 1 & 2 Benefits at end of years 1 & 2 Annual benefits at end of years 3 - 10 Solution 90K
0 1 2 3 4 5 6 7 8 9 10
90K
90K 90K
90K
90K
90K
90K
14-3 A van is bought for $15,000 in Year 0. It generates revenues of $25,000 per year and expenses of $20,000 per year in each of the Years 1 thru 5. Its salvage value is zero after 5 years. Ignoring income taxes, compute the benefit/cost ratio if i = 10%. Solution PWB = 25,000(P/A, 10%, 5) = $94,775 PWC = 20,000 (P/A, 10%, 5) + 15,000 = $90,820
PWB 94,775 B Ratio " " " 1.0435 C PWC 90,820
211
14-4 A tax-exempt municipality is considering the construction of an impoundment for city water supplies. Two different sites have been selected as technically, politically, socially, and financially feasible. The city council has asked you to do a benefit-cost analysis of the alternatives and recommend the best site. The city uses a 6% interest rate in all analysis of this type. Year Rattlesnake Canyon 0 -$15,000,000 1 - 75 +2,000,000 Which should you recommend? Solution Rattlesnake
PW of Benefits PW of Cost
Blue Basin
3 x 106 (P/A, 6%, 75) 27 x 106
B C
ratio
BB - RC -12,000,000 +1,000,000 =
1 x 106 (P/A, 6%, 75) 12 x 106
= 1.37 > 1
14-5 A city engineer is deciding which of two bids for a new computer to accept. Using benefit-cost analysis, which alternative should be selected if the interest rate is 10% per year? Computer A B Cost $48,000 40,000 Annual Benefits $13,000 12,000 Salvage $0 0 Useful life 6 years 6 years
212
Solution
Alternative A: PWB = 13,000(P/A, 10%, 6) = $56,615. PWC = 48,000 B/C = 56,615/48,000 = 1.179 (OK) Alternative B PWB = 12,000(P/A, 10%, 6) = $52,260 PWC = 40,000 B/C = 52,260 / 40,000 = 1.3065 (OK) Incremental Analysis
!B 56,516 $ 52 ,260 4 ,355 " " " .544 !C 48,000 $ 40,000 8,000
!B < 1.0 !C
14-6 Given four mutually exclusive alternatives, each with a useful life of 20 years and no salvage value, have been presented to the city council of Anytown USA. Which alternative should be selected? A B C D PW of Costs $4,000 $9,000 $6,000 $2,000 PW of Benefits 6,480 11,250 5,700 4,700 Solution B/C C < 1 # eliminate (rearrange) PW of Cost PW of Benefits !B !C !B/!C D 2,000 4,700 A-D 1,780 2,000 .89. A 4,000 6,480 B-A 4,770 5,000 95 B 9,000 11,250 A 1.62 B 2.35 C 0.95 D 2.35
! 1.0
213
14-7 The state engineer estimates that the cost of a canal is $680 million. The legislative analyst estimates the equivalent annual cost of the investment for the canal to be $20.4 million. If the analyst expects the canal to last indefinitely, what interest rate is he using to compute the equivalent annual cost (EAC)? If the canal lasts only 50 years, what interest rate will the analyst be assuming if he believes the EAC to be the same $20.4 million? Solution (a) A = P(A/P, i%, n) For n % &, (A/P, i%, n) = i A=Pxi i = A/P = (b)
20.4 680
= .03
or
i = 3%
= .03
i = 1.75%
14-8 The local city recorder is deciding between two different phone answering systems for her office. The information concerning the two machines is presented below. Machine X Y Cost Annual Savings $1,000 $300 1,200 325 Useful Life 5 years 5 years Salvage $0 0
With an assumed interest rate of 12%, which system would you recommend? Solution Machine X: PWC = $1,000 PWB = $300(P/A, 12%, 5) = $1,081.5
B/C ratio = 1.0815/1,000 = 1.0815 Machine Y: PWC = $1,200 PWB = $325(P/A, 12%, 5) = $1,171.63 # Select machine X
214
14-9 The city council of Arson, Michigan is debating whether to buy a new fire truck to increase protection for the city. The financial analyst has prepared the following data: First cost Annual Maintenance Useful Life Salvage value Annual Reduction in fire damage Truck A $50,000 5,000 7 years 6,000 20,000 Truck B $60,000 4,000 7 years 6,000 21,000
(a) Using the modified B/C ratio method determine whether the city should buy a new truck, and which one to buy if it will be paid for with money borrowed at an interest rate 7%. (b) How would the decision be affected if inflation were considered? Assume maintenance cost, salvage and fire damage are responsive to inflation. (c) Is it possible would change if the interest rate were lower? Why or why not? Solution (a) In the modified B/C ratio, all annual cash flows belong in the numerator while first cost and salvage belong in the denominator. Either present or uniform equivalent methods may be used to relate cash flows.
20,000 $ 5,000 ' B* " 172 . (- 1) ) , " ( C + A 50,000(A / P, 7% , 7) - 5,000(A / F, 7% , 7) (21,000 $ 20,000) $ (4,000 $ 5,000) ' B* " " 108 . (- 1) ) , ( C + B-A 10,000(A / P, 7% , 7)
#A is acceptable
#B is better than A
Truck B should be purchased (b) Since both future cost (maintenance) and benefits (reduced damage and salvage) are responsive to inflation, the decisions are not affected by inflation. (c) No. Lower i (MARR) favors higher investment cost projects and truck B is already the highest cost alternative.
215
14-10 The Tennessee department of Transportation (TDOT) is considering building its first tolledbypass around the town of Greenfield, Tennessee. The initial cost of the bypass is estimated to be $5.7 million. The installation of the tollbooth along the bypass is estimated to be about $1.3 million. The annual maintenance cost of the bypass is to be $85,000 each year while the annual maintenance cost of the tollbooth is to be $65,000 each year. Due to the construction of the bypass, Greenfield is expected to receive estimated tax savings of $225,000 per year. In addition, TDOT has projected savings of $100,000 each year. Once the tollbooth is constructed, each car that passes through the toll is expected to pay a fare of $0.90. The city estimates that there will be a total of 500,000 cars using the bypass each year. Other relevant data is listed below. Resurfacing cost (every 7 years) Shoulder grading/rework 4% of bypass initial cost 90% of resurfacing cost
If the state uses an interest rate of 7%, should the toll-bypass be constructed? What is the projects total yearly worth? Assume perpetual life. Solution: A. Use benefit/cost analysis. (Use AW analysis) Benefits Greenfield Tax Annual Revenue Bypass Construction Annual Savings Tollbooth Revenue $0.90(500,000) $225,000.00 $100,000.00 $450,000.00 AWB = $ 775,000.00
Costs Toll First Cost $1,300,000(A/P, 7%, &) $ 91,000.00 Bypass First Cost $5,700,000(A/P, 7%, &) $399,000.00 Toll Maintenance $ 85,000.00 Bypass Maintenance $ 65,000.00 Resurfacing 0.04($5,700,000(A/P, 7%, 7)) $ 26,536.80 Shoulder Grading/Rework 0.90(0.04($5,700,000(A/P, 7%, 7))) $ 23,721.12 AWC = ($690,257.92)
B AWB $775,000.00 " " " 1123 . C AWC $690,257.92
216
14-11 The town of Oakville is evaluating a proposal that it erect and operate a structure for parking in the downtown area. Numerous design proposals were considered. Data for the two best proposals is presented below. Cost of site and construction Annual revenue from parking fees Annual maintenance cost Service life Design A $2,220,000 765,000 410,000 20 years Design B $1,850,000 575,000 295,000 20 years
At the end of each five year period the parking facility will require a maintenance overhaul. The cost for design A is estimated to be $650,000 and for design B, $375,000. At the end of the service life the facility will be torn down and the land sold. Demolition costs and proceeds from the sale of the land are presented below. Demolition costs Proceeds from sale of land $530,000 530,000 $550,000 530,000
If the citys interest rate is 10%, determine the B/C ratio of each design proposal. Solution Design A PWB = 765,000(P/A, 10%, 20) = $6,513,210 PWC = 2,220,000 + 410,000(P/A, 10%, 20) + 650,000(P/F, 10%, 5,10, and 15) = $6,500,510
B $6,513,210 " " 1001 . C $6,500,510
(Note: Demolition and proceeds from sale net to $0) Design B PWB = 575,000 + 530,000(P/F, 10%, 20) = $4,974,308 PWC = 1,850,000 + 295,000(P/A, 10%, 20) + 375,000(P/F, 10%, 5, 10, and 15) + 550,000(P/F, 10%, 20) = $4,910,535
B $4,974,308 " " 1013 . C $4,910,535
217
14-12 A new Water Treatment Plant will cost the city of Frogjump $2,000,000 dollars to build and $150,000 per year to operate for its 30-year life. At the end of 30 yrs, the salvage value will be 0. Due to more efficient operation of the water plant, it is expected to lower the cost of utility bills for each customer $50 per year. There are 6,000 customers that are billed in Frogjump. The plant will reduce air quality by an estimated $5 per resident per year. The population of Frogjump is currently 18,000 and is expected to remain relatively constant over the life of the plant. If 3% is used for the evaluation of public works projects, should the Water Treatment Plant be built? Solution First Cost: $2,000,000 Annual Operating Cost: $150,000/yr Additional Annual Cost: $5/year x 18,000 residents = $90,000/year Annual Benefits: $50/year x 6,000 customers = $300,000/year Life: 30 yrs Interest: 3% Using B/C ratio AWB/AWC should be greater than or equal to 1 AWB = $300,000 AWC = $2,000,000(A/P, 3%, 30) = $342,000
B AWB $300,000 " " " 0.8772 C AWC $342,000
14-13 The expansion of the hotel and conference center at Wicker Valley State Park is under study. The initial investment and annual operating benefits and cost are very different due to the differing magnitudes of the project under consideration. These can be summarized as follows: Alternative A Investment cost $180,000 Annual operating costs 16,000 Annual benefits 53,000 Alternative B $100,000 12,000 35,000 Alternative C $280,000 28,000 77,000
Using a MARR of 10%, which alternative should be selected for a ten-year useful life analysis period? Use benefit/cost ratio analysis.
218
Solution 1.
#All economically attractive 2. Incremental B/C Analysis Cost B < Cost A < Cost C A-B !Benefits = 53,000 - 35,000 = 18,000 !Costs = (180,000 - 1000,000)(A/P, 10%, 10) + (16,000 - 12,000) = 17,016
18,000 ' B* " " 106 . (- 1) ) , ( C + A-B 17,016
#Keep A
C-A !Benefits = 77,000 - 53,000 = 24,000 !Costs = (280,000 - 180,000)(A/P, 10%, 10) + (28,000 - 16,000) = 28,270
24,000 ' B* " " 0.85 (< 1) ) , ( C + C-A 28,270
#Keep A
Select Alternative A. (If we consider the operating costs as a reduction to the annual benefits a different numerical value for the B/C ratio might be found. The decision of which alternative is best will, however, be the same.)
Chapter 15 Inflation
15-1 A European investor lives near to one of his countrys borders. In Country A (Where he lives), an 8% interest rate is offered in banks, and the inflation rate is 3%. Country B, on the other hand, has an inflation rate of 23%, and their banks are offering 26% interest on deposits. (a) What is the real or effective interest rate that this person gets when investing in his Country? (b) This investor believes that the currency of Country B will not change in its value relative to the value of the currency of Country A during this year. In which country would he get a larger effective interest rate? (c) Suppose now that he invests in a bank in Country B, and that his prediction was wrong. The currency of Country B was devaluated 20% with respect to the exchange value of Country As currency. What is the effective interest rate that he obtained? Solution (a) i' = ? if i = 80%, f = 3% i = i' + f + i'F .08 = i' + .03 + i'(.03) i' = 0.0485 = 4.85% b) if investment in Country A: i'A = 0.0485 if investment in Country B: iB = 26%, fA = 3% (note that he lives in Country A. Inflation of Country B does not affect him directly) i'B =
i B ! fA 1" fA
= 0.2233 = 22.33%
219
220
Chapter 15 Inflation
The amount collected at end of 1 year (measured in currency A) =
(10 . ! 0.2) ! # "# $
Due to the devaluation
(126 . ) ! " $
= 1.008X
= 0.008
but during the year the inflation in Country A (where he lives) was 3%, therefore i = 0.008 f = 0.03 i' = ? i' =
0.008 ! 0.03 1" 0.03
= -0.02136
15-2 The first sewage treatment plant for Athens, Georgia cost about $2 million in 1964. The utilized capacity of the plant was 5 million gallons/day (mgd). Using the commonly accepted value of 135 gallons/person/day of sewage flow, find the cost per person for the plant. Adjust the cost to 1984 dollars with inflation at 6%. What is the annual capital expense per person if the useful life is 30 years and the value of money is 100%. Solution Population equivalents = 5 mgd/135 = 37,037 Cost per capita =
$2,000,000 = $54 37,037
1984$, F = 54(F/P, 6%, 20) = $173.18 Annual Cost, A = 173.18(A/P, 10%, 30) = $18.37
15-3 How much life insurance should a person buy if he wants to leave enough money to his family, so they get $25,000 per year in interest, of consent Year 0 value dollars? The interest rate expected from banks is 11%, while the inflation rate is expected to be 4% per year.
Chapter 15 Inflation
Solution The actual (effective) rate that the family will be getting is i' =
i! f 1" f
221
= 0.0673 = 6.73%
25,000 0.0673
= $371,471
15-4 Property, in the form of unimproved land, is purchased at a cost of $8000 and is held for six years when it is sold for $32,600. An average of $220 each year is paid in property tax and may be treated at an interest of 12%. The long-term capital gain tax is 15% of the long-term capital gain tax is 15% of the long-term capital gain. Inflation during the period is treated as 7% per year. What is the annual rate of return for this investment? Solution Long term gains = 32,600 8,000 = 24,600 Tax on long-term gain = .15 x 24,600 = 3,690 Property tax = 220(F/A, 12%, 6) = 1,785.30 Adjusted FW = 32,600 - 3,690 - 1785.30 = 27,624.70 also FW = 8,000(1 + ieq)6
& 1 " i eq '
% $
1 27,124.70 6 8,000
' 1.2257
1" i '
15-5 The auto of your dreams costs $20,000 today. You have found a way to earn 15% tax free on an auto purchase account. If you expect the cost of your dream auto to increase by 10% per year, how much would you need to deposit in the auto purchase account to provide for the purchase of the auto 5 years from now? Solution
222
Chapter 15 Inflation
Cost of Auto 5 years hence (F) = P(1 + inflation rate)n = 20,000 (1 + 0.10)5 = 32,210 Amount to deposit now to have $32,210 five years hence P = F(P/F, i%, n) = 32,210(P/F, 15%, 5) = $16,014.81
15-6 On January 1, 1975 the National Price Index was 208.5, and on January 1, 1985 it was 516.71. What was the inflation rate, compounded annually, over that 10-year period? If that rate continues to hold for the next 10 years, what National Price Index can be expected on January 1,1995? Solution NPW = 0 -208.5 + 516.71(P/F, if, 10) (P/F, if, 10) =
208.5 516.71
= 0.4035 Trial & Error Solution: Try i = 9%: -208.5 + 516.71(0.4224) = + 9.76 Try i = 10%: -208.5 + 516.71(0.3855) = - 9.31 By interpolation if = 9.51% National Price Index1995 = 516.71(1 + 0.0951)10 = 1,281.69 15-7 A department store offers two options to buy a new color TV which has a price of $440.00. A customer can either pay cash and receive immediately a discount of $49.00 or he can pay for the TV on the installment plan. The installment plan has a nominal rate of 12% compounded biyearly and would require an initial down payment of $44.00 followed by four equal payments (principle and interest) every six months for two years. If for the typical customer the real minimum attractive rate of return is 5%, what is the maximum effective annual inflation rate for the next two years that would make paying cash preferred to paying installments? All figures above are quoted in time zero dollars.
Solution
Chapter 15 Inflation
The monthly payments in nominal dollars if the installment plan was selected would be (-440 + 44)(A/P, 6%, 4) = -$114.28
223
The breakeven inflation rate is that such that NPVBUY = NPVINSTALL or NPVBUY - INSTALL = 0 NPVB - I = ((-440 + 49) + 44) + 114.28(P/A, i1/2, 4) = 0 (P/A, i1/2, 4) = 3.0364 therefore the nominal effective semi-annual MARR would have to be i1/2 = .115. The nominal effective annual rate would be i = (1.115)2 1 = 0.2432 The effective annual inflation rate can now be computed from the formula (1.2432) = (1.05) (1 + f) f = .1840
15-8 An automobile that cost $16,500 in 1998 has an equivalent model four years later in 2002 that cost $19,250. If inflation is considered the cause of the increase, what was the average annual rate of inflation? Solution F = P(1 + if) 19,250 = 16,500(1 + if)
19 ,250 16,500
= (1 + if)
1 + if = (1.167)1/4 = 1.039 if = 3.9% 15-9 A machine has a first cost of $100,000 (in todays dollar) and a salvage value of $20,000 (in then current dollars) at the end of its ten year life. Each year it will eliminate one full-time worker. A worker costs $30,000 (todays dollars) in salary and benefits. Labor costs are expected to escalate at 10% per year. Operating and maintenance costs will be $10,000 per year (todays dollars) and will escalate at 7% per year. Construct a table showing before-tax cash flows in current dollars, and in todays dollars. The inflation rate is 7%.
224
Year 0 1 2 3 4 5 6 7 8 9 10
Chapter 15 Inflation
Savings 33,000 36,300 39,930 43,923 48,315 53,147 58,462 64,308 70,738 77,812 O&M -10,700 -11,449 -12,250 -13,108 -14,026 -15,007 -16,058 -17,182 -18,385 -19,672 Capital -100,000 Total -100,000 22,300 24,851 27,680 30,815 34,290 38,140 42,404 47,126 52,354 78,141 Dollars -100,000 20,841 21,706 22,595 23,509 24,448 25,414 26,407 27,428 28,477 39,723
20,000
15-10 A project has been analyzed assuming 6% inflation and is found to have a monetary internal rate of return (IRR) of 22%. What is the real IRR for the project? Solution Real IRR = (1.22)/(1.06) - 1 = 0.1509 or 15.09%
15-11 A company requires a real MARR of 12%. What monetary MARR should they use if inflation is expected to be 7%? Solution Monetary MARR = (1.12)(1.07) - 1 = 0.1984 or 19.84%
15-12 The real interest rate is 4%. The inflation rate is 8%. What is the apparent interest rate? Solution i = i' + f + i'f = 0.04 + 0.08 + 0.04(0.08) = 12.32%
15-13 A solar energy book gives values for a solar system as follows: Initial cost, $6,500; Initial fuel savings, $500/year; Expected life, 15 years; Value of Money, 10%; Inflation, 12%; and
Chapter 15 Inflation
225
Incremental income tax rate, 25%. If we define the payback condition as the time required for the present worth of the accumulated benefit to equal the accumulated present worth of the system cost, what is the time required to reach the payback condition? Since the income tax benefit is related to the annual interest expense, treat it as a reduction of the annual cost. Solution Annualizing P: A = 6,500(A/P, 10%, 15) = 854.75
1 + iC = (1.10)(1 + 0.25 x 0.10) = 1.1275 PW of costs = 854.75(P/A, 12.75%, 15) = 5,595.82 1 + ieq =
1+ i f ' 112 . 110 .
1" i
= 1.018
The solution strategy is to find the time for the PW of benefits to equal PW of cost. When the combined effect of the two rates on a distributed A amount are opposed then the net effect retains the direction of the longer rate. The inflation rate is greater than the time value of money, which is abnormal. To solve this problem, find the PW of benefit, and to do that we must get FW of the equivalent rate, ieq. Try 10 years: Try 11 years: FW = 500(F/A, 1.8%, 10) = 500 (10.850) = $5,425.06 FW = 500(F/A, 1.8%, 11) = 500 (12.045) = $6,022.72 By interpolation payback = 10.3 years
15-14 Compute the internal rate of return based on constant (Year 0) dollars for the following after-tax cash flow given in current or actual dollars. Inflation is assumed to be 7% per year. (Round to the nearest dollar). After Tax Cash Flow In Actual Dollars -$10,000 3,745 4,007 4,288 4,588
226
Chapter 15 Inflation
Year 1998 (0) 1999 (1) 2000 (2) 2001 (3) 2002 (4) In Constant Dollars -$10,000 3,745(1.07)-1 = 3,500 4,007(1.02)-2 = 3,500 4,288(1.07)-3 = 3,500 4,588(1.07)-4 = 3,500
NPW = 0 at IRR 0 = -10,000 + 3,500(P/A, i%, 4) (P/A, i%, 4) = 10,000/3,500 = 2.857 Searching tables where n = 4, i = IRR = 15%
15-15 The capital cost of a wastewater treatment plant for a small town of 6,000 people was estimated to be about $85/person in 1969. If a modest estimate of the rate of inflation is 5.5% for the period to 1984, what is the per capita capital cost of a treatment plant now? Solution F = P(1+ if)n = 85(1+.055)15 = 85(2.232) = $189.76
15-16 A lot purchased for $4,500 is held for five years and sold for $13,500. the average annual property tax is $45 and may be accounted for at an interest rate of 12%. The income tax on the long term capital gain is at the rate of 15% of the gain. What is the rate of return on the investment if the allowance for inflation is treated at an average annual rate of 7%?
Chapter 15 Inflation
Tax on long term gain = (.15)(9,000) = 1,350 Property tax = 45(F/A, 12%, 5) = 285.89 Adjusted FW = 13,500 - 1,350 - 285.89 = 11,864.12 also FW = 4,500(1 + ieq)5
& 1 " i eq '
227
% $
1 11,864.12 5 4,500
' 1.214
1" i '
15-17 Undeveloped property near the planned site of an interest highway is established to be worth $48,000 in six years when the construction of the highway is completed. Consider a 15% capital gains tax on the gain, an annual property tax of 0.85% of the purchase price, an annual inflation rate of 7%, and an expected return of 15% on the investment. What is the indicated maximum purchase price now? Solution Let X = purchase cost 1 + ieq = (1.15)(1.07) = 1.231 Annual property tax = .0085X FW of property tax = .0085X(F/A, 23.1%, 6) = .0909X Adjusted return = 48,000 - .15(48,000 - X) - .0909X Also = X(1.231)6 = 3.48X Therefore 40,800 + .15 X - .0909X = 3.48X X = $11,927 purchase cost
15-18 Jack purchases a lot for $40,000 cash and plans to sell it after 5 years. What should he sell it for if he wants a 20% before-tax rate of return, after taking the 5% annual inflation rate into account?
228
Solution
Chapter 15 Inflation
15-19 Minor Oil Co. owns several gas wells and is negotiating a 10-year contract to sell the gas from these wells to Major Oil Co. They are negotiating on the price of the gas the first year, $ per thousand cubic feet (KCF), and on the escalation clause, the percentage rate of increase in the price every year thereafter. Minor experts the wells to produce 33,000 KCF the first year and to decline at the rate of 18% every year thereafter. Minor has agreed to spend $500,000 now to lay pipelines from each well to Majors nearby refinery. What should the minimum price be the first year and what should the escalation rate be if Minor wants their revenue each year to remain constant (uniform) over the life of the contract. Assume an end-of-year convention and a minimum attractive rate of return (MARR) of 15%. Solution Required income to earn the 15% MARR on $500,000: EAB = 500,000(A/P, 15%, 10) = = $99,650. First year price = $99,650/33,000KCF = $3.02/KCF Annual production declines to (1 - 0.18) of initial rate each year. Let f = required annual escalation rate Then (1 - 0.18)(1+f) = 1 to keep the revenue constant 1 f = (1! 0 -1 .18) = 0.2195/year
15-20 A solar system costs $6500 initially and qualifies for a federal tax credit (40% of cost, not to exceed $4,000). The cost of money is 10%, and inflation is expected to be 7% during the life of the system. The expected life of the system is 15 years with zero salvage value. The homeowner is in the 40% income tax bracket. The initial fuel saving is estimated at $500 for the first year and will increase in response to inflation. The annual maintenance cost of the system is established at 5% of the annualized cost of the system. What is the time required for the payback condition to be reached for this investment? Solution Adjust initial cost by tax credit: P = .60(6,500) = 3,900 Annualized cost: A = 3,900(A/P, 10%, 15) = 512.85
Chapter 15 Inflation
1 + ic =
110 . (1" .40(.10)) (1.05)
229
= 1.0895
PW of costs = 512.85(P/A, 8.95%, 15) = 512.85(8.086) = 4,146.67 1 + ieq = (1 + i)/(1 + if) = 1.10/1.07 = 1.028 Try 9 years: Try 10 years: PW = 500 (P/A, 2.8%, n) = 500 (7.868) = $3,934.18 PW = 500 (P/A, 2.8%, n) = 500 (8.618) = $4,308.97 By interpolation payback = 9.6 years
15-21 Acme Company is considering the purchase of a new machine for $30,000 with an expected life of 20 years when it is established the salvage value will be zero. An investment tax credit of 7% will be allowed. The incremental tax rate (tax bracket) for the company is 36%, inflation is established to be 7%, and the value of money is 12%. If the annual benefit is established to be $2,500 per year over current production and maintenance costs, what will be the time required for the payback condition to be reached (that is, the point where the present worth of benefits up to payback equals the present worth of costs for the life expectancy of the equipment)? Solution Tax credit reduces initial cost P = 30,000 x .93 = 27,900 Annualize P: A = 27,900(A/P, 12%, 20) = 3735.81 ic combines interest i with income tax credit rate 1 + ic = (1 + i)(1 + .36 x .12) = 1.1684 PW of cost = 3,735.81(P/A, 16.84%, 20) = 3,735.81(5.674) = 21.197 1 + ieq = (1 + i)/(1 + if) = 1.12/1.07 = 1.0467 Try 11 years: Try 12 years: PW = 2,500 (P/A, 4.67%, n) = $21,130 PW = 2,500 (P/A, 4.67%, n) = $22,575 By interpolation payback = 11.05 years
15-22 The net cost of a solar system for a home is $8,000 and it is expected to last 20 years. If the value of money is 10%, inflation is expected to be 8%, and the initial annual fuel saving is $750, what is the time for the payback condition to be reached for the system? Assume the homeowner is in the 30% income tax bracket.
230
Solution
Chapter 15 Inflation
Annualize P: A = 8,000 (A/P, 10%, 20) = 940 1 + ic = (1.10)(1 + .10 x .30) = 1.133 PW of Cost = 940(P/A, 13.3%, 20) = 940(6.900) = 6,486 1 + ieq = (1 + i)/(1 + if) = 1.10/1.08 = 1.0185 Try 9 years: Try 10 years: PW = 940(P/A, 1.85%, n) = $6,171 PW = 940(P/A, 1.85%, n) = $6,790 By interpolation payback = 9.5 years
15-23 A undeveloped percent of land in Clarke County, Georgia was purchased in 1980 for $4,850. The property tax was $8 for the first year and is assumed to have increased by $ 2 per year. The capital gain tax is 13.6% of long term capital gain. Inflation for the period is treated at an 8% annual rate. A 16% rate of return on the investment is desired. What is the indicated sale price in 1985? Solution 1 + ieq = (1.16)(1.08) = 1.2528 FW of property tax = [8 + 2(A/G, 25.28%, 5)] [F/A, 25.28%, 5] = [8+ 2(3.12)] [8.252] = 91.74 Let X = selling price Long term capital gains tax = 0.136(X - 4,850) = .136X - 659.60 Adjusted return = X - [.136X - 659.60 + 91.74] = .864X + 567.86 Also = 4,850(1.2528) = 14,967.54 .864X + 567.86 = 14,967.54 .864X = 14,399.68 X = $16,666.31 selling price 15-24 A company has designed a VLSI circuit and a production system to manufacture it. It is believe that it can sell 100,00 circuits per year if the price in then-current dollars is cut 20% per year (for example, if the unit price in the first year is $100, then the price in years 2 through 5 would be $80, $64, $51.20, and $40.96).
Chapter 15 Inflation
231
The required revenue for the five years is $2,500.00 per year in todays dollars. The real and monetary costs of capital are 8.8% and 16.416% respectively. What should the then-current dollar selling price be in each of the years 1 through 5? Solution Let R be the required revenue in year 1, then the required revenue in years 2 through 5 is .8R, .64R, .512R, and .0496R. Since these are in then-current $, (2,500,000)(P/A, 8.8%, 5) = R(1.16416)-1 + 0.8R(1.16416)-2 + .064R(1.16416)-3 + 0.512R(1.16416)-4 + 0.4096R(1.16416)-5 9,774,800 = 2.32523R R = 4,203,804 or a unit price of $42.04 in year 1 $33.63 in year 2 $26.90 in year 3 $21.52 in year 4 $17.22 in year 5
15-25 Steve Luckee has just been informed that he has won $1 million in a local state lottery; unfortunately the Internal Revenue Service has also been so informed. Due to tax considerations, Steve has a choice in the manner in which he will receive his winnings. He can either receive the entire amount today, in which case his combined federal an, state income tax rate will soar to 74%, or he can receive the money in uniform annual payments of $50,000 over the next 20 years, beginning one year from today, in which case his annual payments will be taxed at a lower rate. Steve expects inflations to be a problem for the next 20 years which will obviously reduce the value of his winnings if he takes the annual payment option. Steve uses a real dollar MARR of 5% in his decisions. (a) If Steve estimates that the inflation rate f would be 6% and his combined federal and state tax rate d would be 40% for the next 20 years, what action should he take? (b) What would be the present worth of the cost of an error in Steves estimates in (a) if the true values eventually prove to be f = 8% and d = 20%? (c) If Steve is certain that 6% f 8% and 20% d 40% for the next 20 years, what is the maximum cost of errors in estimating f and d?
Solution (a) The value of the entire amount is: (1 - .74)($1M) = R$260,000. The real NPV of the payments would be: NPV = (1 - .4)($1M/20)(P/A, (1.05)(1.06) - 1,20) = R$234,288
232
Chapter 15 Inflation
Take the entire amount today, it is worth more. (b) The real NPV of the payments would be: NPV = (1 - .2)(50K)(P/A, (1.05)(1.08) - 1, 20) = R$274,370 which is more than the entire amount today. Steves errors in estimation would cost him 274, 370 - 260,000 = R$14,370. (c) The maximum cost of an error will occur one of two ways, either Steves estimates indicates he should take the entire amount today when he shouldnt or that he take the payments when he shouldnt. If he takes the entire amount when shouldnt, then for f = .06 and d = .2, and the payments are worth NPV = .8(50K)(P/A, (1.05) (1.06) - 1, 20) = R$312,384, and the cost is $52,384. If he takes the payments when he shouldnt, then if f = .08 and d = .4, and the payments are worth .6(50K)(P/A, (1.05)(1.08) - 1, 20) = R$205,777, and the cost is $54,223. The maximum cost is $54,223.
15-26 An investor is considering the purchase of a bond. The bond has a face value of $1000, a coupon rate (interest rate) of i = 6% compounded annually, pays interest once a year, and matures in 8 years. This investors real MARR is 25%. (a) If the investor expects an inflation rate of 4% per year for the next 8 years, how much should he be willing to pay for the bond? (b) If, however, the investor expects a deflation rate of 18% per year for the next 8 years, how much should he be willing to pay for the bond? Solution (a) The net cash flows for bond in nominal dollars are t Net cash flows 0 N$F.C. 1-8 +60 = .06(1,000) 8 + 1,000
If the investor wants to earn a real 25%, but expects inflation at 4%, his nominal MARR must be (1.25)(1.04) - 1 = .30, and therefore NPV = 0 at IRR 0 = -F.C. + 60(P/A, .30, 8) + 1,000(P/F, .30, 8)
Chapter 15 Inflation
F.C. = $250.50 He should be willing to pay no more than $177.
233
(b) If the deflation rate (negative inflation) is 18% and he still expects to earn a real 25% then the real MARR is (1.25)(0.82) - 1 = .025, 0 = -F.C. + 60(P/A, .025, 8) + 1,000(P/F, .025, 8) F.C. = $1,251
15-27 An investment in undeveloped land of $9,000 was held for four years and sold for $21,250. During this time property tax was paid that was, on the average, 0.4% of the purchase price. Inflation in this time period average 7% and the income tax was 15.2% of the long term capital gain. What rate of return was obtained on the investment? Solution Long term capital gains tax = 0.152(21,250 - 9,000) = $1,862 Property. Tax = .004 x 9,000 = $36/year FW of property Tax = 36(F/A, ieq, 4) 1 + ieq =
= 1.2115
(1st estimate)
(2nd estimate)
Check:
9,000(1.284) = $19,191 Property Tax = 197 L.T. Capital Gain Tax = 1,862 $21,250
16-2 In 2000 a new 21-kW power substation was built in Gibson county Tennessee for 1.4 million dollars. Weakley county, a neighboring county, is planning on building a similar though smaller substation in 2003. An 18-kW substation is planned for Weakley county. The inflation rate between 2001 and 2003 has averaged 1.5% per year. If the capacity exponent is .85 for this type of facility what is the estimated cost of construction? Solution Cost of the 21-kW substation in 2003 dollars = 1,400,000(1.015)3 = $1,463,950 Cx = Ck(Sx/Sk)n C21 = C18(18/21).85 = 1,463,950(.8772) = $1,284,177
16-3 The time required to produce the first gizmo is 1500 blips. Determine the time required to produce the 450th gizmo if the learning-curve coefficient is .85. Solution Ti = T1! (ln i/ln 2) T450 = 1500(.85)(ln 450/ln 2) = 358.1 blips
235
236
16-4 Four operations are required to produce a certain product produced by ABC Manufacturing. Using information presented in the table below, determine the labor cost of producing the 1000 piece. Time required for 1st piece 1 hour 15 minutes 2 hours 2 hours 45 minutes 4 hours 10 minutes Learning curve coefficient .90 .82 .98 .74 Labor cost per hour $ 8.50 12.00 7.75 10.50
T1000 = 75(.90)(ln 1000/ln 2) = 26.25 minutes Cost = 26.25/60 x 8.50 = $3.72 Operation 2 T1000 = 120(.82)(ln 1000/ln 2) = 16.61 minutes Cost = 16.61/60 x 12.00 = $3.32 Operation 3 T1000 = 165(.98)(ln 1000/ln 2) = 134.91 minutes Cost = 134.91/60 x 7.75 = $17.43 Operation 4 T1000 = 250(.74)(ln 1000/ln 2) = 12.44 minutes Cost = 12.44/60 x 10.50 = $2.18 Total cost = 3.72 + 3.32 + 17.43 + 2.18 = $26.65
237
16-5 Turbo Tire has developed a new tire that it believes will be very competitive in the market place. It expects that the tire will be a solid seller for the next several years. The marketing department estimates that the upper limit in the demand will be 800,000 tires. Initially the tires will be sold for $56.00 each and after 2 years the price will increase by $8.00 each year until the price reaches $80.00. The cost of production for each tire is estimated to be $32.00 the first year and will increase by $4.00 each year. Assume that a Pearl curve with a = 6,000 and b = 4 applies. What will be the expected demand for the next 5 years? What are the expected revenues (gross and net) for the same period? Solution D = L/(1 + ae-bt) = 800,000/(1 + 6,000e-4t) Year Demand Revenue Cost Net Revenue 1 7,214 $403,984 $230,848 $173,136 2 265,536 14,870,016 9,562,896 5,307,120 3 771,556 49,379,584 30,862,240 18,517,344 4 799,460 57,561,120 35,176,240 22,384,880 5 799,990 63,999,200 38,399,520 25,599,680
16-6 American Petroleum (AP) recently completed construction on a large refinery in Texas. The final construction cost was $17,500,000. The refinery covers a total of 340 acres. The Expansion and Acquisition Department at AP is currently working on plans for a new refinery for the panhandle of Oklahoma. The anticipated size is approximately 260 acres. If the capacity exponent is .67 for this type of facility what is the estimated cost of construction? Solution Cx = Ck(Sx/Sk)n C260 = C340(260/340).67 = 17,500,00(.83549) = $14,621,075
16-7 A new training program at Arid Industries is intended to lower the learning curve coefficient of a certain molding operation that currently costs $95.50/hour. The current coefficient is .87 and the program hopes to lower the coefficient by 10%. Assuming the time to mold the first product is 8 hours. What cost savings can be realized when the 2000th piece is produced if the program is successful?
238
Solution
Ti = T1! (ln i/ln 2) Without the training program: T2000 = 8(.87) (ln 2000/ln 2) = 1.74 hours With the training program: T2000 = 8(.783) (ln 2000/ln 2) = .547 hours Cost savings = (1.74 - .547)(95.50) = $113.93
16-8 The following data is has been provided by XYZ Manufacturing concerning one of their most popular products. Estimate the selling price per unit. Labor Factory overhead Material costs Packing cost Sales commission Profit Solution Labor Cost = Factory overhead = Material cost = Packing Cost = 12.8 x 18.75 92% of labor = $240.00 = 220.80 = 65.10 10% of material costs = 6.51 $532.41 = 12.8 hours at $18.75/hour = 92% of labor = $65.10 = 10% of materials = 10% of selling price = 22% of selling price
Let X be the selling price 0.10X + 0.22X + 532.41 = X 0.68X = 532.41 X = 532.41/0.68 = $782.96
239
240
1
2
7,500 X
3
$0
For X > 0
17-2 The annual income from an apartment house is $20,000. The annual expense is estimated to be $2000. If the apartment house can be bought today for $149,000, what is the breakeven resale price in ten years with 10% considered a suitable interest rate? Solution P = (AINCOME - AEXPENSES)(P/A, i %, n) + FRE-SALE(P/F, i %, n) 149,000 = (20,000 - 2,000)(P/A, 10%, 10) + FRE-SALE (P/F, 10%, 10) 149,000 = 18,000(6.145) + FRE-SALE (.3855) FRE-SALE = $99,584.95 17-3 Oliver Douglas decides to install a fuel storage system for his farm that will save him an estimated 6.5 cents/gallon on his fuel cost. Initial cost of the system is $10,000 and the annual maintenance is a uniform gradient amount of $25. After a period of 10 years the estimated salvage is $3,000. If money is worth 12%, what is the breakeven quantity of fuel? Solution EAC =(10,000 - 3,000)(A/P, 12%, 10) + 3,000(.12) + 25(A/G, 12%, 10) = $1,688.63 EAB = Gallons(.065) = $G(.065) 0 = -1,688.63 + G(.065) G = 25,979 gallons
17-4 A land surveyor just starting in private practice needs a van to carry crew and equipment. He can lease a used van for $3,000 per year, paid at the beginning of each year, in which case maintenance is provided. Alternatively, he can buy a used van for $7,000 and pay for maintenance himself. He expects to keep the van three years at which time he could sell it for $1,500. What is the most he should pay for uniform annual maintenance to make it worthwhile buying the van instead of leasing it, if his MARR is 20%?
241
17-5 The investment in a crane is expected to produce profit from its rental as shown below, over the next six years. Assume the salvage value is zero. Assuming 12% interest, what is the breakeven cost of the crane? Year 1 2 3 4 5 6 Solution PWPROFIT = 15,000(P/A, 12%, 6) - 2,500(P/G, 12%, 6) = $39,340 CostBE = $39,340 17-6 ABC Manufacturing has a before-tax minimum attractive rate of return (MARR) of 12% on new investments. What uniform annual benefit would Investment B have to generate to make it preferable to Investment A? Year 0 1-6 Solution NPW of A = - 60 + 15(P/A, 12%, 6) = 1.665 NPW of B % 1.665 = - 45 + B(P/A, 12%, 6) &' B = 11,351 B > $11,351 per year Investment A - $60,000 +15,000 Investment B - $45,000 ? Profit $15,000 12,500 10,000 7,500 5,000 2,500
242
17-8 Data for two drill presses under consideration by B&R Gears are listed below. Assuming an interest rate of 12%, what salvage value of press B will make the two alternatives equal? A First cost Annual maintenance Salvage value Useful life Solution EAC = P(A/P, i %, n) - S(A/F, i %, n) + Other Costs DRILL PRESS A: EAC = 30,000(A/P, 12%, 6) - 5,000(A/F, 12%, 6) + 1,500 = $8,180 DRILL PRESS B: EAC = 36,000(A/P, 12%, 6) - SV(A/F, 12%, 6) + 2,000 = $10,755.20 - SV(.1232) Setting the two EAC equal 8,180 = 10,755.20 - SV(.1232) SV = $20,903 $30,000 1,500 5,000 6 years B $36,000 2,000 ? 6 years
17-9 Dolphin Inc. trains mine seeking dolphins in a 5-mine tank. They are considering purchasing a new tank. A new tank costs $750,000 and realistic dummy mines cost $250,000. The new tank will allow the company to train 3 dolphins per year and will last 10 years costing $50,000 per year to maintain. How much must Dolphin Inc. receive (per dolphin) from the Navy in order to breakeven if the MARR equals 5%? Solution NPV = -Cost - Cost of Mines - Annual Maintenance(P/A, 5$, 10) + Income(P/A, 5%, 10) = -750,000 - 250,000(5) - 50,000(P/A, 5%, 10) + 3(x)(P/A, 5%, 10) x = $103,000
Determine the NPW if TGGs MARR is 6%. Solution E(Savings) = .2(65,000) + .5(82,000) + .25(90,000) + .05(105,000) = $81,750 E(Salvage) = .2(40,000) + .5(55,000) + .25(65,000) + .05(75,000) = $55,500 NPW = -500,000 + 81,750(P/A, 6%, 8) + 55,500(P/F, 6%, 8) = $42,489
18-2 The two finalists in a tennis tournament are playing the championship. The winner will receive $60,000 and the runner-up $35,000. Determine the expected winnings for each participant if the players are considered to be evenly matched. What would the expected winnings be if one player were favored by 4 to 1 odds. Solution Evenly matched, both players expected winnings will be the same. Winnings = .5(60,000) + .5(35,000) = $47,500
243
244
Assume player A is favored by 4 to 1 odds. The probability that A wins is then 4/5. Player As expected winnings = .8(60,000) + .2(35,000) = $55,000 Player Bs expected winnings = .2(60,000) + .8(35,000) = $40,000
18-3 Consolidate Edison Power is evaluating the construction of a new electric generation facility. The two choices are a coal burning plant (CB) or a gaseous diffusion (GD) plant. The CB plant will cost $150 per megawatt to construct, and the GD plant will cost $300 per megawatt. Due to uncertainties concerning fuel availability and the impact of future air- and water- quality regulations, the useful life of each plant is unknown, but the following probability estimates have been made Probabilities Useful life (years) CB plant GD Plant 10 .10 .05 20 .50 .25 30 .30 .50 40 .10 .20 (a) Determine the expected life of each plant. (b) Based on the ratio of construction cost per megawatt to expected life, which plant would you recommend ConEd build? Solution (a) Expected life Coal burning = .10(10) + .50(20) + .30(30) + .10(40) = 24 years
Gaseous diffusion = .05(10) + .25(20) + .50(30) + .20(40) = 28.5 years (b) Ratios Coal burning = 150/24 = $6.25 per megawatt per year
Gaseous diffusion = 300/28.5 = $10.53 per megawatt per year Recommend the coal burning plant
245
18-4 Palmer Potatoes Chips Inc. must purchase new potato peeling equipment for its Martin, Tennessee plant. The plant engineer has determined there are three possible set-ups that can be purchased. Relevant data are present below. All machines are expected to be used six years and PPC Inc,s MARR is 10%. Which machine should be chosen? Naked Peel First Cost Annual Costs
Salvage Value
Salvage Value
Salvage Value
Solution Naked Peel E(Annual costs) = .2(3000) + .7(4500) + .1(5500) = $4,300 E(Salvage value) = .7(7500) + .3(9500) = $8,100 NPW = -45,000 - 4300(P/A, 10%, 6) + 8100(P/F, 10%, 6) = -$59,154 Skinner E(Annual costs) = $6,300 E(Salvage value) = $7,000 NPW = -$75,485
246
Peel-O-Matic E(Annual costs) = $7,150 E(Salvage value) = $8,700 NPW = -$102,227 Choose Naked Peel
18-5 Acme Insurance Company offers an insurance policy that pays $1,000 for lost luggage on a cruise. Historically the company pays this amount in 1 out of every 200 policies it sells. What is the minimum amount Acme must charge for such a policy if they desire to make at least $10 dollars per policy? Solution The probability that a loss occurs is
1 = .005 200
The expected loss to the company is therefore .005(1,000) = $5 To make a profit of $10 from each policy sold, they must charge $15 per policy
18-6 A roulette wheel consists of 18 black slots, 18 red slots, and 2 green slots. If a $100 bet is placed on black, what is the expected gain or loss? (A bet on black or red pays even money.) Solution The probability of black occurring =
18 38
247
18-7 Crush Cola Company must purchase a bottle capping machine. Information concerning the machine and possible cash flows is presented below. First Cost Annual Savings Annual Costs Actual Salvage Value p = .30 $40,000 2,000 7,000 4,000 p = .50 $40,000 3,500 5,000 5,000 p = .20 $40,000 5,000 4,000 6,500
The machine is expected to have a useful life of 10 years and straight-line depreciation to a salvage value of $5,000 will be used. Crush pays taxes at the marginal rate of 34% and has a MARR of 12%. Determine the NPW of the machine. Solution Depreciation =
40,000 ! 5000 = $3,500 10
E(Saving/Costs) = -5,000(.30) 1,500(.50) + 1,000(.20) = -$2,050 E(Salvage Value) = 4,000(.30) + 5,000(.50) + 6,500(.20) = $5,000
Taxable Year 0 1-10 10 BTCF -2,050 5,000 Depreciation 3,500 Income -5,550 Taxes -1,887 ATCF -40,000 -163 5,000
18-8 Krispy Kookies is considering the purchase of new dough mixing equipment. The NPW and estimated probabilities of the four possible outcomes are presented in the table below. Calculate the risk associated with this proposal. Outcome 1 2 3 4 NPW $34,560 38,760 42,790 52,330 Probability .15 .25 .40 .20
248
Solution
E(NPW) = .15(34,560) + .25(38,760) + .40(42,790) + .20(52,330) = $42,456 E(NPW2) = .15(34,5602) + .25(38,7602) + .40(42,7902) + .20(52,3302) = 1,834,822,860 StdDev(NPW) =
18-9 Northeast Airlines is considering bidding for a new route to Asia. The route is expected to command a rather large price. There is uncertainty associated with all elements of the investment. The winner of the route will have use of it for five years before the bidding process is repeated. Information developed by Northeast concerning estimated cash flows and probabilities is summarized below. Based on a MARR of 8% determine the risk associated with route.
NPW 1 = -11,500,000 + 3,000,000(P/A, 8%, 5) = $479,000 NPW 2 = -22,250,000 + 5,700,000(P/A, 8%, 5) = $510,100 NPW 3 = -27,500,000 + 7,000,000(P/A, 8%, 5) = $451,000 NPW 4 = -38,250,000 + 9,600,000(P/A, 8%, 5) = $ 82,800 E(NPW) = .15(479,000) + .45(510,100) + .30(451,000) + .10(82,800) = $444,975 E(NPW2) = .15(479,0002) + .45(510,1002) + .30(451,0002) + .10(82,8002) = 213,212,938,500 StdDev(NPW) =
18-10 A new heat exchanger must be installed by CSI Inc. Alternative A has an initial cost of $33,400 and alternative B has an initial cost of $47,500. Both alternatives are expected to last ten years. The annual cost of operating the heat exchanger depends on ambient temperature in the plant and energy costs. The estimate of the cost and probabilities for each alternative is presented below. If CSI has a MARR of 8% and uses rate of return analysis for all capital decisions, which exchanger should they purchase?
249
Alternative B E(Annual Cost) = .20(4,000) + .60(5,275) + .15(6,450) + .05(8,500) = $5,357.50 Incremental Analysis is required B-A NPW = 0 at IRR 0 = (-47,500 -(-33,400)) + (-5,375.50 - (-7112.50))(P/A, i%, 10) 0 = -14,100 + 1737(P/A, i%, 10) (P/A, i%, 10) = 8.12 i = 4% P/A = 8.111 IRR ! 4% CSI should purchase the least expensive alternative, A
250
18-11 The probability that a machine will last a certain number of years is given in the following table. Years of Life 10 11 12 13 14 15 Probability of Obtaining Life 0.15 0.20 0.25 0.20 0.15 0.05
What is the expected life of the machine? Solution Expected value = 10(0.15) + 11(0.20) + 12(0.25) + 13(0.20) + 14(0.15) + 15(0.05) = 12.15 years
D 4 4 *Rank order: 1 = worst and 4 = best Solution Site A = .40(1) Site B = .40(2.5) Site C = .40(2.5) Site D = .40(4) + .45(2) + .15(3) = 1.75 + .45(3) + .15(4) = 2.95 + .45(1) + .15(2) = 1.75 + .45(4) + .15(1) = 3.55
19-2 LowTech Inc. is currently studying alternatives to its present computer system. The NPW has a weight 3 times as great as the reliability of the system. Using the information below, which system is preferable? Is any system dominated? System SimTech Byte MBI Glitch Doorway NPW $5,000 4,500 7,500 6,000 3,500 Reliability High Very High Low Average Very low
251
252
Solution
System SimTech Byte MBI Glitch Doorway SimTech Byte MBI Glitch Doorway
NPW Rank 3 2 5 4 1
Reliability Rank 4 5 2 3 1
= .75(3) + .25(4) = 3.25 = .75(2) + .25(5) = 2.75 = .75(5) + .25(2) = 4.25 = .75(4) + .25(3) = 3.75 = .75(1) + .25(1) = 1.00
MBI is the preferred system based on maximizing selection criteria. Doorway is dominated.
19-3 Consolidate Edison Power is evaluating the construction of a new electric generation facility. The two choices are a coal burning plant (CB), a gaseous diffusion (GD) or a light nuclear reator (LNR) plant. The CB plant will cost $150 per megawatt to construct, GD plant will cost $300 per megawatt, and the LNR $450 per megawatt. Fuel availability and the impact of future air- and water- quality, public opinion, and construction costs are the factors of concern to ConEd in the decision as to which facility to build. The cost is weighted as 60% of the decision with the remaining factors evenly weighted. Determine the preferable plant to build based on these factors. Fuel Availability High High Medium Air-quality Low High Medium Water-quality High High Low Public Opinion Very low Medium Very low
Cost 3 2 1
Air-quality 1 3 2
253
19-4 Jill Smyth is trying to decide which university she should attend in the fall. She has decided there are several factors that she needs to consider. The universities she is considering are The University of Columbus, The University of Knoxville, The University of Lexington, and Louisville State University. She has decided to use the following factors and weights in her decision. Scholarship Amount 40% Academic Reputation 25% Success of Football Team 15% Party Reputation 20%
Factor Weight
Information for her four universities of choice is summarized below: Scholarship Amount University of Columbus $15,000 University of Knoxville 6,750 University of Lexington 10,000 Louisville State 4,575 *Record in most recent season. Help Jill make her decision. Solution Scholarship Academic Amount Reputation University of Columbus 4 4 University of Knoxville 2 1 University of Lexington 3 3 Louisville State 1 2 *Assumes excellent reputation is best. U of C = .4(4) + .25(4) + .15(4) + .20(3) = 3.80 U of K = .4(2) + .25(1) + .15(3) + .20(4) = 2.30 U of L = .4(3) + .25(3) + .15(1) + .20(2) = 2.50 LSU = .4(1) + .25(2) + .15(2) + .20(1) = 1.40 Based on Jills decision criteria she should attend the University of Columbus. Success of Football Team* 4 3 1 2 Party Reputation* 3 4 2 1 Academic Reputation Excellent Poor Above Average Average Success of Football Team* 13-0 7-6 3-10 4-7 Party Reputation Above Average Excellent Average Poor
254
19-5 The local fire department is in the process of choosing new tires for one of its trucks. The tires are rated on cost, expected life, and other issues. The two most important are cost and life. These factors need to be rated on a scale of 1 to 10. The best cost is considered $225, and 10 years is considered the best life. Develop the ratings for these two factors based on the following: Tire Wingedfoot HotRock Hooper BF Good ManyMiles Cost $250 350 300 325 225 Life 10 20 16 18 14
Solution Score(x) = 10(Worst value - x)/(Worst value - Best value) Cost Wingedfoot HotRock Hooper BF Goodmiles ManyMiles Life Wingedfoot HotRock Hooper BF Goodmiles ManyMiles Score = 10(10 - 10)/(10 - 20) = 0.00 Score = 10(10 - 20)/(10 - 20) = 10.00 Score = 10(10 - 16)/(10 - 20) = 6.00 Score = 10(10 - 18)/(10 - 20) = 8.00 Score = 10(10 - 14)/(10 - 20) = 4.00 Score = 10(350 - 250)/(350 - 225) = 8.00 Score = 10(350 - 350)/(350 - 225) = 0.00 Score = 10(350 - 300)/(350 - 225) = 4.00 Score = 10(350 - 325)/(350 - 225) = 2.00 Score = 10(350 - 225)/(350 - 225) = 10.00
A-2 Billy Bobs Towing and Repair Service has provided the following financial information: Cash Accounts receivable Accounts payable Securities Parts Inventories Prepaid expenses Accrued expense $80,000 120,000 200,000 75,000 42,000 30,000 15,000
Determine the (a) current ratio, (b) the quick ratio, and the (c) available working capital.
255
256
Solution
80,000 ! 120,000 ! 75,000 ! 42 ,000 200,000 ! 15,000 80,000 ! 120,000 ! 75,000 200,000 ! 15,000
= 1.47
= 1.28
A-3 The following financial information was taken from the income statement of Firerock Industries Revenues Sales Operating revenue Nonoperationg revenue Expenses Total operating expenses Interest payments
Taxes paid for the year equaled $110,000. Determine the (a) net income before taxes, (b) net profit (loss), (c) interest coverage and (d) net profit ratio. Solution (a) Net income before taxes = 3.2M + 2.0M + 3.4M - 6.7M - .5M = $1,400,000 (b) Net profit = 1.4M - .11M = $1,290,000 (c) Interest coverage =
Total Income Interest Payments
= 3.8
= .4031
257
A-5 Abby Manufacturing produces numerous childrens toys. The Dr. Dolittle Farm is one of the most popular sellers. Indirect cost to be allocated to production of the toy is to be calculated based on direct materials allocation. The total production overhead for the facility where the toy is produced is $750,000. The direct material total for the facility is $8,350,000. The cost of the direct materials used in production of the Dr. Dolittle is $7.45 per unit. The total labor (both direct and indirect) for the production of the toy is $9.35 per unit. The production schedule for the coming year calls for 300,000 units to be produced. If Abby desires a 35% profit on the toy, what should the wholesale price be? Solution Total labor Total materials Overhead = 9.35 x 300,000 = 2,805,000 = 7.45 x 300,000 = 2,235,000 = (2,235,000/8,350,000) x 750,000 = $200,748.50
Total production cost = 2,805,000 + 2,235,000 + 200,748.50 = $5,240,748.50 Cost per unit = 5,240,748.50/300,000 = $17.47 Wholesale price = 17.47(1.35) = $23.58
258
A-6 Brown Box Inc manufactures shipping boxes for a wide variety of industries. Their model XLLong has the following direct manufacturing costs per unit: Direct materials costs Direct labor costs $0.25 2.75
Overhead for the entire manufacturing plant is $4,000,000 per year. Direct labor costs are used to allocate the overhead. The total direct labor costs are estimated to be $5,500,000. The expected demand for this particular model is 200,000 boxes for the year. Determine the cost per unit. Solution Cost per unit = Direct materials cost + Direct labor costs + Overhead costs Overhead costs Direct labor cost = 200,000 x $2.75 = 550,000 Allocation of overhead = 4,000,000 x Cost per unit = .25 + 2.75 + 2.00 = $5.00
550,000 5,500,000
= 400,000/200,000 = $2/box
A-7 The following financial information is know about Rapid Delivery Inc.: Acid-test ratio Cash on hand Accounts receivable Market value of securities held Inventories Other assets Fixed assets Total liabilities 1.3867 $ 72,000 102,000 34,000 143,000 16,000 215,000 400,000
Determine the (a) current assets, (b) current liabilities, (c) total assets, and (d) owners equity.
259
Current liabilities =
= $149,996
(c) Total assets = 351,000 + 215,000 + 16,000 = $582,000 (d) Owners equity = Total assets - Total liabilities = 582,000 - 400,000 = $182,000
A-8 Determine the current and quick ratios for Harbor Master Boats Inc. Does the company appear to be reasonably sound from a financial viewpoint based on these two ratios? Harbor Master Boats Inc. Balance Sheet, January 1, 200X Assets Current Assets Cash Accounts Receivable Inventory Total Current Assets Fixed Assets Land Plant Equipment Total Fixed Assets Liabilities Current Liabilities Accounts Payable Notes Payable Accrued Expense Total Current Liabilities Long Term Debt Total Liabilities Equity Stock Retained Earnings Total Net Worth Total Liabilities and Net Worth
2,400,000 2,000,000 900,000 5,300,000 3,000,000 8,300,000 2,000,000 2,500,000 4,500,000 12,800,000
Total Assets
12,800,000
260
Solution
Current ratio =
4,000,000 5,300,000
= .755
Quick ratio =
= .377
Based on these two ratios the company is not in very sound financially. The current typically should be greater than 2. The quick ratio indicates the companys inability to pay off current liabilities with quick capital.
APPENDIX B
559
560
1
/4%
/4%
561
1
/2% Single Payment Compound Amount Factor Find F Given P F/P 1.005 1.010 1.015 1.020 1.025 1.030 1.036 1.041 1.046 1.051 1.056 1.062 1.067 1.072 1.078 1.083 1.088 1.094 1.099 1.105 1.110 1.116 1.122 1.127 1.133 1.138 1.144 1.150 1.156 1.161 1.197 1.221 1.270 1.283 1.296 1.349 1.418 1.432 1.490 1.520 1.567 1.614 1.647 1.680 1.819 3.310 6.023 10.957 Present Worth Factor Find P Given F P/F .9950 .9901 .9851 .9802 .9754 .9705 .9657 .9609 .9561 .9513 .9466 .9419 .9372 .9326 .9279 .9233 .9187 .9141 .9096 .9051 .9006 .8961 .8916 .8872 .8828 .8784 .8740 .8697 .8653 .8610 .8356 .8191 .7871 .7793 .7716 .7414 .7053 .6983 .6710 .6577 .6383 .6195 .6073 .5953 .5496 .3021 .1660 .0913 Sinking Fund Factor Find A Given F A/F 1.0000 .4988 .3317 .2481 .1980 .1646 .1407 .1228 .1089 .0978 .0887 .0811 .0746 .0691 .0644 .0602 .0565 .0532 .0503 .0477 .0453 .0431 .0411 .0393 .0377 .0361 .0347 .0334 .0321 .0310 .0254 .0226 .0185 .0177 .0169 .0143 .0120 .0116 .0102 .00961 .00883 .00814 .00773 .00735 .00610 .00216 .00100 .00050
Compound Interest Factors Uniform Payment Series Capital Recovery Factor Find A Given P A/P 1.0050 .5038 .3367 .2531 .2030 .1696 .1457 .1278 .1139 .1028 .0937 .0861 .0796 .0741 .0694 .0652 .0615 .0582 .0553 .0527 .0503 .0481 .0461 .0443 .0427 .0411 .0397 .0384 .0371 .0360 .0304 .0276 .0235 .0227 .0219 .0193 .0170 .0166 .0152 .0146 .0138 .0131 .0127 .0124 .0111 .00716 .00600 .00550 Compound Amount Factor Find F Given A F/A 1.000 2.005 3.015 4.030 5.050 6.076 7.106 8.141 9.182 10.228 11.279 12.336 13.397 14.464 15.537 16.614 17.697 18.786 19.880 20.979 22.084 23.194 24.310 25.432 26.559 27.692 28.830 29.975 31.124 32.280 39.336 44.159 54.098 56.645 59.218 69.770 83.566 86.409 98.068 104.074 113.311 122.829 129.334 135.970 163.880 462.041 1 004.5 1 991.5 Present Worth Factor Find P Given A P/A 0.995 1.985 2.970 3.951 4.926 5.896 6.862 7.823 8.779 9.730 10.677 11.619 12.556 13.489 14.417 15.340 16.259 17.173 18.082 18.987 19.888 20.784 21.676 22.563 23.446 24.324 25.198 26.068 26.933 27.794 32.871 36.172 42.580 44.143 45.690 51.726 58.939 60.340 65.802 68.453 72.331 76.095 78.543 80.942 90.074 139.581 166.792 181.748 Arithmetic Gradient Gradient Uniform Series Find A Given G A/G 0 0.499 0.996 1.494 1.990 2.486 2.980 3.474 3.967 4.459 4.950 5.441 5.931 6.419 6.907 7.394 7.880 8.366 8.850 9.334 9.817 10.300 10.781 11.261 11.741 12.220 12.698 13.175 13.651 14.127 16.962 18.836 22.544 23.463 24.378 28.007 32.468 33.351 36.848 38.576 41.145 43.685 45.361 47.025 53.551 96.113 128.324 151.795 Gradient Present Worth Find P Given G P/G 0 0.991 2.959 5.903 9.803 14.660 20.448 27.178 34.825 43.389 52.855 63.218 74.465 86.590 99.574 113.427 128.125 143.668 160.037 177.237 195.245 214.070 233.680 254.088 275.273 297.233 319.955 343.439 367.672 392.640 557.564 681.341 959.928 1 035.70 1 113.82 1 448.65 1 913.65 2 012.35 2 424.65 2 640.67 2 976.08 3 324.19 3 562.80 3 806.29 4 823.52 13 415.56 21 403.32 27 588.37
/2%
562
3
/4%
/4%
563
1%
564
1 1/4%
565
1 1/2%
566
1 3/4%
567
2%
568
2 1/2%
n 1 2 3 4 5 6 7 8 9 10 11 12 13 14 15 16 17 18 19 20 21 22 23 24 25 26 27 28 29 30 31 32 33 34 35 40 45 50 55 60 65 70 75 80 85 90 95 100
n 1 2 3 4 5 6 7 8 9 10 11 12 13 14 15 16 17 18 19 20 21 22 23 24 25 26 27 28 29 30 31 32 33 34 35 40 45 50 55 60 65 70 75 80 85 90 95 100
569
3%
n 1 2 3 4 5 6 7 8 9 10 11 12 13 14 15 16 17 18 19 20 21 22 23 24 25 26 27 28 29 30 31 32 33 34 35 40 45 50 55 60 65 70 75 80 85 90 95 100
n 1 2 3 4 5 6 7 8 9 10 11 12 13 14 15 16 17 18 19 20 21 22 23 24 25 26 27 28 29 30 31 32 33 34 35 40 45 50 55 60 65 70 75 80 85 90 95 100
570
3 1/2%
n 1 2 3 4 5 6 7 8 9 10 11 12 13 14 15 16 17 18 19 20 21 22 23 24 25 26 27 28 29 30 31 32 33 34 35 40 45 50 55 60 65 70 75 80 85 90 95 100
n 1 2 3 4 5 6 7 8 9 10 11 12 13 14 15 16 17 18 19 20 21 22 23 24 25 26 27 28 29 30 31 32 33 34 35 40 45 50 55 60 65 70 75 80 85 90 95 100
571
4%
n 1 2 3 4 5 6 7 8 9 10 11 12 13 14 15 16 17 18 19 20 21 22 23 24 25 26 27 28 29 30 31 32 33 34 35 40 45 50 55 60 65 70 75 80 85 90 95 100
n 1 2 3 4 5 6 7 8 9 10 11 12 13 14 15 16 17 18 19 20 21 22 23 24 25 26 27 28 29 30 31 32 33 34 35 40 45 50 55 60 65 70 75 80 85 90 95 100
572
4 1/2%
n 1 2 3 4 5 6 7 8 9 10 11 12 13 14 15 16 17 18 19 20 21 22 23 24 25 26 27 28 29 30 31 32 33 34 35 40 45 50 55 60 65 70 75 80 85 90 95 100
n 1 2 3 4 5 6 7 8 9 10 11 12 13 14 15 16 17 18 19 20 21 22 23 24 25 26 27 28 29 30 31 32 33 34 35 40 45 50 55 60 65 70 75 80 85 90 95 100
573
5%
n 1 2 3 4 5 6 7 8 9 10 11 12 13 14 15 16 17 18 19 20 21 22 23 24 25 26 27 28 29 30 31 32 33 34 35 40 45 50 55 60 65 70 75 80 85 90 95 100
n 1 2 3 4 5 6 7 8 9 10 11 12 13 14 15 16 17 18 19 20 21 22 23 24 25 26 27 28 29 30 31 32 33 34 35 40 45 50 55 60 65 70 75 80 85 90 95 100
574
6%
n 1 2 3 4 5 6 7 8 9 10 11 12 13 14 15 16 17 18 19 20 21 22 23 24 25 26 27 28 29 30 31 32 33 34 35 40 45 50 55 60 65 70 75 80 85 90 95 100
n 1 2 3 4 5 6 7 8 9 10 11 12 13 14 15 16 17 18 19 20 21 22 23 24 25 26 27 28 29 30 31 32 33 34 35 40 45 50 55 60 65 70 75 80 85 90 95 100
575
7%
n 1 2 3 4 5 6 7 8 9 10 11 12 13 14 15 16 17 18 19 20 21 22 23 24 25 26 27 28 29 30 31 32 33 34 35 40 45 50 55 60 65 70 75 80 85 90 95 100
n 1 2 3 4 5 6 7 8 9 10 11 12 13 14 15 16 17 18 19 20 21 22 23 24 25 26 27 28 29 30 31 32 33 34 35 40 45 50 55 60 65 70 75 80 85 90 95 100
576
8%
n 1 2 3 4 5 6 7 8 9 10 11 12 13 14 15 16 17 18 19 20 21 22 23 24 25 26 27 28 29 30 31 32 33 34 35 40 45 50 55 60 65 70 75 80 85 90 95 100
n 1 2 3 4 5 6 7 8 9 10 11 12 13 14 15 16 17 18 19 20 21 22 23 24 25 26 27 28 29 30 31 32 33 34 35 40 45 50 55 60 65 70 75 80 85 90 95 100
577
9%
n 1 2 3 4 5 6 7 8 9 10 11 12 13 14 15 16 17 18 19 20 21 22 23 24 25 26 27 28 29 30 31 32 33 34 35 40 45 50 55 60 65 70 75 80 85 90 95 100
n 1 2 3 4 5 6 7 8 9 10 11 12 13 14 15 16 17 18 19 20 21 22 23 24 25 26 27 28 29 30 31 32 33 34 35 40 45 50 55 60 65 70 75 80 85 90 95 100
578
10%
n 1 2 3 4 5 6 7 8 9 10 11 12 13 14 15 16 17 18 19 20 21 22 23 24 25 26 27 28 29 30 31 32 33 34 35 40 45 50 55 60 65 70 75 80 85 90 95 100
n 1 2 3 4 5 6 7 8 9 10 11 12 13 14 15 16 17 18 19 20 21 22 23 24 25 26 27 28 29 30 31 32 33 34 35 40 45 50 55 60 65 70 75 80 85 90 95 100
579
12%
n 1 2 3 4 5 6 7 8 9 10 11 12 13 14 15 16 17 18 19 20 21 22 23 24 25 26 27 28 29 30 31 32 33 34 35 40 45 50 55 60 65 70 75 80 85 90 95 100
n 1 2 3 4 5 6 7 8 9 10 11 12 13 14 15 16 17 18 19 20 21 22 23 24 25 26 27 28 29 30 31 32 33 34 35 40 45 50 55 60 65 70 75 80 85 90 95 100
580
15%
n 1 2 3 4 5 6 7 8 9 10 11 12 13 14 15 16 17 18 19 20 21 22 23 24 25 26 27 28 29 30 31 32 33 34 35 40 45 50 55 60 65 70 75 80 85
n 1 2 3 4 5 6 7 8 9 10 11 12 13 14 15 16 17 18 19 20 21 22 23 24 25 26 27 28 29 30 31 32 33 34 35 40 45 50 55 60 65 70 75 80 85
581
18%
n 1 2 3 4 5 6 7 8 9 10 11 12 13 14 15 16 17 18 19 20 21 22 23 24 25 26 27 28 29 30 31 32 33 34 35 40 45 50 55 60 65 70
n 1 2 3 4 5 6 7 8 9 10 11 12 13 14 15 16 17 18 19 20 21 22 23 24 25 26 27 28 29 30 31 32 33 34 35 40 45 50 55 60 65 70
582
20%
n 1 2 3 4 5 6 7 8 9 10 11 12 13 14 15 16 17 18 19 20 21 22 23 24 25 26 27 28 29 30 31 32 33 34 35 40 45 50 55 60
n 1 2 3 4 5 6 7 8 9 10 11 12 13 14 15 16 17 18 19 20 21 22 23 24 25 26 27 28 29 30 31 32 33 34 35 40 45 50 55 60
583
25%
n 1 2 3 4 5 6 7 8 9 10 11 12 13 14 15 16 17 18 19 20 21 22 23 24 25 26 27 28 29 30 31 32 33 34 35 40 45 50 55
n 1 2 3 4 5 6 7 8 9 10 11 12 13 14 15 16 17 18 19 20 21 22 23 24 25 26 27 28 29 30 31 32 33 34 35 40 45 50 55
584
30%
n 1 2 3 4 5 6 7 8 9 10 11 12 13 14 15 16 17 18 19 20 21 22 23 24 25 26 27 28 29 30 31 32 33 34 35 40 45
n 1 2 3 4 5 6 7 8 9 10 11 12 13 14 15 16 17 18 19 20 21 22 23 24 25 26 27 28 29 30 31 32 33 34 35 40 45
585
35%
n 1 2 3 4 5 6 7 8 9 10 11 12 13 14 15 16 17 18 19 20 21 22 23 24 25 26 27 28 29 30 31 32 33 34 35
n 1 2 3 4 5 6 7 8 9 10 11 12 13 14 15 16 17 18 19 20 21 22 23 24 25 26 27 28 29 30 31 32 33 34 35
586
40%
n 1 2 3 4 5 6 7 8 9 10 11 12 13 14 15 16 17 18 19 20 21 22 23 24 25 26 27 28 29 30 31 32 33 34 35
n 1 2 3 4 5 6 7 8 9 10 11 12 13 14 15 16 17 18 19 20 21 22 23 24 25 26 27 28 29 30 31 32 33 34 35
587
45%
n 1 2 3 4 5 6 7 8 9 10 11 12 13 14 15 16 17 18 19 20 21 22 23 24 25 26 27 28 29 30 31 32 33 34 35
n 1 2 3 4 5 6 7 8 9 10 11 12 13 14 15 16 17 18 19 20 21 22 23 24 25 26 27 28 29 30 31 32 33 34 35
588
50%
n 1 2 3 4 5 6 7 8 9 10 11 12 13 14 15 16 17 18 19 20 21 22 23 24 25 26 27 28 29 30 31 32
n 1 2 3 4 5 6 7 8 9 10 11 12 13 14 15 16 17 18 19 20 21 22 23 24 25 26 27 28 29 30 31 32
589
60%
n 1 2 3 4 5 6 7 8 9 10 11 12 13 14 15 16 17 18 19 20 21 22 23 24 25 26 27 28
n 1 2 3 4 5 6 7 8 9 10 11 12 13 14 15 16 17 18 19 20 21 22 23 24 25 26 27 28
590
rn .01 .02 .03 .04 .05 .06 .07 .08 .09 .10 .11 .12 .13 .14 .15 .16 .17 .18 .19 .20 .21 .22 .23 .24 .25 .26 .27 .28 .29 .30 .31 .32 .33 .34 .35 .36 .37 .38 .39 .40 .41 .42 .43 .44 .45 .46 .47 .48 .49 .50
rn .51 .52 .53 .54 .55 .56 .57 .58 .59 .60 .61 .62 .63 .64 .65 .66 .67 .68 .69 .70 .71 .72 .73 .74 .75 .76 .77 .78 .79 .80 .81 .82 .83 .84 .85 .86 .87 .88 .89 .90 .91 .92 .93 .94 .95 .96 .97 .98 .99 1.00